Intraining Exam 2019

Ace your homework & exams now with Quizwiz!

A 63-year-old female sees you for evaluation of recurrent right foot swelling and redness. She has a history of obesity and type 2 diabetes with retinopathy, nephropathy, and peripheral neuropathy. She presented with similar symptoms 2 weeks ago and was diagnosed with cellulitis and treated with a 10-day course of amoxicillin/clavulanate (Augmentin). Her symptoms seemed to initially improve with this therapy along with elevation of the foot but then worsened. She does not have any pain in the foot, fever, or chills. She does not recall any trauma or other inciting event. The patient's vital signs include a temperature of 37.1°C (98.8°F), a pulse rate of 72 beats/min, and a blood pressure of 124/82 mm Hg. Her right foot appears swollen, red, and warm to the touch, and is not tender to palpation. There are no open sores or calluses. Her dorsalis pedis pulse is 2+. Monofilament testing confirms a diagnosis of peripheral neuropathy. A WBC count is normal. Radiographs reveal soft-tissue edema with no other abnormalities. The most appropriate treatment at this point would be A) immobilization B) antibiotics C) bisphosphonates D) corticosteroids E) surgical repair

A. This patient has acute Charcot neuroarthropathy, an inflammatory condition that occurs in obese patients with peripheral neuropathy and ultimately leads to foot deformities (the classic rocker-bottom foot) and resultant ulcerations and infections. Its clinical appearance can easily be initially mistaken for cellulitis. However, the absence of tenderness and other signs of infection such as fever, an elevated WBC count, and inflammatory markers is not consistent with cellulitis. Radiography is an appropriate initial imaging modality but the results are often interpreted as normal early in the disease process. MRI is the modality of choice for a definitive diagnosis and may demonstrate periarticular bone marrow edema, adjacent soft-tissue edema, joint effusion, and microtrabecular or stress fractures. The treatment of acute Charcot neuroarthropathy is immobilization with total contact casting, which increases the total surface area of contact to the entire lower extremity, distributing pressure away from the foot. Immobilization is typically required for at least 3-4 months but in some cases may be needed for up to 12 months. Bisphosphonates were found to be ineffective as adjunctive therapy in acute Charcot neuroarthropathy. Corticosteroids and antibiotics have no role in the treatment of Charcot foot but would be appropriate therapy for cellulitis or gout, which are important alternative diagnoses to consider. The role of surgery is more controversial but may be indicated in the acute phase of Charcot neuroarthropathy in patients with severe dislocation or instability.

A 17-year-old male presents with right lateral ankle pain after twisting his ankle in a basketball game earlier today. The area is mildly swollen. A radiograph shows soft-tissue edema but no evidence of fracture. Which one of the following would be the most appropriate management of this condition? A) Use of a functional ankle brace with early mobilization B) Prolonged use of a patellar tendon strap to help unload the ankle joint C) Immobilization with compression taping for a minimum of 2 weeks D) Referral to physical therapy for neuromuscular retraining

ANSWER: A Early mobilization leads to better outcomes with ankle sprains, and using a functional ankle brace such as a semi-rigid air stirrup brace or a soft lace-up ankle brace will protect the ankle from inversion and eversion sprains while still allowing for mobility during physical activity. These braces lead to improved functional outcomes at 1 month when compared to elastic compression taping. Patellar tendon straps relieve the pain associated with patellar tendinopathy but are not indicated with acute ankle sprains. Use of a semi-rigid air stirrup splint has been shown to be superior to neuromuscular training for prevention of recurrent ankle sprains.

Which one of the following calculated values is most useful for evaluating the effectiveness of a clinical intervention? A) Number needed to treat B) Relative risk reduction C) Specificity D) 95% confidence interval E) P-value

ANSWER: A A basic understanding of statistics is essential to evaluate clinical evidence. The number needed to treat, which represents the number of patients that must be treated to result in one positive outcome, is one of the most intuitive statistical concepts in explaining the power or relative lack thereof of an intervention. Relative risk reduction in particular can be misleading, since a seemingly large relative risk reduction may result in a very small absolute risk reduction if the prevalence of the disorder in question is low. Specificity indicates the ability of a test to accurately identify a condition but does not indicate the effectiveness of an intervention. A 95% confidence interval relates to statistical variation and the relationship of an outcome to chance. P-value deals with statistical significance but not magnitude of the effect.

A 17-year-old female presents to your office with open and closed comedones on her nose, forehead, and chin. No cystic lesions are noted and the acne does not extend to her back or chest. Of the following, which one would be most effective? A) Topical adapalene (Differin) and benzoyl peroxide B) Topical clindamycin (Cleocin T) C) Topical salicylic acid D) Topical tazarotene (Tazorac) E) Oral norethindrone/ethinyl estradiol

ANSWER: A Acne affects 85% of 12- to 25-year-olds in the United States. This patient has mild acne as evidenced by the absence of cystic lesions and localization to the facial T-zone. Topical retinoids are first-line treatment for any level of severity of acne (SOR A). Adapalene is an effective retinoid and is available over-the-counter. Benzoyl peroxide is also very effective in the control of acne because it reduces the concentration of cystic acne with no risk of bacterial resistance. The combination of a topical retinoid and benzoyl peroxide is more effective than either agent alone. Topical antibiotics such as clindamycin and erythromycin should not be used as monotherapy due to high rates of microbial resistance. There is little evidence that salicylic acid is effective in combating acne despite its widespread use. Tazarotene is effective in the treatment of acne but is teratogenic (pregnancy category X) and should be avoided in women of reproductive age. Combined oral contraceptives can be effective, but norethindrone and other first-generation progestins are androgenic and can worsen acne.

A 67-year-old male presents for a Medicare wellness visit. He underwent basic laboratory work prior to the office visit. He is feeling well and does not have any concerns or symptoms. His blood pressure is 127/76 mm Hg, his heart rate is 64 beats/min, and he is afebrile. A comprehensive metabolic panel is unremarkable. A CBC shows the following results: WBCs 7500/mm3 (N 4100-10,900) RBCs 4.05 million/mm3 (N 4.70-6.10) Hemoglobin 12.9 g/dL (N 14.0-18.0) Hematocrit 39% (N 42-52) Mean corpuscular volume 82 m 3 (N 80-95) Platelets 197,000/mm3 (N 130,000-448,000) Which one of the following would be the most appropriate next step in the workup of this patient? A) A serum ferritin level B) A serum transferrin receptor-ferritin index C) Oral iron supplementation, and a repeat CBC in 4 weeks D) Referral to a gastroenterologist

ANSWER: A Anemia is often diagnosed incidentally on laboratory testing and is often asymptomatic. It is associated with increased morbidity and mortality in older adults, and is often caused by nutritional deficiencies, chronic kidney disease, occult blood loss from gastrointestinal malignancies, or chronic inflammation. However, in many patients the cause remains unknown. A detailed history and physical examination are indicated. In patients with normocytic or microcytic anemia, a serum ferritin level should be ordered. A low serum ferritin level is associated with iron deficiency and should be further evaluated so the underlying cause can be addressed. A serum transferrin-receptor-ferritin index should be determined for patients with a serum ferritin level between 46 and 100 ng/mL to distinguish between iron deficiency anemia and other types of anemia. Referring this patient to a gastroenterologist would not be indicated at this time.

A 64-year-old female with a long-standing history of COPD presents with a 1-day history of dizziness and worsening shortness of breath. An EKG is shown below. Which one of the following is the most likely cause of her symptoms? A) Atrial fibrillation B) Multifocal atrial tachycardia C) Myocardial infarction D) Pulmonary embolism

ANSWER: A Atrial fibrillation is characterized by an irregularly irregular rhythm, without any discernible P waves. In multifocal atrial tachycardia the P waves show varying morphology, and this rhythm is typically seen in patients with underlying lung disease, especially COPD. Acute ST-segment elevation myocardial infarction is characterized by the presence of hyperacute ST-segment and T-wave changes in contiguous EKG leads, accompanied by reciprocal changes. Pulmonary embolism is characterized by EKG findings of sinus tachycardia (a heart rate 100 beats/min), nonspecific ST-T changes, and new-onset right bundle branch block (SOR C).

A 60-year-old female has recently moved to your area from another state and sees you to establish care. Her past records are not available at the time of her visit but she has a history of ER+/PR+ breast cancer diagnosed 2 years ago. She was treated with lumpectomy, radiation, and chemotherapy. She does not have any current symptoms and there is no family history of breast or ovarian cancer. Her last visit with a physician was 6 months ago. She had a mammogram at that time. A breast MRI was done at the time of her cancer diagnosis. Her last Papanicolaou (Pap) test was normal 1 year ago. Which one of the following should be performed at this time? A) A history and physical examination only B) A Pap test C) Mammography D) Breast MRI E) Echocardiography

ANSWER: A Breast cancer survivors should undergo a history and physical examination every 3-6 months for the first 3 years after treatment, then every 6-12 months for the next 2 years, and then annually thereafter (SOR C). Papanicolaou testing guidelines do not change for patients with a history of breast cancer. Screening should be repeated every 3-5 years according to American Society for Colposcopy and Cervical Pathology (ASCCP) guidelines. Mammograms of both breasts or the remaining breast are recommended no more often than yearly (SOR A). Breast MRI is not recommended on a regular basis unless the patient has a high risk of recurrence, a significant family history of breast or ovarian cancer, or a personal history of Hodgkin's disease (SOR C). While it is important to be alert for signs of cardiotoxicity due to prior chemotherapy, echocardiography is indicated only if the patient has cardiac symptoms. Routine echocardiography is not recommended (SOR C).

A 62-year-old female with stage 3 chronic kidney disease and an estimated glomerular filtration rate of 37 mL/min/1.73 m2 is found to have a mildly low ionized calcium level. Which one of the following would you expect to see if her hypocalcemia is secondary to her chronic kidney disease? A) Elevated parathyroid hormone (PTH) and elevated phosphorus B) Elevated PTH and low phosphorus C) Low PTH and elevated phosphorus D) Low PTH and low phosphorus

ANSWER: A Chronic kidney disease-mineral and bone disorder (CKD-MBD) is found in many patients with CKD and is associated with an increased risk of bone fractures and cardiovascular events due to vascular calcification. In patients with CKD, phosphate is not appropriately excreted and the subsequent hyperphosphatemia leads to secondary hyperparathyroidism and binding of calcium. Decreased production of calcitriol in patients with CKD also leads to hypocalcemic hyperparathyroidism. Patients with CKD stages 3a-5 should have phosphorus, calcium, parathyroid hormone, and 25-hydroxyvitamin D levels checked regularly, and consultation with a nephrologist or endocrinologist should be obtained if CKD-MBD is suspected

A 58-year-old male sees you for follow-up of diabetic gastroparesis. He has tried managing his symptoms with more frequent meals and taking in more calories in semisolid or liquid form. These approaches have been unsuccessful in controlling his symptoms and he would like to try a medication. Which one of the following would be considered first-line pharmacotherapy for this patient? A) Metoclopramide (Reglan) B) Nortriptyline (Pamelor) C) Omeprazole (Prilosec) D) Ondansetron (Zofran) E) Ranitidine (Zantac)

ANSWER: A Diabetic gastroparesis is a delay in the emptying of food from the upper gastrointestinal tract in the absence of a mechanical obstruction of the stomach or duodenum. Metoclopramide is the only prokinetic agent that has been studied specifically for long-term use in gastroparesis and is considered first-line therapy (SOR B). It is among the only FDA-approved medications for gastroparesis. Nortriptyline is a prokinetic agent but has not been shown to be more effective than placebo for decreasing gastroparesis symptoms. Proton pump inhibitors such as omeprazole, histamine H2-receptor antagonists such as ranitidine, and ondansetron delay gastric emptying and should be withheld in patients with gastroparesis whenever possible.

An 84-year-old male nursing home resident with dementia is noted to have a weight loss of about 5% in the past 6 months. Which one of the following would be most appropriate? A) Avoiding dietary restrictions B) An appetite stimulant C) Vitamin B12, vitamin D, and selenium supplements D) An omega-3 fatty acid supplement E) Tube feeding

ANSWER: A Effective interventions for weight loss in nursing home patients include providing meals in a pleasant, home-like environment. Avoiding dietary restrictions has low quality evidence of effectiveness. There is high quality evidence that initiating tube feedings in patients with severe dementia is not only ineffective but may lead to problems such as decubitus ulcers and aspiration. There is low to very low evidence of the effectiveness for prescribing appetite stimulants, selenium, vitamin B, or vitamin D supplements unless there is a documented deficiency. Neither quality of life nor survival is improved.

A 56-year-old male with a history of hepatitis C cirrhosis is admitted to the hospital with gastrointestinal (GI) bleeding. The patient has been stable, taking only furosemide (Lasix) and spironolactone (Aldactone). Upper GI endoscopy confirms variceal bleeding and the gastroenterologist performs appropriate variceal banding. A nurse calls you because laboratory studies that were ordered in the emergency department reveal a serum ammonia level of 120 g/dL (N 39-90). The patient has no signs of confusion, insomnia, or decreased mental alertness. A physical examination reveals mild ascites but no other abnormalities. Which one of the following would be most appropriate for addressing the elevated ammonia level? A) No additional treatment B) Lactulose (Kristalose) C) Methotrexate D) Neomycin E) Prednisone

ANSWER: A Elevated ammonia levels may occur with urea cycle disorders, portosystemic shunting, urinary tract infection from urease-producing organisms, gastrointestinal bleeding, shock, renal disease, heavy exercise, smoking, parenteral nutrition, salicylate intoxication, use of medications including diuretics, and alcohol use. In patients with chronic liver disease, hepatic encephalopathy is diagnosed based on the overall clinical presentation and not on ammonia levels. A normal ammonia level does not exclude the diagnosis of hepatic encephalopathy, nor does an elevated ammonia level establish the diagnosis. This patient had an elevated serum ammonia level that was found incidentally during his hospital admission for gastrointestinal bleeding. Because there was no clinically significant encephalopathy, treatment based on ammonia levels is not indicated. The patient's elevated ammonia level was probably from diuretic use. Lactulose, methotrexate, neomycin, or prednisone would not be appropriate.

You are evaluating a 2-year-old white male who was seen in the emergency department 1 week ago for a generalized seizure. The seizure lasted approximately 45 seconds and was followed by a postictal state, but there have been no recurrences. A CBC, electrolyte levels, MRI of the brain, and an EEG were all normal. The parents of the child ask your advice regarding drug therapy. You should inform the parents that antiepileptic drugs A) are not indicated at this time B) should be started with a single drug C) should be started with drugs from two different classes D) have few adverse effects

ANSWER: A For patients who experience a seizure, the risk factors for recurrent seizures include two unprovoked seizures more than 24 hours apart, epileptiform abnormalities on an EEG, abnormal brain imaging results, severe head trauma, and a syndrome associated with epilepsy. In children who do not have any of these risk factors, antiepileptic drug therapy is not indicated after a first unprovoked seizure. When there are no risk factors there is no difference in 1- to 2-year seizure remission rates if antiepileptic drug therapy is started after a second seizure as opposed to starting it after the first seizure. If therapy is indicated, monotherapy should be the initial approach. The risk of adverse effects of drug therapy are as high as 50%. This includes subtle cognitive and behavioral effects in many cases.

A 60-year-old patient is admitted to the hospital for a COPD exacerbation. For this patient, which one of the following would be the most appropriate prednisone dosage? A) 40 mg daily for 5 days B) 40 mg daily for 10 days C) 60 mg daily, tapered over 6 days D) 60 mg daily for 10 days

ANSWER: A For patients with a COPD exacerbation, systemic glucocorticoids can improve FEV1, improve oxygenation, shorten recovery time, and reduce the length of hospitalization (level of evidence A). Prednisone, 40 mg daily for 5 days, is recommended for COPD exacerbations. Studies have shown that oral administration is equally efficacious compared to the intravenous route. The duration of therapy should not be longer than 5-7 days (level of evidence A).

According to U.S. Preventive Services Task Force guidelines, which one of the following interventions is recommended after 12 weeks gestation in women who are at high risk for preeclampsia? A) Aspirin, 81 mg daily B) Magnesium gluconate, 1500 mg daily C) Fish oil, 1000 mg daily D) Vitamin C, 1000 mg daily E) Vitamin D, 2000 IU daily

ANSWER: A For women at high risk of developing preeclampsia, the U.S. Preventive Services Task Force (USPSTF) recommends starting low-dose aspirin after 12 weeks gestation (B recommendation). While calcium appears to be helpful in preventing preeclampsia for women with a diet deficient in calcium, the evidence is not yet conclusive. The USPSTF recommendation does not address the use of fish oil, magnesium gluconate, vitamin C, or vitamin D for the prevention of preeclampsia

A 25-year-old female was involved in a motor vehicle accident 2 weeks ago. A chest radiograph to assess for rib fractures revealed bilateral hilar lymphadenopathy. She thinks that her mother had a similar finding when she was younger. Records from the emergency department reveal that a CBC, comprehensive metabolic panel, and urinalysis were all normal. The patient has never been sexually active, does not take any medications, and does not smoke or use any illicit drugs. Her rib pain has since resolved and she has no other symptoms. She does not have a cough, dyspnea, weight loss, or skin lesions. Spirometry in the office today is normal. Which one of the following would be the most appropriate next step? A) A follow-up visit and a repeat chest radiograph in 6 months B) Oral prednisone, 40 mg daily for 4 weeks C) CT of the chest, abdomen, and pelvis D) Formal pulmonary function tests E) Referral for bronchoscopy with a biopsy

ANSWER: A Given this patient's age, lack of symptoms, and possible family history, the presence of asymptomatic bilateral hilar lymphadenopathy most likely represents stage 1 pulmonary sarcoidosis. Because the patient does not have any symptoms and stage 1 sarcoidosis resolves in most cases, the most prudent course is to reevaluate her in 6 months with a careful history, a physical examination, and a chest radiograph. Given the normal spirometry results, pulmonary function tests are not needed at this time. Neither CT nor a lung biopsy would change management at this time. Treatment is not indicated in stage 1 sarcoidosis but would be merited if she developed increasing pulmonary symptoms or any extrapulmonary symptoms.

A 28-year-old male presents to your office in January with a 1-day history of cough and nasal congestion. He has not had any fever, shortness of breath, or chest pain. An examination reveals some rhinorrhea and hoarseness. A lung examination is normal. The patient asks you to prescribe albuterol (Proventil, Ventolin) because it seemed to help a friend who had similar symptoms. Which one of the following would be the most appropriate evidence-based response to the patient's request? A) Do not prescribe albuterol B) Prescribe albuterol alone C) Prescribe albuterol and oral corticosteroids D) Prescribe albuterol and inhaled corticosteroids

ANSWER: A In a Cochrane review of five trials of 2-agonist therapy in adults, there was no significant difference between 2-agonists and placebo in cough reduction. Based on reports of adverse effects, the number needed to harm is 2. It is important to set reasonable expectations for cough duration after an acute respiratory illnes

A 55-year-old patient with a history of alcoholism is admitted through the emergency department with acute pancreatitis. Which one of the following tests performed at the time of admission can best predict the severity of pancreatitis? A) Hematocrit B) C-reactive protein C) Serum amylase D) Serum lipase E) CT of the abdomen

ANSWER: A Knowing the severity of pancreatitis helps predict how aggressive management should be. Hematocrit, BUN, and creatinine levels are the most useful predictors of the severity of pancreatitis, reflecting the degree of intravascular volume depletion. C-reactive protein is often elevated, but it is not as useful as hematocrit for predicting severity. Serum amylase and lipase have no prognostic value. CT evidence of severe pancreatitis lags behind clinical and laboratory evidence, and early CT underestimates the severity of the acute process

An 80-year-old former smoker sees you for a 6-month follow-up for hypertension. He is taking carvedilol (Coreg), amlodipine (Norvasc), and low-dose aspirin. His home blood pressure readings have been 130-150/80-90 mm Hg. Over the last 4 months he has developed pain in his thighs when walking to his mailbox a block away. The pain resolves after he sits for a few minutes. On examination he has a blood pressure of 135/85 mm Hg, a heart rate of 72 beats/min, a BMI of 26 kg/m2 , and an oxygen saturation of 95% on room air. Examinations of the heart and lungs are normal. There is dependent rubor of both legs but posterior tibial pulses are palpable. No ulcerations are noted. You obtain ankle-brachial indices of 0.85 on the left and 0.80 on the right. You prescribe a daily walking program. Which one of the following additional measures would be most appropriate for this patient? A) Add atorvastatin (Lipitor) B) Add clopidogrel (Plavix) C) Add lisinopril (Prinivil, Zestril) to achieve a goal blood pressure <120/80 mm Hg D) Discontinue aspirin and start warfarin (Coumadin) E) Refer to a vascular surgeon

ANSWER: A Management of asymptomatic peripheral artery disease (PAD) should initially be conservative and should include a walking program (SOR A), smoking cessation, and a healthy diet. Statins should be started for all patients with PAD regardless of their LDL-cholesterol levels (SOR A). High-intensity statins should be used if tolerated. A single antiplatelet agent is recommended for patients with PAD. Both aspirin and clopidogrel are effective in the reduction of stroke, but the combination of the two is recommended only after revascularization surgery. Blood pressure control is indicated in patients with PAD but no antihypertensive class is clearly superior to another, although there is some evidence that ACE inhibitors may have additional benefits in terms of walking and pain. In an 80-year-old patient, lowering blood pressure below 120/80 mm Hg can be associated with significant side effects, including a greater risk of falls. Anticoagulants have not been shown to reduce the risk of major cardiovascular events in patients with PAD and they increase the risk of life-threatening bleeding. Referral to a vascular surgeon or for angiography is indicated if conservative therapy fails or symptoms worsen acutely, pain occurs at rest, or the patient develops ulcerations or loss of tissue.

An unimmunized child is brought to the local health department and diagnosed with measles. All individuals at the day care facility that the child attends were exposed to this child about 48 hours prior to the diagnosis. Which one of the following should be given the MMR vaccine as postexposure prophylaxis? A) An otherwise healthy 9-month-old child who is up to date on all age-appropriate immunizations B) A 5-year-old child with asthma who received a second dose of MMR 2 weeks ago C) A pregnant 24-year-old day care staff member who received two doses of MMR as a child D) A pregnant 36-year-old day care staff member who had one dose of MMR vaccine last year when she started working at the day care facility E) A 52-year-old nurse who works part-time at the day care facility and received two doses of MMR as an adult

ANSWER: A Measles outbreaks are becoming more common and the CDC has outlined who should receive postexposure prophylaxis with the MMR vaccine. To be effective as postexposure prophylaxis the vaccine must be administered within 72 hours of exposure. Infants <12 months of age are considered to be at high risk for complications from measles and should receive postexposure MMR vaccine, although intramuscular immunoglobulin is also an option. Children who are otherwise fully vaccinated do not need additional prophylaxis. Pregnant women cannot receive the MMR vaccine due to fetal risk, but they should receive intravenous immunoglobulin if they do not have evidence of immunity. Health care workers only need to be given the MMR vaccine as prophylaxis if they did not receive two doses previously.

A 67-year-old male diagnosed with polymyalgia rheumatica is started on long-term prednisone therapy. Which one of the following is the recommended first-line agent to prevent steroid-induced osteoporosis? A) Alendronate (Fosamax) B) High-dose vitamin D C) Raloxifene (Evista) D) Teriparatide (Forteo)

ANSWER: A Patients are at risk of developing glucocorticoid-induced osteoporosis if they are on long-term glucocorticoid therapy, defined as >2.5 mg of prednisone for a duration of 3 months or longer. The American College of Rheumatology recommends pharmacologic treatment for these patients, as well as for patients receiving glucocorticoids who have a bone mineral density T-score -2.5 at either the spine or the femoral neck and are either male and 50 years of age or female and postmenopausal. Therapy is also recommended in patients 40 years of age who do not meet these criteria but have a 10-year risk of major osteoporotic fracture of at least 20% or a risk of hip fracture of at least 3% according to the FRAX tool. Oral bisphosphonates are recommended as first-line agents for preventing glucocorticoid-induced osteoporotic fractures, although intravenous bisphosphonates can be used if patients are unable to use the oral forms. Supplementation of calcium (800-1000 mg) and vitamin D (400-800 IU) is also recommended. Raloxifene and teriparatide are options when bisphosphonate therapy fails or is contraindicated (SOR A).

A 52-year-old male sees you for follow-up after his third episode of acute gout in the last year. The patient has otherwise been in good health with recent normal laboratory results, including normal renal function. You decide to begin therapy with allopurinol (Zyloprim). To reduce the risk of recurrent gout episodes, you also initiate therapy with which one of the following for the next 6 months? A) Colchicine (Colcrys) B) Methotrexate C) Prednisone D) Probenecid

ANSWER: A Strong evidence suggests that prophylaxis with either colchicine or NSAIDs reduces the risk for acute gout attacks in patients starting urate-lowering therapy. The optimal duration of such prophylactic therapy is unknown, but moderate evidence suggests that it should be longer than 8 weeks. Although prednisone would be helpful it is not the preferred agent in this patient with normal renal function. Prophylaxis with aspirin, methotrexate, or probenecid is also not appropriate.

A 27-year-old white female with a history of mania sees you because of polyuria and increased thirst over the past month. She has taken lithium, 1800 mg daily, for 3 years and her mania is well controlled. She has not lost weight and there is no family history of her current problem. There are no orthostatic blood pressure changes. Laboratory Findings Serum sodium 145 mEq/L (N 135-145) Serum potassium 4.5 mEq/L (N 3.5-5.0) Serum glucose 92 mg/dL Serum creatinine 0.9 mg/dL (N 0.6-1.5) Serum lithium 1.38 mEq/L (therapeutic range 0.5-1.5) Urine volume 6.85 L/24 hr Urine osmolality 161 mOsm/kg H2O There is no significant change in urine osmolality in response to the administration of vasopressin. Which one of the following is the most likely cause of this patient's problem? A) Drug-induced nephrogenic diabetes insipidus B) Borderline diabetes mellitus C) Panhypopituitarism D) Psychogenic water drinking

ANSWER: A Polyuria occurs in 20%-70% of patients on long-term lithium therapy, even when plasma lithium levels are in the therapeutic range. This is a result of impaired renal concentrating ability that is resistant to vasopressin (nephrogenic diabetes insipidus). Inappropriate antidiuretic hormone secretion causes hyponatremia and fluid retention. The diuresis associated with diabetes mellitus is a result of the osmotic effect of increased serum glucose, which is not present in this case. Patients with hypothalamic or pituitary injuries may develop central diabetes insipidus, which responds to exogenous vasopressin. Psychogenic water drinking occurs in psychiatric patients, but would not be expected to cause impairment of renal concentration or hypernatremia.

A 32-year-old female presents with heat intolerance, excessive weight loss, and anxiety. She gave birth 6 months ago and recently stopped breastfeeding. On examination her thyroid gland is slightly diffusely enlarged and nontender. Laboratory studies reveal a decreased TSH level and elevated free T3 and T4 levels. You suspect that she has postpartum thyroiditis. Which one of the following tests would be most useful to confirm the diagnosis? A) Radioactive iodine uptake B) Thyroid peroxidase antibody levels C) Thyroid ultrasonography D) Thyrotropin receptor antibody levels

ANSWER: A Postpartum thyroiditis is defined as a transient or persistent thyroid dysfunction that occurs within 1 year of childbirth, miscarriage, or medical abortion. Release of preformed thyroid hormone in the bloodstream initially results in hyperthyroidism. During the hyperthyroid phase, radioactive iodine uptake will be low, which can help to confirm the diagnosis. Pregnancy and breastfeeding are contraindications to radionuclide imaging. Thyroid peroxidase antibody levels are elevated with chronic autoimmune thyroiditis (Hashimoto's thyroiditis), and patients present with symptoms of hypothyroidism. The Endocrine Society and American Association of Clinical Endocrinologists do not recommend routine thyroid ultrasonography in patients with abnormal thyroid function tests if there is no palpable abnormality of the thyroid gland. Thyrotropin receptor antibody levels are positive in Graves disease.

A 7-year-old female is brought to your office by her mother, who says that the child has developed underarm odor and is beginning to develop acne. The patient has an unremarkable history, was born at full term, and has no chronic medical problems. There is no family history of endocrine disorders or precocious puberty. An examination reveals normal vital signs and a normal BMI, and her height is stable on the growth curve with no increased velocity. Her sexual maturity rating is stage 1. She has some open and closed comedones on her forehead and back. Which one of the following would be most appropriate at this point? A) Reassurance and surveillance over the next 3-6 months B) TSH, LH, and FSH levels C) Bone age radiography D) Bone age radiography and TSH, LH, and FSH levels

ANSWER: A Premature adrenarche without development of secondary sex characteristics is usually idiopathic and does not lead to an abnormal pattern of development. Reassurance and surveillance over the next 3-6 months would be most appropriate at this time. Laboratory studies and radiography warrant consideration if the patient develops secondary sex characteristics before the age of 8, or if her height velocity increases rapidly during the surveillance period.

A 34-year-old female presents to your office after she was bitten on the hand by a neighbor's cat. The patient has no allergies and has been in good health. You decide to treat the patient with a prophylactic antibiotic. Which one of the following would be the antibiotic of choice? A) Amoxicillin/clavulanate (Augmentin) B) Azithromycin (Zithromax) C) Cephalexin (Keflex) D) Clindamycin (Cleocin) E) Metronidazole (Flagyl)

ANSWER: A Prophylactic antibiotics should be given for all closed-fist injuries unless the skin has not been penetrated, and for puncture wounds caused by cat bites. The antibiotic should have both aerobic and anaerobic activity and include Pasteurella coverage for animal bites and Eikenella coverage for human bites. Suggested regimens include amoxicillin/clavulanate. If the patient is allergic to penicillin, clindamycin plus levofloxacin or moxifloxacin, which has anaerobic coverage, can be used. Azithromycin, cephalexin, and metronidazole are not first-line antibiotics following a cat bite.

A 38-year-old female presents for evaluation of infertility after being unable to conceive for the past 14 months. She has a history of type 2 diabetes, obesity, and hypothyroidism, and takes metformin (Glucophage) and levothyroxine (Synthroid) daily. A review of systems is notable for menses that occur once every 35-50 days and persistent dark hair growth on her chin and areolae. An examination is remarkable only for a blood pressure of 142/95 mm Hg, a BMI of 37 kg/m2 , and the hair growth described by the patient. Which one of the findings in this patient is a required diagnostic criterion for polycystic ovary syndrome? A) Hyperandrogenism B) Hypertension C) Hypothyroidism D) Infertility E) Obesity

ANSWER: A Several professional organizations have published criteria for the diagnosis of polycystic ovary syndrome (PCOS) using various combinations of hyperandrogenism (clinical or biochemical), ovulatory dysfunction (typically oligomenorrhea), and the presence of at least one polycystic ovary by imaging criteria. Hypertension, hypothyroidism, infertility, and obesity are common symptoms in patients with PCOS but are not diagnostic. The National Institutes of Health advises that patients must have both hyperandrogenism and oligomenorrhea to meet the criteria for PCOS. The Endocrine Society recommends diagnosing PCOS based on the 2003 Rotterdam criteria, which require the presence of two of the following: hyperandrogenism, ovulatory dysfunction, and at least one polycystic ovary. The Androgen Excess and PCOS Society says that patients must have hyperandrogenism plus either oligomenorrhea or at least one polycystic ovary for the diagnosis to be made.

A 32-year-old female presents with a 4-month history of nasal drainage, congestion, and loss of her sense of smell. She reports having a cold about 4 months ago that never resolved. On examination the nasal turbinates are swollen and you note mucopurulent drainage on the right. Which one of the following is the most likely cause of her symptoms? A) Chronic rhinosinusitis B) Granulomatosis with polyangiitis (Wegener's granulomatosis) C) Nasal polyposis D) Sarcoidosis E) Seasonal allergic rhinitis

ANSWER: A The American Academy of Otolaryngology defines chronic rhinosinusitis as the presence of two of four cardinal symptoms, which include nasal drainage, nasal obstruction, facial pain or pressure, and hyposmia or anosmia, along with objective signs on examination or radiographic studies. This patient has three cardinal symptoms of chronic rhinosinusitis and objective evidence on the physical examination. No nasal polyps were seen on the examination. Granulomatosis with polyangiitis and sarcoidosis can both present similarly but are uncommon causes of chronic rhinosinusitis. Allergic rhinitis can be associated with chronic rhinosinusitis but would also present with allergic symptoms

A 30-year-old gravida 2 para 1 in her second trimester is evaluated for hypothyroidism. The normal TSH range in pregnancy is A) lower than in the nonpregnant state B) higher than in the nonpregnant state C) the same as in the nonpregnant state D) not useful for evaluating hypothyroidism after the first trimester

ANSWER: A The TSH reference range is lower during pregnancy because of the cross-reactivity of the -subunit of hCG. Levels of hCG peak during weeks 7-13 of pregnancy, and hCG has mild TSH-like activity, leading to slightly high free T4 levels in early pregnancy. This leads to a feedback decrease in TSH.

A previously healthy 45-year-old female presents with upper abdominal pain and dysphagia. An upper GI series reveals no significant reflux. On esophagogastroduodenoscopy the esophagus has a ringed appearance and a biopsy reveals >15 eosinophils/hpf. Helicobacter pylori testing is negative. She does not currently take any medications. Which one of the following would be the best initial treatment? A) Budesonide oral suspension, 1 mg twice daily B) Fexofenadine, 180 mg daily C) Pantoprazole (Protonix), 40 mg once daily D) Prednisone, 40 mg daily for 7 days E) Ranitidine (Zantac), 150 mg once daily

ANSWER: A The clinical presentation and esophagogastroduodenoscopy findings indicate eosinophilic esophagitis (EoE) in this patient. In the absence of other causes of eosinophilia, the presence of >15 eosinophils/hpf is considered diagnostic. Application of corticosteroids to the esophagus is generally the treatment of choice, either in the form of an oral suspension of budesonide or an inhaled corticosteroid sprayed into the mouth and swallowed. Although EoE can occur in patients with other atopic illnesses, this patient does not have any symptoms of allergies or asthma, so an antihistamine such as fexofenadine is not needed. EoE does not respond to proton pump inhibitor therapy or H2-blocker therapy and systemic corticosteroids are not necessary.

A 35-year-old female is admitted to the hospital with fever, tachycardia, and hypotension. Her medical history is significant for current injection drug use. Which one of the following is the most common infection in patients such as this? A) Cellulitis B) Endocarditis C) Osteomyelitis D) Pneumonia E) Pyelonephritis

ANSWER: A The most common complications for patients who inject drugs are skin and soft-tissue infections. These are the most frequent infections leading to hospitalization. This patient is also at risk for endocarditis, osteomyelitis, pneumonia, and pyelonephritis, but these infections are less common.

A 21-year-old male presents to your office with excessive sweating in the axillae, palms, and soles. This has been a problem for several years and interferes with his ability to participate in daily activities without discomfort and embarrassment. After ruling out possible secondary causes you diagnose primary focal hyperhidrosis. Which one of the following would you recommend as a first-line treatment? A) Topical 20% aluminum chloride (Drysol) B) Topical 2% glycopyrrolate C) Oral oxybutynin D) Localized application of microwave energy E) Sympathetic denervation

ANSWER: A The recommended first-line treatment for primary focal hyperhidrosis is topical 20% aluminum chloride. It should be applied to affected areas nightly for 6-8 hours and works by obstructing the eccrine sweat glands and destroying secretory cells. Iontophoresis and botulinum toxin are alternative first- or second-line therapies for palmar and plantar hyperhidrosis and hyperhidrosis affecting the axillae, palms, soles, or face. Topical 2% glycopyrrolate must be compounded by a pharmacy and is indicated only for craniofacial hyperhidrosis. Oral anticholinergics such as oxybutynin can be considered if other first-line treatments fail. However, up to 10% of patients will stop taking these medications due to side effects such as dry mouth, constipation, urinary retention, and blurred vision. Microwave technology is a newer treatment option that has shown some promising results but should not be recommended as a first-line treatment. Sympathetic denervation should be used only if other less invasive therapies have already been tried.

A 63-year-old male with a history of controlled hypertension and benign prostatic hyperplasia presents to your office after an emergency department visit for a closed, displaced left clavicle fracture. The injury occurred when he slipped on ice while shoveling snow. His orthopedist has recommended surgery and has requested a preoperative evaluation. The patient's blood pressure is 128/82 mm Hg and cardiovascular and pulmonary examinations are normal. You determine that his risk for a major adverse cardiac event is <1% for this nonurgent and low-risk surgical intervention. He does not have any chest pain or shortness of breath. He has pain with movement of his left arm and is wearing a sling. Which one of the following would be most appropriate at this point? A) No further testing B) An EKG C) Exercise stress testing D) A chest radiograph E) Echocardiography

ANSWER: A This asymptomatic patient is at low risk for a major adverse cardiac event (<1%) and requires no further testing prior to undergoing a low-risk, nonurgent surgical intervention. A routine preoperative EKG does not provide any benefit for asymptomatic patients undergoing low-risk surgical procedures (level of evidence 2). A preoperative EKG may be considered for asymptomatic patients without known coronary artery disease who are undergoing intermediate and high-risk surgeries (SOR B). A preoperative EKG is reasonable for patients who have known heart disease, peripheral artery disease, or cerebrovascular disease who are undergoing intermediate and high-risk surgeries (SOR B). Exercise stress testing for noncardiac surgery is not useful for patients at low risk but may be helpful for patients with an elevated risk and unknown functional capacity (SOR B). Routine chest radiography has no role in the preoperative evaluation. Routine preoperative echocardiography to assess left ventricular function is not recommended in asymptomatic patients (SOR B). It is reasonable for patients with dyspnea of unknown etiology and patients with worsening heart failure symptoms to undergo preoperative evaluation of left ventricular function (SOR C). Preoperative echocardiography is recommended for patients with known or suspected moderate- or high-degree valvular heart disease (SOR C).

A 58-year-old male with well controlled type 2 diabetes presents for follow-up of ongoing exertional chest pain that has been present for months with no significant change. He has a remote history of myocardial infarction and recently underwent cardiac catheterization, which showed non-obstructing coronary artery disease with a left ventricular ejection fraction of 60%. His current medications include aspirin, 81 mg; atorvastatin (Lipitor), 80 mg; lisinopril (Prinivil, Zestril), 40 mg; metoprolol succinate (Toprol-XL), 100 mg; and metformin (Glucophage), 1000 mg twice daily. Home blood pressure monitoring shows an average blood pressure of 142/92 mm Hg and a pulse rate of 58 beats/min. A physical examination, including a cardiovascular examination, is unremarkable. Which one of the following medications would be the best addition to his current regimen? A) Amlodipine (Norvasc), 2.5 mg daily B) Diltiazem extended release (Cardizem LA), 120 mg daily C) Isosorbide mononitrate extended release, 30 mg daily D) Nifedipine (Procardia), 30 mg 3 times daily E) Ranolazine (Ranexa), 500 mg twice daily

ANSWER: A This patient has a blood pressure that is elevated according to all major current guidelines. Amlodipine, a long-acting dihydropyridine calcium channel blocker (CCB), is the best pharmacologic choice because it will lower blood pressure and treat angina without the risk of heart block. Short-acting CCBs such as non-extended-release nifedipine may cause reflex tachycardia and are not recommended. Nondihydropyridine CCBs such as diltiazem would put this patient at risk for heart block because he is already taking long-acting metoprolol and his heart rate is in the 50s. Long-acting nitrates and ranolazine are options to treat stable angina but would not be effective blood pressure medications.

A 67-year-old male presents to your office for evaluation of chronic redness, flaking, and discomfort of his eyelids. Additionally, his eyes feel irritated, dry, and sandpapery at times. He has had difficulties with these symptoms on and off throughout his life but they have worsened lately. He has not had any vision changes and does not wear contact lenses. On examination his eyelids appear red and mildly swollen with yellow crusting at the bases of the eyelashes. You note bilateral mild conjunctival injection. Visual acuity is intact, as are pupil reactions and extraocular movements. Which one of the following treatments is appropriate first-line therapy for this condition? A) Warm compresses and gentle cleansing with a mild shampoo B) Sodium sulfacetamide eye drops C) Topical betamethasone D) Oral acyclovir (Zovirax) E) Oral cephalexin (Keflex)

ANSWER: A This patient has blepharitis, a chronic inflammation of the eyelids. Seborrhea is a common cause in older adults. In younger patients including children, colonization with Staphylococcus may be a contributing factor. Meibomian gland dysfunction is often part of this condition, contributing to a reduced quality of tear films, which leads to dry eyes and irritation. Other diagnoses to consider in this patient include conjunctivitis, preseptal cellulitis, and Sjögren's syndrome. Conjunctivitis typically involves the conjunctiva and an eye discharge but less involvement of the eyelids is present. Cellulitis is an acute rather than chronic condition and involves more pain and swelling. Sjögren's syndrome causes dry eye but not inflammatory changes of the lid. The initial treatment of blepharitis consists of lid hygiene using warm compresses to remove dried secretions and debris. Mild shampoo can help in this process and aid in keeping the bacterial colonization load down. In severe or recalcitrant cases a topical antibiotic ointment may be applied to the lids. Oral antibiotics can be considered for more severe cases. Ref: Turnbull AM, Mayfield MP: Blepharitis. BMJ 2012;344:e3328.

A 67-year-old male presents with a 3-month history of a worsening cough, shortness of breath, and paroxysmal nocturnal dyspnea. On examination he has a heart rate of 78 beats/min, a blood pressure of 138/88 mm Hg, a respiratory rate of 18/min, and an oxygen saturation of 94% on room air. A physical examination reveals jugular venous distention, bibasilar crackles, 2+ pitting ankle edema, and a displaced apical impulse. An echocardiogram shows normal valvular structure and function with a left ventricular ejection fraction of 55% and diastolic dysfunction. Which one of the following would be most appropriate for this patient? A) Furosemide (Lasix), 20 mg daily B) Isosorbide mononitrate extended release, 60 mg daily C) Lisinopril (Prinivil, Zestril), 20 mg daily D) Losartan (Cozaar), 50 mg daily E) Sacubitril/valsartan (Entresto), 24/26 mg twice daily

ANSWER: A This patient has diastolic heart failure, also called heart failure with preserved ejection fraction (HFpEF). Patients who have HFpEF with active fluid overload should be treated with diuretics such as furosemide (SOR B). If concomitant hypertension is present along with HFpEF, the patient's blood pressure should be treated according to accepted evidence-based hypertension guidelines (SOR C). Although this patient's blood pressure is elevated, a diagnosis of hypertension has not been confirmed, so antihypertensives are not warranted at this time.

During a newborn examination you note a foot deformity, with the front half of the foot turned inward. Applying gentle pressure to the forefoot while holding the heel steady brings the heel and forefoot into alignment. Which one of the following would you recommend? A) Observation only B) Adjustable shoes C) Serial casting D) Surgical correction

ANSWER: A This patient has flexible metatarsus adductus, the most common congenital foot deformity. Flexible metatarsus adductus usually resolves spontaneously by 1 year of age and does not require treatment. Rigid metatarsus adductus should be treated with serial casting. Using adjustable shoes is an alternative that is less expensive than serial casting for motivated parents with children who are not yet walking. Surgical correction should be reserved for older children who are already walking or for those with persistent symptomatic metatarsus adductus that is resistant to casting.

A 2-year-old male is brought to your office by his mother. The child has a 2-day history of a runny nose and mild cough associated with a subjective fever. The cough worsened last night. The patient has had a reduced appetite but a good intake of fluids. On examination the child has an axillary temperature of 37.4°C (99.3°F), a heart rate of 120 beats/min, a respiratory rate of 26/min, a weight of 16 kg (35 lb), and an oxygen saturation of 96% on room air. He appears mildly ill but is alert and does not show any signs of distress, and has a prominent high-pitched barking cough. You note that he has clear rhinorrhea, the tympanic membranes are normal, and the oropharynx is moist and clear. Auscultation reveals inspiratory stridor, but there are no signs of respiratory distress. The patient's skin has good turgor with no rash. Which one of the following would be the most appropriate next step in the management of this child? A) Administration of dexamethasone, 0.6 mg/kg orally in a single dose B) Initiation of oral amoxicillin, 40 mg/kg twice daily C) Administration of albuterol, 2.5 mg/3 mL via nebulizer D) Administration of racemic epinephrine 2.25% solution (Asthmanefrin), 0.5 mL via nebulizer E) Transfer to a hospital emergency department for stabilization and hospital admission

ANSWER: A This patient has mild croup based on the clinical findings. A single dose of dexamethasone is recommended in all cases of croup (SOR A). Hospitalization is not necessary if the child is stable. Racemic epinephrine, which has been shown to reduce symptoms at 30 minutes but not at 2 hours or 6 hours, is recommended for the treatment of moderate to severe croup when patients are being observed in a medical setting such as the emergency department or hospital (SOR A). Amoxicillin and albuterol are not indicated in the management of croup.

A 59-year-old plumber presents with swelling of his left elbow. An examination reveals swelling over the olecranon but no erythema or warmth. His uric acid levels are normal and he has no history of inflammatory disease. He has not had a fever. The swollen area is not painful and he has a normal range of motion. Which one of the following would be the most appropriate next step? A) Padding, ice, and elevation B) Empiric antibiotics C) Bursal aspiration D) Plain radiographs E) Orthopedic referral

ANSWER: A This patient has olecranon bursitis, which is a superficial bursitis caused by chronic microtraumas to the affected area. The initial management for this condition includes conservative measures such as padding, elevation, icing, and analgesics (SOR B). If significant pain is associated with the swelling, or a decrease in range of motion is present due to severe swelling, aspiration should be offered. This is not indicated in this particular case and should be avoided to reduce the risk of septic bursitis. Septic bursitis would require empiric antibiotics to cover common skin organisms (SOR B). Aspiration should be performed if infection is suspected and the aspirate should be sent for a cell count, Gram stain, culture, glucose measurement, and crystal analysis (SOR C). Plain radiographs are indicated only if there is acute trauma and concern that a fracture may be present. If recurrent superficial bursitis occurs, a referral for surgery is indicated.

A 15-year-old male presents for a sports preparticipation examination. His family history includes hypertension in his mother. There are no other concerns. He does not take any medications or supplements. An examination shows a BMI at the 75th percentile for his age. His weight is 59 kg (130 lb) and his blood pressure is 130/88 mm Hg. On repeat measurements 30 minutes apart, his blood pressure is 132/90 mm Hg and 132/88 mm Hg. The remainder of the examination, including vital signs, is normal. Which one of the following would you recommend at this time? A) Therapeutic lifestyle changes with close follow-up in 2-3 weeks B) Yearly blood pressure monitoring C) Atenolol (Tenormin), 25 mg daily D) Enalapril (Vasotec), 5 mg daily E) Hydrochlorothiazide, 12.5 mg daily

ANSWER: A This patient has stage 1 hypertension based on his average blood pressure of 131/88 mm Hg after three measurements. He is asymptomatic so the initial recommendation is therapeutic lifestyle changes and close follow-up in 2-3 weeks (SOR C). If his blood pressure remains elevated, an evaluation for secondary hypertension would be appropriate. Medication would be appropriate for stage 1 hypertension with either evidence of end-organ disease or persistent hypertension after a trial of therapeutic lifestyle changes or progression to stage 2 hypertension (SOR C). Thiazide diuretics or ACE inhibitors would be appropriate choices for initial treatment. -Blockers are no longer considered first-line treatment for hypertension in adolescents or children. This patient would need blood pressure monitoring at every visit due to his elevated blood pressure.

A 42-year-old female presents for follow-up after being treated for recurrent respiratory problems at an urgent care facility. She is feeling a little better after a short course of oral prednisone and use of an albuterol (Proventil, Ventolin) inhaler. She has had a gradual increase in shortness of breath, a chronic cough, and a decrease in her usual activity level over the past year. She has brought a copy of a recent chest radiograph report for your review that describes panlobular basal emphysema. She does not have a history of smoking, secondhand smoke exposure, or occupational exposures. Spirometry in the office reveals an FEV1/FVC ratio of 0.67 with no change after bronchodilator administration. Which one of the following underlying conditions is the most likely cause for this clinical presentation? A) 1-Antitrypsin deficiency B) Bronchiectasis C) Diffuse panbronchiolitis D) Interstitial lung disease E) Left heart failure

ANSWER: A This patient presents with symptoms of chronic obstructive lung disease, and spirometry confirms airflow limitation or obstruction with an FEV1/FVC <0.7. Her age, the lack of tobacco smoke or occupational exposures, and the chest radiograph findings are typical of 1-antitrypsin deficiency. While left heart failure, interstitial lung disease, bronchiectasis, and diffuse panbronchiolitis are all causes of chronic cough, they are not necessarily associated with the development of COPD and these spirometry findings. Furthermore, the radiologic findings in this patient are not consistent with these conditions. Left heart failure would present with pulmonary edema on a chest radiograph and volume restriction on pulmonary function testing. Bronchiectasis would present with bronchial dilation and bronchial wall thickening on a chest radiograph. Interstitial lung disease would present with reticular or increased interstitial markings. Diffuse panbronchiolitis would present with diffuse small centrilobular nodular opacities along with hyperinflation.

A 34-year-old male with sickle cell disease has a new onset of mild to moderate thirst and polyuria. He ate a large meal about 2 hours ago. An examination reveals a BMI of 32 kg/m2 . Results of a urinalysis performed by your staff include 3+ glucose and no ketones. His blood glucose level is 288 mg/dL and his hemoglobin A1c is 5.2%. Which one of the following would be most appropriate at this point to help diagnose and monitor this patient's glycemic control? A) A serum fructosamine level B) A repeat hemoglobin A1c C) A 2-hour glucose tolerance test D) Hemoglobin electrophoresis E) Referral to an endocrinologist

ANSWER: A This patient with sickle cell disease has a new onset of diabetes mellitus. Hemoglobinopathies falsely lower hemoglobin A1c as a result of hemolysis and abnormal glycation. Fructosamine correlates well with hemoglobin A1c levels and is recommended instead of hemoglobin A1c for monitoring glucose control in patients with diabetes and hemoglobinopathies. A 2-hour glucose tolerance test or hemoglobin electrophoresis would not provide useful information. Referral to an endocrinologist is not indicated at this point because the patient has not failed primary care management.

A 6-month-old male is brought to the urgent care center with a 3-day history of rhinorrhea, cough, and increased respiratory effort. His temperature is 37.5°C (99.5°F), his heart rate is 120 beats/min, his respiratory rate is 42/min, and his oxygen saturation is 96% on room air. On examination the child appears well hydrated with clear secretions from his nasal passages, there is diffuse wheezing heard bilaterally, and there is no nasal flaring or retractions. The mother states that the child has a decreased appetite but is drinking a normal amount of fluids. Which one of the following would be the most appropriate management for this patient? A) Supportive therapy only B) Bronchodilators C) A corticosteroid taper D) Epinephrine E) Nebulized hypertonic saline

ANSWER: A This patient's symptoms and the examination suggest viral bronchiolitis. Supportive therapy, including adequate hydration, is recommended for treatment. Treatment with bronchodilators, epinephrine, hypertonic saline, or corticosteroids is not indicated (SOR A).

A 44-year-old female presents for a pretravel consultation and asks about medication options for traveler's diarrhea. She will be on an organized tour traveling to a country with a very low risk for this problem. She plans to take all precautions to further reduce her risk but would also like you to recommend a medication she can take. Which one of the following would be an appropriate recommendation? A) A short course of azithromycin (Zithromax) if she develops diarrhea B) Loperamide (Imodium) daily, starting 1 day prior to travel and continued until 1 day after returning home C) Probiotics daily, starting 1 week prior to travel and continued until 1 week after returning home D) Ciprofloxacin (Cipro) daily, starting 2 weeks prior to travel and continued until 4 weeks after returning home E) Bismuth subsalicylate daily, starting 2 weeks prior to travel and continued until 4 weeks after returning home

ANSWER: A Traveler's diarrhea is the most common infection in international travelers. A short course of antibiotics can be taken after a traveler develops diarrhea and usually shortens the duration of symptoms (SOR A). Azithromycin is preferred to treat severe traveler's diarrhea. Rifaximin or fluoroquinolones may be used to treat severe nondysenteric traveler's diarrhea. Prophylactic antibiotics are not routinely recommended. For patients at high risk, bismuth subsalicylate reduces the risk but does not need to be initiated prior to travel. There is insufficient evidence for the use of probiotics to prevent traveler's diarrhea. Loperamide can be used with or without antibiotics after symptoms develop but is not recommended for prophylaxis.

An otherwise healthy 42-year-old male presents to your office with low back pain that started a week ago after he lifted a heavy box. Since the time of his injury he has been having consistent pain, numbness, and tingling that radiates down the back of his right leg to his calf. Which one of the following would you order at this time? A) No imaging B) Plain radiography C) CT D) MRI

ANSWER: A Uncomplicated acute low back pain and/or radiculopathy is a benign, self-limited condition and early imaging is associated with worse overall outcomes and is likely to identify minor abnormalities even in asymptomatic patients. Imaging for acute low back pain should be reserved for cases that are suspicious for cauda equina syndrome, malignancy, fracture, or infection. In the absence of red flags such as progressive motor or sensory loss, new urinary retention or overflow incontinence, a history of cancer, a recent invasive spinal procedure, or significant trauma relative to age, imaging is not warranted regardless of whether radiculopathy is present, unless symptoms persist despite a trial of at least 6 weeks of medical management and physical therapy.

A 77-year-old Spanish-speaking female with end-stage heart failure has elected hospice care to be provided at home for the duration of her life. A trained interpreter is available for assistance when you see the patient and is present in the room. Which one of the following is considered a best practice when using interpreters? A) Addressing the patient directly when speaking B) Seating the interpreter closest to the clinician, slightly in front of the patient, to observe body language when translating C) Asking the interpreter to serve as a witness for a consent form for hospice D) Explaining to the interpreter the entire care plan, then having him or her repeat it back to the patient E) Explaining in full detail all possible scenarios for symptom management and what to expect

ANSWER: A When professional interpreters participate in patient care it is important to speak directly in the first person, using "I" statements rather than statements that start with "tell her" (SOR C). It is ideal to seat the interpreter next to or slightly behind the patient, so that the patient is the focus of the interaction. Sentence-by-sentence interpretation can prevent miscommunication errors, as opposed to expecting the interpreter to remember every detail of a complex care plan. It is not appropriate for the medical interpreter to also serve as a witness to consent. Focusing on three or fewer key points rather than over-communicating multiple complex issues increases the likelihood that the patient will comprehend the plan of care.

A 48-year-old male presents with psychogenic erectile dysfunction in conjunction with depression. You decide that treatment of his underlying mood disorder is the best initial step. Which one of the following antidepressants would be most appropriate? A) Bupropion (Wellbutrin) B) Citalopram (Celexa) C) Nortriptyline (Pamelor) D) Sertraline (Zoloft) E) Venlafaxine (Effexor XR)

ANSWER: A When psychogenic erectile dysfunction (ED) coexists with depression, treatment of the underlying mood disorder is often an appropriate first step (SOR C). An antidepressant that is less likely to worsen the ED, such as bupropion, mirtazapine, or fluvoxamine, should be chosen. Antidepressants that are more likely to cause sexual side effects should be avoided, including SSRIs, SNRIs, and tricyclic and tetracyclic antidepressants. Phosphodiesterase-5 inhibitors are the first line of treatment for ED (SOR A) and can be used effectively in men with depression, in combination with treatments for mood disorders.

The majority of U.S. women who unintentionally become pregnant choose to A) carry the pregnancy to term and keep the baby B) carry the pregnancy to term and place the baby for adoption C) terminate the pregnancy by medical means legally D) terminate the pregnancy by surgical means legally E) terminate the pregnancy illegally

ANSWER: A While recent data is not available, 45% of pregnancies in the United States in 2011 were unintentional. About 42% of pregnant women with unintended pregnancies chose to terminate the pregnancy by one means or another. Of the 58% that chose to carry the pregnancy to term, only about 1% placed the infant for adoption.

A 58-year-old female with COPD asks what she can do to avoid hospitalization. She does not have any other medical problems. Which one of the following interventions has been shown to reduce respiratory-related hospital admissions in patients such as this? A) Written self-management plans that include smoking cessation plans B) Regular physical activity C) Regular assessment of FEV1 D) Nightly CPAP therapy E) Daily oxygen therapy

ANSWER: A Written self-management plans have been shown to decrease respiratory-related hospitalizations in patients with COPD. Although regular physical activity has clear health benefits, the methods are so varied in studies of physical activity that there is currently no strong evidence to show it reduces hospitalizations in COPD patients. Although FEV1 is important for predicting hospitalizations for a population, it is not accurate enough to be useful in an individual patient. Daily oxygen therapy does not help to postpone the first hospitalization. Nightly CPAP therapy reduces hospitalizations in patients with COPD and sleep apnea, but not those with COPD alone.

A 46-year-old female presents to your office with a 24-hour history of moderate pleuritic chest pain. She does not take any medications and has been in excellent health. A physical examination is normal, including pulse oximetry, and a chest radiograph is also normal. You diagnose viral pleurisy. Which one of the following would be the most appropriate treatment? A) Colchicine (Colcrys) B) Ibuprofen C) Prednisone D) Tramadol (Ultram)

ANSWER: B NSAIDs such as ibuprofen should be used as first-line treatment for the control of pleuritic pain (SOR B). NSAIDs do not have the analgesic potency of narcotics, but they do not cause respiratory suppression and do not change the patient's sensorium. Corticosteroids should be reserved for patients who cannot take NSAIDs. Colchicine is used to treat pericarditis but not pleuritic pain.

A 26-year-old G2P1001 at 30 weeks gestation was recently diagnosed with gestational diabetes and is ready to start testing her blood glucose at home. Which one of the following is the recommended goal for fasting blood glucose in this patient? A) <75 mg/dL B) <95 mg/dL C) <120 mg/dL D) <150 mg/dL E) <180 mg/dL

ANSWER: B The goal fasting blood glucose level in patients with gestational diabetes is <95 mg/dL. A fasting glucose level <80 mg/dL is associated with increased maternal and fetal complications. The goal 2-hour postprandial glucose level is <120 mg/dL and the goal 1-hour postprandial glucose level is <140 mg/dL.

Which one of the following is the most common cause of hirsutism in premenopausal women? A) Idiopathic hirsutism B) Polycystic ovary syndrome C) Androgen-secreting tumors D) Nonclassic congenital adrenal hyperplasia E) Medication side effects

ANSWER: B The most common cause of hirsutism in premenopausal women is polycystic ovary syndrome, which accounts for 75%-80% of cases. The second most common cause is idiopathic hirsutism, which accounts for 5%-20% of cases. Other causes are rare, but should be considered when evaluating new cases of hirsutism.

A 22-year-old gravida 1 para 1 presents with her 4-day-old male infant for follow-up after an uncomplicated vaginal delivery at term. She is exclusively breastfeeding and the child's weight is currently 7% below his birth weight and 2% below his weight at hospital discharge 2 days ago. You observe her breastfeeding technique in the office to look for feeding problems. Which one of the following would raise concerns that the mother or infant may be having difficulty with breastfeeding? A) The infant's chin is pressed against the breast B) The infant's cheeks are dimpled during feeding C) The mother's nipples are everted D) The mother's areola is not visible during breastfeeding E) The infant has yellow stools

ANSWER: B An infant's dimpled cheeks during breastfeeding is a sign of a poor latch, with the infant not getting enough of the breast in the mouth. With a good latch, most or all of the mother's areola is completely in the infant's mouth and thus not visible during breastfeeding. The infant's chin, not the nose, should be against the breast. Inverted (rather than everted) or very large nipples may be problematic and require additional lactation support. The transition of the infant stool from dark meconium stools to yellow stools is a sign that the infant is digesting milk.

A 35-year-old gravida 2 para 2 reports diminished sexual arousal since initiating antidepressant therapy with sertraline (Zoloft). She has normal menstrual cycles and does not have any other symptoms. Which one of the following would you recommend? A) Black cohosh, 40 mg daily B) Bupropion (Wellbutrin), 150 mg twice daily C) Ethinyl estradiol, 0.5 mg daily D) Ospemifene (Osphena), 60 mg daily E) A testosterone patch, 300 g applied twice weekly

ANSWER: B Bupropion can improve antidepressant-related sexual arousal dysfunction (SOR B). Black cohosh is considered a safe alternative for treating menopausal vasomotor symptoms, but not for treating sexual arousal dysfunction in women who are premenopausal. Ethinyl estradiol may be taken to improve sexual dysfunction related to menopausal symptoms. Vaginal estrogen therapy is recommended over oral estrogen when vaginal dryness is the primary symptom. Ospemifene is indicated for dyspareunia related to vulvar and vaginal atrophy due to menopause. Testosterone has proven to be effective for treating menopause9 related low sexual desire but the evidence is limited due to the lack of long-term data. The Endocrine Society recommends consideration of a 3- to 6-month course of testosterone specifically for postmenopausal women with low sexual desire.

A 13-year-old male is admitted to the hospital with diabetic ketoacidosis. Aggressive fluid resuscitation with normal saline was initiated in the emergency department and the following laboratory results were obtained: Glucose 400 mg/dL Sodium 136 mEq/L (N 136-145) Potassium 2.8 mEq/L (N 3.5-5.1) Bicarbonate 15 mEq/L (N 22-29) Anion gap 4 mEq/L (N 10-20) In addition to continued fluid resuscitation, which one of the following would be the most appropriate next step in the management of this patient? A) Administration of sodium bicarbonate B) Potassium replacement C) An intravenous insulin drip D) Subcutaneous insulin using a basal/bolus technique E) Bedside ketone capillary measurement

ANSWER: B Correcting the potassium level is the best treatment choice for this patient. A low serum potassium level in diabetic ketoacidosis (DKA) indicates a significant potassium deficiency, placing the patient at risk for a cardiac arrhythmia, among other complications. Potassium deficiency is usually the product of urinary losses due to glucose osmotic diuresis and secondary hyperaldosteronism. However, serum potassium can remain normal when there is a whole body deficiency, as a result of movement of potassium out of cells in response to the acidosis, insulin deficiency, and hyperosmolality. This patient's serum potassium is low, which indicates severe deficiency.Sodium bicarbonate can be used in some cases of DKA but is not the most important next step in this case. If insulin therapy is initiated prior to potassium replacement the insulin can force more potassium into cells, worsening the serum deficiency. If the serum potassium level is <3.3 mEq/L potassium should be replaced prior to insulin therapy. Ketone measurement can also be part of DKA management but is not the most appropriate next step for this patient

A 79-year-old female comes to your office with her daughter to discuss concerns about the mother's endurance. The daughter thinks her mother is losing strength more quickly than she should. The patient's past medical history includes osteoporosis and hypertension. She is using a cane when needed for support but has not fallen. She is not sleeping well and her insomnia seems to have worsened over the last year. They both agree she is not depressed, and her PHQ-2 score today is 0. After a discussion with the patient and her daughter and a physical examination, you are concerned that the patient is at risk for frailty-related health outcomes. Which one of the following interventions would you recommend as most important to improve this patient's health outcomes and physical function? A) Aerobic exercise B) Resistance training C) Nutritional counseling D) Vitamin D E) Mirtazapine (Remeron)

ANSWER: B Frailty is an increasingly common problem in the geriatric population, especially as individuals are living longer. The current literature supports resistance training to improve physical function before aerobic exercise is introduced. Although there was hope that nutritional counseling would improve outcomes, it does not seem to have an effect. Vitamin D supplementation also does not improve outcomes in these individuals. Mirtazapine is on the Beers list, and without a compelling indication such as weight loss with depression it should be avoided in the elderly.

A 14-year-old male sees you for a well child examination. He had one dose of HPV vaccine at his last well child examination 1 year ago. Which one of the following is true regarding HPV vaccine for this patient? A) He does not require additional HPV vaccine B) He should receive one dose of the vaccine now and no additional HPV vaccine in the future C) He should receive the vaccine now and again in 4 months D) He should receive the vaccine now and again in 6 months E) He should receive the vaccine now, in 2 months, and in 4 months

ANSWER: B HPV vaccine is currently recommended for males and females at age 11. Catch-up vaccination is recommended until age 21 in males and 26 in females. Children who receive the first dose of the vaccine before the age of 15 and receive two doses are considered adequately vaccinated. If the first dose is given after age 15, a three-dose series is recommended.

A 15-year-old male presents with a 2-day history of dark-colored urine, lower extremity edema, and fatigue. Approximately 2 weeks ago he said he had a "bad sore throat" that was treated empirically with amoxicillin. On examination his blood pressure is 144/92 mm Hg, his pulse rate is 76 beats/min, and his other vital signs are normal. Other than mild dependent edema there are no additional significant physical examination findings. A urinalysis dipstick shows 3+ hematuria. Which one of the following findings on microscopic evaluation of the urine sediment would help to confirm the diagnosis in this patient? A) Gram-positive cocci in chains B) RBC casts C) WBC casts D) Eosinophils E) Oxalate crystals

ANSWER: B This is a classic presentation for acute poststreptococcal glomerulonephritis (APSGN), with the onset of gross hematuria associated with hypertension and systemic edema. This is most commonly seen in school-age children, usually 1-2 weeks after an episode of pharyngitis or 3-4 weeks after an episode of impetigo, caused by so-called nephritogenic strains of Group A -hemolytic Streptococcus. The hematuria is caused by immune complex-mediated glomerular injury. Bacteriuria may be seen in both upper and lower urinary tract infections, but may also be a spurious finding, especially with the combined presence of epithelial cells. The classic finding on microscopic urinalysis for acute glomerulonephritis is the presence of RBC casts. WBC casts are seen with acute pyelonephritis. The presence of urinary eosinophils indicates acute interstitial nephritis. Calcium oxalate makes up the most common type of kidney stones. Antibiotics prescribed for antecedent pharyngitis do not prevent APSGN. Treatment is supportive, controlling blood pressure and edema with a thiazide or a loop diuretic. The prognosis for resolution and full recovery of the vast majority of patients with APSGN is excellent, especially in the pediatric age group.

A 70-year-old male presents to your office for follow-up after he was hospitalized for acute coronary syndrome. He has not experienced any pain since discharge and is currently in a supervised cardiac rehabilitation exercise program. His medications include aspirin, lisinopril (Prinivil, Zestril), and metoprolol, but he was unable to tolerate atorvastatin (Lipitor), 40 mg daily, because he developed muscle aches. Which one of the following would you recommend? A) Evolocumab (Repatha) B) Ezetimibe/simvastatin (Vytorin) C) Fenofibrate (Tricor) D) Niacin E) Omega-3 fatty acid supplements

ANSWER: B High-intensity statin therapy is recommended for patients younger than 75 years of age with known coronary artery disease. For those who are intolerant of high-intensity statins, a trial of a moderate-intensity statin is appropriate. There is evidence to support ezetimibe plus a statin in patients with acute coronary syndrome or chronic kidney disease. Omega-3 fatty acids, fibrates, and niacin should not be prescribed for primary or secondary prevention of atherosclerotic cardiovascular disease because they do not affect patient-oriented outcomes. PCSK9 inhibitors such as evolocumab are injectable monoclonal antibodies that lower LDL-cholesterol levels significantly and have produced some promising results, but more studies are needed to determine when this would be cost effective.

A 25-year-old female has a heart murmur on her postpartum visit. This was first noted at the age of 20. She has been asymptomatic. The murmur is systolic and increases in intensity with Valsalva maneuvers. Further questioning reveals that her two sisters died suddenly from cardiac problems in their early twenties. This patient should be evaluated for A) dilated cardiomyopathy B) hypertrophic cardiomyopathy C) peripartum cardiomyopathy D) restrictive cardiomyopathy

ANSWER: B Hypertrophic cardiomyopathy is the most common type of cardiomyopathy, with a prevalence of 1:500. It involves left ventricular hypertrophy without chamber dilatation. It is caused by autosomal dominant genetic mutations and is associated with sudden death. Dilated cardiomyopathy is a leading cause of heart failure but most patients are symptomatic. Peripartum cardiomyopathy may occur during and after pregnancy and presents as heart failure. Restrictive cardiomyopathy presents with right-sided heart fa

An 85-year-old female presents to your office to establish care. She has a history of essential hypertension, bilateral osteoarthritis of the knees, depression, and hypothyroidism. The patient feels well and has only mild pain from her osteoarthritis. A recent basic metabolic panel showed a sodium level of 132 mEq/L (N 135-145) but was otherwise unremarkable. A TSH level was 1.07 U/mL (N 0.45-4.50). Which one of the medications in this patient's current regimen is most likely causing the hyponatremia? A) Acetaminophen B) Hydrochlorothiazide C) Levothyroxine (Synthroid) D) Metoprolol succinate (Toprol-XL) E) Vitamin D

ANSWER: B Hyponatremia is an extremely common condition, affecting 4%-7% of the ambulatory population. While a number of medications can cause hyponatremia, among the most common offenders are SSRIs and thiazide diuretics. Acetaminophen and metoprolol are not known to affect sodium levels. Vitamin D deficiency has also been linked to hyponatremia so supplementation would likely improve rather than worsen hyponatremia.

A 2-year-old female is brought to your office for a well child check. She had an episode of coughing and mild bronchospasm 3 months ago that was successfully treated with albuterol (Proventil, Ventolin). The mother asks you if there are any factors that would increase the patient's risk of asthma. Which one of the following factors would increase this patient's risk of asthma? A) Living in a high microbial environment B) Exposure to respiratory syncytial virus C) Recurrent otitis media D) Persistent lactose intolerance

ANSWER: B Immunologic profiles of patients with asthma are influenced by environmental exposures. Those who are exposed to respiratory syncytial virus as an infant have an increased risk, whereas those who are exposed to a high microbial environment have a lower risk than those without such exposure. Otitis media and lactose intolerance are not known to be associated with asthma risk.

A 78-year-old female with Alzheimer's disease is accompanied to an office visit by her daughter. The daughter has asked to complete an advance directive giving her medical power of attorney. Which one of the following would indicate that the patient lacks capacity to make decisions with regard to completing her medical directive? A) A dementing illness B) Inconsistent answers to questions C) Lack of orientation to time D) Asking that her son make medical decisions for her instead of her daughter E) A score of 24/30 on the Mini-Mental State Examination

ANSWER: B In order for patients to show they have the capacity to make a decision they must demonstrate an understanding of the situation, including the risks, benefits, and consequences of the decision or refusal of care. If a patient gives inconsistent answers to questions after multiple explanations, this indicates that there is a lack of understanding and would meet one of the criteria to determine that the patient lacks the capacity to make that decision. The presence of dementia can be associated with an increased incidence of having a lack of capacity; however, a diagnosis of dementia by itself does not indicate that the patient lacks the capacity to make a decision. While disorientation to time or a lower score on the Mini-Mental State Examination is associated with an increased risk of lacking capacity, these findings alone would not be enough to determine that the patient lacks capacity. The patient asking that her son be her medical decision maker instead of her daughter would not be an indication that she lacks capacity.

A 48-year-old female presents with the nail findings shown below. Her past medical history is significant for systemic lupus erythematosus. This finding is known as A) clubbing B) koilonychia C) leukonychia D) onychomycosis E) squamous cell carcinoma

ANSWER: B Koilonychia, also known as spoon nail, is a finding that can be associated with multiple systemic conditions such as iron deficiency, hypothyroidism, and systemic lupus erythematosus. Clubbing of the nails involves thickening of the soft tissue proximal to the nail. Leukonychia is a white discoloration of the nail plate and can be a normal variant. Onychomycosis is a fungal infection generally presenting as a thickened, yellow, dystrophic nail. Squamous cell carcinoma presents as an erythematous, proliferating mass that disrupts normal nail morphology.

The American Academy of Pediatrics recommends measuring a child's hemoglobin level at what age? A) 6 months B) 12 months C) 18 months D) 2 years E) 5 years

ANSWER: B Multiple reports have associated iron deficiency with impaired neurodevelopment and it is therefore essential to ensure adequate iron intake. Based on expert opinion, the American Academy of Pediatrics recommends measuring a child's hemoglobin level at 12 months of age.

A 52-year-old female with metastatic breast cancer is hospitalized for treatment of complications from her cancer treatment. She has developed a new onset of back pain that has been progressively worsening over the past few hours. The pain is worse when she is lying down and is not responsive to pain medication. Which one of the following would be the most appropriate next step to address this patient's back pain? A) Increase the dosage of her immediate-release morphine B) Increase the dosage of her sustained-release morphine C) Order cyclobenzaprine D) Order an urgent MRI E) Order a physical therapy consultation for mobility

ANSWER: B In order for patients to show they have the capacity to make a decision they must demonstrate an understanding of the situation, including the risks, benefits, and consequences of the decision or refusal of care. If a patient gives inconsistent answers to questions after multiple explanations, this indicates that there is a lack of understanding and would meet one of the criteria to determine that the patient lacks the capacity to make that decision. The presence of dementia can be associated with an increased incidence of having a lack of capacity; however, a diagnosis of dementia by itself does not indicate that the patient lacks the capacity to make a decision. While disorientation to time or a lower score on the Mini-Mental State Examination is associated with an increased risk of lacking capacity, these findings alone would not be enough to determine that the patient lacks capacity. The patient asking that her son be her medical decision maker instead of her daughter would not be an indication that she lacks capacity. Ref: Barstow C, Shahan B, Roberts M: Evaluating medical decision-making capacity in practice. Am Fam Physician 2018;98(1):40-46.

Which one of the following would be an indication for emergency contraception after unprotected intercourse? A) Being 2 days late for a depot medroxyprogesterone acetate (Depo-Provera) injection, which was given 1 week prior to intercourse B) Forgetting to change an etonogestrel/ethinyl estradiol vaginal ring (NuvaRing) when the current ring has been in place for 6 weeks C) Missing one dose of a combined oral contraceptive pill in the month prior to a sexual encounter D) Failure to use backup protection such as a condom when using a progestin-only mini-pill with no missed doses E) When a patient reports a sexual encounter that occurred 2 weeks ago

ANSWER: B In patients who use the etonogestrel/ethinyl estradiol vaginal ring, emergency contraception is recommended if they have intercourse when the same ring has been in place for 5 weeks. Backup protection is advised until the new ring has been in place for 1 week. Backup protection is generally unnecessary with the depot medroxyprogesterone injection unless the patient is more than 2 weeks late for her injection. Backup protection is then recommended for 7 days after the injection. Emergency contraception is recommended for women who miss two or more doses of their combined contraceptive in 1 month, especially if these doses were missed during the first week and unprotected intercourse occurs during that week. Backup protection should continue until the oral contraceptive pill has been taken for 7 consecutive days. Emergency contraception is generally only recommended up to 5 days after unprotected intercourse. The progestin-only mini-pill is effective alone for contraception when taken as prescribed.

A 35-year-old male presents with a 2-week history of lower extremity edema. He is in good health and does not take any medications. You note weight gain, and mild dyspnea with exertion. An examination is unremarkable except for 2+ to 3+ pitting edema of the lower extremities to his knees bilaterally. A CBC and metabolic panel are unremarkable except for a low albumin level. A urinalysis reveals 3+ protein on the dipstick with no microscopic findings. Which one of the following would be the most appropriate next step? A) Urine microscopy to check for eosinophils B) A spot urine protein/creatinine ratio C) Renal ultrasonography D) Echocardiography E) Referral for a renal biopsy

ANSWER: B Individuals with nephrotic syndrome often present with edema and fatigue with no evidence of severe liver disease or heart failure. Hallmarks of this problem include heavy proteinuria, hypoalbuminemia, and peripheral edema, often with hyperlipidemia as well. While most of these cases are idiopathic, secondary causes such as diabetes mellitus, systemic lupus erythematosus, and medication reactions should be considered. To confirm proteinuria in the nephrotic range a spot urine protein/creatinine ratio is now suggested instead of a 24-hour collection of urine. Checking urine for eosinophils has been recommended in the past for evaluation for acute interstitial nephritis but subsequent studies have shown a lack of specificity and sensitivity. Renal ultrasonography would be indicated if the glomerular filtration rate were reduced. Echocardiography would be appropriate if heart failure were suspected. While a renal biopsy is often recommended, it is most useful in patients with suspected underlying systemic lupus erythematosus or similar disorders when a biopsy can guide management decisions and prognosis.

You and your office educator are preparing printed educational materials for patients, families, and caregivers. Based on current recommendations, the educator advises that these should be written at or below what reading level? A) First grade B) Fifth grade C) Eighth grade D) Tenth grade E) Twelfth grade

ANSWER: B It is recommended that all printed patient education information be written at or below a fifth- to sixth-grade reading level. This aligns with universal health literacy precautions in which easy-to-understand concepts and terms are used for all patients instead of focusing on those with low literacy. These literacy precautions are recommended by professional organizations such as the American Medical Association and the Agency for Healthcare Research and Quality.

An otherwise healthy 55-year-old male who is visiting from Arizona presents to your office with a 4-week history of intermittent fevers, night sweats, dry cough, weight loss, and myalgia. The patient has no other recent history of travel. Of the following, the most likely cause of his symptoms is A) blastomycosis B) coccidioidomycosis C) cryptococcosis D) histoplasmosis E) mucormycosis

ANSWER: B Knowledge of endemic fungi capable of causing infection in otherwise healthy patients can be very helpful in ensuring an appropriate evaluation. Coccidioidomycosis is a common infection in the southwestern United States. In addition to the symptoms in this patient, coccidioidomycosis can also present with a rash such as erythema nodosum. Histoplasmosis is most common in the Midwest and with low-level exposure symptoms are usually mild or absent. Blastomycosis is also present in the Midwest, as well as in the Atlantic and southeastern states. Symptoms include an abrupt onset of fever, chills, pleuritic chest pain, arthralgias, and myalgias. The cough is initially nonproductive but frequently becomes purulent. Cryptococcosis and mucormycosis are more opportunistic infections occurring in immunocompromised hosts.

A patient with a BMI of 32 kg/m2 has type 2 diabetes that is currently controlled by lifestyle interventions, including moderate-intensity physical activity and healthy low-calorie meals. The patient asks about nonnutritive sweeteners, containing few or no calories. According to the American Diabetes Association, which one of the following would be the most appropriate advice? A) Sucrose (table sugar) is preferred B) Nonnutritive sweeteners are acceptable to use C) Nonnutritive sweeteners worsen glucose control D) Sucralose-based sweeteners, such as Splenda, should be avoided E) Sweeteners with aspartame, such as Equal, should be avoided

ANSWER: B Nonnutritive sweeteners contain few or no calories. According to the American Diabetes Association, nonnutritive sweeteners may be acceptable to use instead of nutritive sweeteners such as sucrose. They should be used in moderation if they are used. The use of nonnutritive sweeteners can help to reduce overall intake of carbohydrates and calories. They do not significantly affect glycemic control. Research is inconsistent regarding the effects of nonnutritive sweeteners on weight loss, but most systematic reviews and meta-analyses demonstrate a benefit. There is no recommendation to avoid sucralose or aspartame in patients with type 2 diabetes. Beverages sweetened with sugar are associated with an increased risk of type 2 diabetes.

In addition to a thorough history and physical examination, the routine evaluation of patients presenting with syncope should include A) a CBC, comprehensive metabolic panel, TSH level, and urinalysis B) orthostatic blood pressure measurements and an EKG C) cardiac stress testing D) echocardiography and Doppler ultrasonography of the carotid arteries E) CT or MRI of the brain

ANSWER: B Orthostatic blood pressure measurement and an EKG are indicated in the routine evaluation of patients with syncope. All other testing should be directed by findings obtained in the history and on the physical examination.

Three weeks after he had knee surgery, a 64-year-old male presents for follow-up of an emergency department visit for a pulmonary embolism. He has no previous history of pulmonary embolism and is otherwise in good health. He is being treated with apixaban (Eliquis). The recommended duration of anticoagulation therapy for this patient is A) 1 month B) 3 months C) 6 months D) 9 months E) 12 months

ANSWER: B Patients who have a venous thromboembolism (VTE) require anticoagulation therapy for treatment and prevention of recurrence. The risk of recurrence is greatest in the first year after the event and remains elevated indefinitely. The risk for VTE recurrence is dependent on patient factors, such as active cancers and thrombophilia. Current guidelines recommend treatment for at least 3 months. In patients who have a reversible provoking factor such as surgery, anticoagulation beyond 3 months is not recommended.

A 39-year-old female presents to your office with progressive swelling in the right leg that has spread over the past 2 days. She feels well otherwise. Her past medical history is significant for adrenal insufficiency treated with oral prednisone, 5 mg daily, and fludrocortisone, 0.1 mg daily. An examination reveals a temperature of 37.8°C (100.0°F), a pulse rate of 88 beats/min, a blood pressure of 105/62 mm Hg, and redness, warmth, and swelling in the right lower extremity. You diagnose cellulitis and prescribe appropriate antibiotics. Which one of the following would be the most appropriate management of her corticosteroid regimen? A) Continue prednisone and fludrocortisone at the current dosages B) Increase prednisone to 15 mg daily and continue fludrocortisone at the current dosage C) Increase prednisone to 15 mg daily and increase fludrocortisone to 0.3 mg daily D) Stop prednisone until the infection resolves but continue fludrocortisone at the current dosage E) Stop both prednisone and fludrocortisone until the infection resolves

ANSWER: B Patients with chronic adrenal insufficiency, either primary or secondary, may not be able to mount a stress response to infection or surgical procedures. Common practice during minor infections is to increase the corticosteroid supplementation (SOR C). Fludrocortisone should be continued, but stress dosing is not necessary.

A 40-year-old female sees you because of burning upper abdominal and chest pain and an acidic taste in her mouth after nearly every meal. She has pain at night that sometimes keeps her awake, but she does not have any nausea, vomiting, difficulty swallowing, bloating, bloody stools, or weight loss. She does not smoke. Which one of the following would be the most appropriate next step? A) Test for Helicobacter pylori and treat if present B) Start a 4- to 8-week trial of a proton pump inhibitor C) Order abdominal ultrasonography D) Schedule esophagogastroduodenoscopy E) Refer to a surgeon to consider fundoplication

ANSWER: B Patients with symptoms typical for GERD can be treated conservatively initially unless there are warning signs such as anemia, weight loss, evidence of bleeding or obstruction, dysphagia, or persistent symptoms despite maximal treatment, or the patient is age 50 or over. In the absence of any of these concerns, medical therapy with a proton pump inhibitor can be initiated. While H2 histamine blockers can also treat reflux symptoms they are somewhat less effective, and stepwise therapy may increase costs. Routine testing for Helicobacter pylori in patients with GERD alone is not recommended because treating H. pylori has been shown in some studies to increase esophagitis and GERD symptoms. However, in the presence of dyspepsia (fullness, bloating, nausea), which can be associated with GERD, testing for and treating H. pylori is expected to be beneficial. This patient has classic signs and symptoms of GERD and abdominal ultrasonography would not be likely to reveal any helpful findings. In the presence of warning signs, esophagogastroduodenoscopy would be indicated to evaluate for a more serious pathology. Surgical intervention for GERD should be reserved for patients who fail maximal medical therapy or patients who are unable to take proton pump inhibitors.

A 1-day-old newborn is brought to your office for a routine examination. His parents report that he is well. The prenatal course and delivery were unremarkable. An examination is normal except for a 1-cm wide dimple on the sacrum, 1 cm superior to the anus. The dimple has a tuft of dark hair. At this point you would recommend A) a follow-up examination in 1 month B) ultrasonography C) MRI D) a fistulogram/sinogram E) a dermatology consultation

ANSWER: B Recognizing clinically significant abnormalities on the newborn examination is important. Newborns with small sacral dimples located far from the anal verge, without other skin findings such as hair, do not need imaging to rule out spinal dysraphism (tethered cord). While the exact parameters of what is considered large (>0.5 cm diameter) and close (within 2.5 cm of the anal verge) can easily be found in reference materials, the dimple described here is clearly concerning and needs imaging. Ultrasonography can accurately and safely detect spinal dysraphism in these cases.

A 50-year-old female with diabetes mellitus presents with an insidious onset of right shoulder pain. She does not recall any injury. The pain is a dull ache that is difficult for her to localize. She has noticed a decreased range of motion, particularly with reaching overhead, and it seems to be getting progressively worse. She has not had any fevers, chills, numbness, weakness, or other joint involvement. There is no tenderness to palpation anywhere, including the acromion, acromioclavicular joint, or biceps tendon. The patient has a marked loss of active and passive range of motion in shoulder flexion, abduction, adduction, and internal rotation. A shoulder radiograph is unremarkable. Which one of the following is the most likely diagnosis? A) Acromioclavicular arthropathy B) Adhesive capsulitis C) Glenohumeral arthritis D) Rotator cuff tendinopathy E) Subacromial bursitis

ANSWER: B Risk factors for adhesive capsulitis include female sex, age between 40 and 60 years of age, diabetes mellitus, and hypothyroidism. This is a clinical diagnosis of exclusion that is based on physical findings of loss of active and passive range of motion. Glenohumeral arthritis may present with similar findings but is ruled out by normal radiographs. A patient with acromioclavicular arthritis will have radiographic findings and a painful cross-arm test. Rotator cuff disease and bursitis typically present with painful active range of motion but preserved passive range of motion and tenderness to palpation.

A 42-year-old male with a history of alcohol abuse was admitted to the hospital last night with mid-epigastric pain and tenderness. The hospital evaluation included an elevated lipase level and a normal ultrasound examination. He was diagnosed with acute pancreatitis, placed on NPO status, and started on intravenous fluids and pain control. This morning he still has moderate epigastric pain and tenderness, and mild nausea but no vomiting. He says he would like to try eating some food. Which one of the following would you recommend at this time? A) Remaining NPO until the pain and tenderness have improved B) Allowing the patient an oral diet as tolerated C) Initiating nasogastric feeding D) Initiating nasojejunal tube feeding E) Initiating total parenteral nutrition

ANSWER: B The American Gastroenterological Association guidelines on acute pancreatitis recommend initiating oral feedings early in the course in order to protect the gut-mucosal barrier, which may limit infectious complications and does not increase hospital length of stay or other complications. Nasogastric or nasojejunal tube feeding may be considered at 3-5 days if oral feedings are not tolerated. Total parenteral nutrition is indicated only when enteral feedings cannot supply adequate caloric intake or are not possible for other reasons. The incidence of single or multiple organ failure or infected necrosis is significantly increased with the use of total parenteral nutrition.

A 7-year-old male received one dose of trivalent inactivated influenza vaccine at another health care facility 5 weeks ago. This was the first time he received influenza vaccine, and it resulted in soreness at the injection site. His mother reports that he has had mild hives after eating peanuts and eggs in the past. Your office has stocked only quadrivalent inactivated influenza vaccine. Which one of the following would you recommend? A) No further influenza immunization this year B) Immunization now with quadrivalent inactivated vaccine C) Delaying immunization until trivalent inactivated vaccine can be given D) Delaying immunization until 8 weeks after the first vaccine E) No immunization because of a potential egg allergy

ANSWER: B The CDC's Advisory Committee on Immunization Practices recommends that patients with egg allergy receive influenza vaccination. Previously unvaccinated patients ages 6 months to 8 years should receive two doses of either trivalent or quadrivalent vaccine separated by 1 month.

Which one of the following is the counseling strategy in the Stages of Change Model? A) Quickly establishing rapport with a patient to improve compliance with recommendations for change B) Assessing the patient's motivation for change and determining where they are in the process C) Focusing on a specific aspect of a problem and offering strategies for coping D) Providing education regarding the behavior in which a change is recommended E) Providing direct advice regarding steps for making a lifestyle change

ANSWER: B The Stages of Change Model assesses the patient's motivation for change and determines which stage of the change process the patient is in. The stages include precontemplation, contemplation, preparation, action, and maintenance. Understanding this helps guide counseling strategies for each individual patient.

Which one of the following is recommended regarding oral fluoride supplementation? A) Supplementation for all children whose primary water source is well water B) Starting supplementation at 6 months of age if the primary water supply is fluoride deficient C) Starting supplementation at 2 years of age even in children who have received topical fluoride varnish D) No supplementation if fluoride varnish is applied once all primary teeth have come in or by 3 years of age, then yearly thereafter E) No supplementation for most children

ANSWER: B The U.S. Preventive Services Task Force recommends oral fluoride supplementation for the prevention of dental caries beginning at age 6 months for children whose primary water supply is fluoride deficient (B recommendation). Well water may be fluoridated naturally depending on the aquifer, but the water is highly variable and should be tested before deciding on the need for supplementation. Testing well water is also advisable because excessive fluoride may lead to fluorosis of the bones. Bottled water is variable, making it undependable as an adequate source of fluoride. Topical fluoride, in toothpaste or applications of fluoride varnish, is effective in preventing tooth decay in children and can be used in addition to properly fluoridated water. Twice-yearly application of fluoride varnish to primary teeth should begin when the first tooth comes in and repeated every 6 months thereafter in children (SOR B).

A 30-year-old female presents with pain over the proximal fifth metatarsal after twisting her ankle. Radiographs reveal a nondisplaced tuberosity avulsion fracture of the fifth metatarsal. Which one of the following would be the most appropriate initial management? A) A short leg walking boot B) A compressive dressing with weight bearing and range-of-motion exercises as tolerated C) A posterior splint with no weight bearing, and follow-up in 3-5 days D) A short leg cast with no weight bearing E) Surgical fixation

ANSWER: B The fifth metatarsal has the least cortical thickness of all of the metatarsals. There are strong ligaments and capsular attachments on the proximal fifth metatarsal that can put significant stress on this area of the bone, leading to fractures. Nondisplaced tuberosity fractures can generally be treated with compressive dressings such as an Aircast or Ace bandage, with weight bearing and range-of-motion exercises as tolerated. Minimally displaced (<3 mm) avulsion fractures of the fifth metatarsal tuberosity can be treated with a short leg walking boot. If the displacement is >3 mm, an orthopedic referral is warranted

A 65-year-old female with hypertension, osteoporosis, and GERD presents to your office for a well woman visit. She reports no new symptoms or concerns. A review of laboratory work performed prior to her visit reveals lipid levels at goal, normal glucose and sodium levels, a calcium level of 10.6 mg/dL (N 8.6-10.3), an albumin level of 4.1 g/dL (N 3.6-5.1), and a 25-hydroxyvitamin D level of 35 ng/mL (N 20-50). Her calcium level was 10.5 mg/dL on a basic metabolic panel 6 months ago. The patient's medications include hydrochlorothiazide, 12.5 mg daily; lisinopril (Prinivil, Zestril), 10 mg daily; alendronate (Fosamax), 70 mg weekly; omeprazole (Prilosec), 20 mg daily as needed; and vitamin D, 2000 IU daily. The patient's blood pressure is 110/60 mm Hg. An examination is normal. In addition to ordering follow-up laboratory studies and scheduling a follow-up visit in 1 month, which one of the following would be most appropriate? A) Discontinue alendronate B) Discontinue hydrochlorothiazide C) Discontinue lisinopril D) Discontinue omeprazole and begin ranitidine (Zantac) E) Increase vitamin D to 5000 IU daily

ANSWER: B This patient has hypercalcemia with a normal albumin level. Hydrochlorothiazide can cause drug-induced hypercalcemia. Alendronate, lisinopril, and omeprazole do not cause hypercalcemia. A high vitamin D level can cause hypercalcemia, so increasing vitamin D is not appropriate at this point. A laboratory evaluation can help differentiate between PTH- and non-PTH-mediated hypercalcemia.

A nulliparous 34-year-old female comes to your office for evaluation of fatigue, hair loss, and anterior neck pain. These symptoms have been gradually worsening for the past few months. Her past medical history is unremarkable. She has gained 5 kg (11 lb) since her last office visit 18 months ago. Examination of the thyroid gland reveals tenderness but no discrete nodules. Her TSH level is 7.5 U/mL (N 0.4-4.2), her T4 level is low, and her thyroid peroxidase antibodies are elevated. Which one of the following would be the most appropriate next step? A) Continue monitoring TSH every 6 months B) Begin thyroid hormone replacement and repeat the TSH level in 6-8 weeks C) Begin thyroid hormone replacement and repeat the TSH level along with a T3 level in 6-8 weeks D) Order ultrasonography of the thyroid E) Order fine-needle aspiration of the thyroid

ANSWER: B This patient has thyroiditis with biochemical evidence for autoimmune (Hashimoto's) thyroiditis. The most appropriate plan of care is to begin thyroid hormone replacement and monitor with a repeat TSH level 6-8 weeks later. It is not necessary to include a T3 level when assessing the levothyroxine dose. There is no need to routinely order thyroid ultrasonography when there are no palpable nodules on a thyroid examination. Fine-needle aspiration may be necessary to rule out infectious thyroiditis when a patient presents with severe thyroid pain and systemic symptoms.

A patient is brought to your office by his spouse because of anger issues over the last 6 weeks that are affecting his marriage and his work. He tells you he has started running 5 miles per day, but is sleeping poorly, and that his mind has been racing during this period. He recently went on a shopping spree and reached the maximum spending limit on his credit card. He has been using alcohol to calm down. He refuses to see a psychiatrist. The most appropriate medication to initiate at this time is A) bupropion (Wellbutrin) B) lithium C) lorazepam (Ativan) D) sertraline (Zoloft) E) trazodone (Oleptro)

ANSWER: B This patient is experiencing a manic or hypomanic episode. Therapeutic options include lithium, anticonvulsants such as divalproex, and antipsychotic medications such as olanzapine. Benzodiazepines such as lorazepam may be of minimal benefit. SSRIs such as sertraline can aggravate mania. Bupropion would help treat an associated depression and trazodone could possibly help the patient sleep, but these medications are ineffective for treating a manic/hypomanic episode.

A 54-year-old male develops chest pain while running. He is rushed to the emergency department of a hospital equipped for percutaneous coronary intervention. An EKG shows 3 mm of ST elevation in the anterior leads. He is diaphoretic and cool with ongoing chest pain. His blood pressure is 80/50 mm Hg, his pulse rate is 116 beats/min, and his oxygen saturation is 98% on room air. You would immediately administer A) a -blocker B) dual antiplatelet therapy and an anticoagulant C) intravenous fibrinolytic therapy D) an intravenous vasopressor

ANSWER: B This patient is likely experiencing an acute anterior wall myocardial infarction with possible incipient cardiogenic shock. Along with initiating the hospital's protocol for myocardial infarction, immediate treatment should include dual antiplatelet therapy with a 325-mg dose of nonenteric aspirin, a P2Y12 inhibitor (clopidogrel, prasugrel, or ticagrelor), and an anticoagulant (unfractionated heparin or bivalirudin). Given the possibility of cardiogenic shock, -blockers should not be used. Unless more than a 2-hour delay in percutaneous coronary intervention is expected, fibrinolytics should not be administered. An intravenous vasopressor is not indicated.

A healthy 49-year-old female presents to your office for a routine health maintenance visit. Since her last visit a year ago she has had only two menstrual periods. She reports sudden sensations of extreme heat in her face, neck, and chest that last just a few minutes but occur throughout the day. These symptoms are very bothersome and interfere with the quality of her sleep. Which one of the following would you recommend to relieve her symptoms? A) Black cohosh B) Combined estrogen and progesterone C) Compounded bioidentical hormones D) Micronized progesterone E) Testosterone

ANSWER: B This patient presents with typical vasomotor symptoms that can begin in perimenopause and affect sleep quality. Hormone therapy is the gold standard for treatment of vasomotor symptoms. Combination estrogen and progesterone therapy is highly effective for vasomotor symptoms and provides protection against uterine neoplasia. Although micronized progesterone decreases vasomotor symptoms there are no long-term studies to assess the safety of progestin-only treatment for menopausal symptoms. Compounded bioidentical hormone therapy creates safety concerns and is not a first-line therapy due to limited government regulation and monitoring, the potential for overdosing and underdosing, impurities or lack of sterility, and the lack of labeling describing risks. Testosterone alone is not FDA-approved for use in women. Additionally, it has not been shown to be beneficial for treatment of vasomotor symptoms in combination with hormone therapy and is associated with significant side effects. It may be useful for hypoactive sexual desire in postmenopausal women. There is insufficient data to recommend the use of herbal remedies such as black cohosh.

When titrating the dosage of opioids, the CDC recommends that you should also consider prescribing naloxone when the opioid dosage reaches what morphine milligram equivalent (MME) per day threshold? A) 30 B) 50 C) 80 D) 90 E) 100

ANSWER: B To mitigate the risk of opioid harm, it is essential to understand morphine milligram equivalents (MME). The evidence shows that the risk of an opioid overdose increases at the threshold of 50 MME/day. It is therefore recommended by the CDC that a prescription for naloxone be ordered when an opioid dosage reaches 50 MME/day, which is a high dosage. In general one should avoid prescribing 90 MME/day because of the substantially higher risk of an overdose at this dosage level.

A 28-year-old white female comes to your office at 37 weeks gestation with a 24-hour history of painful vesicles on the vulva. She does not have a past history of similar lesions. You make a presumptive diagnosis of genital herpes. Of the following, the most sensitive and specific test is A) exfoliative cytology (Tzanck test) B) a polymerase chain reaction (PCR) test C) an enzyme-linked immunosorbent assay (ELISA) D) HSV serology (IgG/IgM)

ANSWER: B When genital herpes occurs during pregnancy, the best method of diagnosis is either a tissue culture or a polymerase chain reaction (PCR) test, which is more sensitive. Enzyme-linked immunosorbent assays are sensitive, but not as sensitive or specific as PCR.

A gravida 2 para 0 at 34 weeks gestation presents to your office because of diffuse itching. She does not have any known allergies other than seasonal allergies, and she does not have any new contacts. An examination is normal other than some scattered excoriations, and there is no other distinct rash. She has tried moisturizers but her symptoms have not improved. Which one of the following would be most appropriate at this point? A) Monitoring for the development of a rash B) Liver function tests and serum bile acid levels C) Topical corticosteroids D) Oral antihistamines E) Varicella-zoster immune globulin

ANSWER: B Whenever a pregnant woman presents with pruritus without a primary rash, it is important to evaluate her for intrahepatic cholestasis of pregnancy. This diagnosis is associated with increased fetal mortality and warrants increased antenatal surveillance as well as possible induction by 35-37 weeks gestation. It is most appropriate to check for elevation of liver function tests and serum bile acids. Emollients, topical corticosteroids, and oral antihistamines can all be helpful for pruritus and certain rashes, but in this patient it is most important to promptly look for the cause of the pruritus. Varicella-zoster immune globulin would be indicated if she had no immunity to varicella and had been exposed to varicella or if she had a rash that was suspected to be chickenpox.

A 67-year-old female who recently moved to your city presents to your office as a new patient. Over the past year she has experienced wheezing and shortness of breath during her morning walks. She has a 35-pack-year smoking history and has been treated with antibiotics at least four times in the past year for respiratory infections. You suspect COPD and perform spirometry before and after a bronchodilator treatment. Which one of the following pulmonary function test results would confirm COPD in this patient? A) A reversible FEV1/FVC ratio <55% B) A reversible FEV1/FVC ratio >75% C) An irreversible FEV1/FVC ratio <65% D) An irreversible FEV1/FVC ratio >85%

ANSWER: C A large cohort study indicated that the Global Initiative for Chronic Obstructive Lung Disease criterion (FEV1/FVC ratio <70%) is more sensitive for COPD in individuals 65 years and older compared to the American Thoracic Society criteria (SOR C). COPD is present if the FEV1/FVC ratio is reduced to <70% and is irreversible with bronchodilator therapy. A reversible response to bronchodilator therapy is more consistent with asthma.

A 72-year-old male with a past history of hypertension, COPD, and pulmonary embolism presents with nonspecific symptoms including fatigue and syncope. You suspect he has pulmonary hypertension. Which one of the following would be the most appropriate initial test? A) Pulmonary function tests B) Chest CT with contrast C) Echocardiography D) A coronary calcium scan E) Right heart catheterization

ANSWER: C According to national guidelines echocardiography is the preferred initial noninvasive testing modality when pulmonary hypertension is suspected (SOR C). Pulmonary function tests provide helpful information in regard to pulmonary capacity but are not necessarily diagnostic of pulmonary hypertension. CT of the chest with contrast will not provide pulmonary pressures but may assist in the detection of pulmonary emboli. A coronary calcium scan may be indicated to evaluate for coronary artery disease but it is not a diagnostic test for pulmonary hypertension. Although right heart catheterization would provide pulmonary pressure values it is considered more invasive than echocardiography and is not always necessary for making the diagnosis.

A 52-year-old female sees you because of hair loss. An examination reveals diffuse thinning of the hair along the vertex, with sparing of the frontal hairline. Follicular orifices are still visible. Which one of the following is the most likely explanation for these findings? A) Alopecia areata B) Anagen effluvium C) Androgenetic alopecia D) Tinea capitis E) Trichorrhexis nodosa

ANSWER: C Androgenetic alopecia is associated with bitemporal thinning of the frontal and vertex scalp in men, but in women the frontal hairline is spared and hair thinning is most apparent at the vertex. There is often a family history of hair loss in patients with androgenetic alopecia. Alopecia areata results in acute, patchy hair loss. Anagen effluvium results in diffuse hair loss days to weeks after exposure to chemotherapeutic agents. The incidence of anagen effluvium after chemotherapy is estimated at 65%. Tinea capitis is a dermatophyte infection of the hair shaft and follicles that results in patchy hair loss and requires systemic antifungal treatment. Trichorrhexis nodosa is characterized by breaks in the hair secondary to trauma or because of fragile hair, often due to excessive brushing, heat application, or hairstyles that lead to pulling on hairs.

A 63-year-old male sees you after carotid ultrasonography at a local health fair showed a 50% occlusion of his left proximal internal carotid artery. He has no significant past medical history and has never had a TIA or stroke. In addition to a healthy diet and exercise, you would recommend A) no further treatment or follow-up B) observation, and repeat ultrasonography in 1 year C) statin therapy, and repeat ultrasonography in 1 year D) statin therapy and referral to a vascular surgeon for consideration of a carotid artery stent E) statin therapy and referral to a vascular surgeon for consideration of carotid endarterectomy

ANSWER: C Asymptomatic carotid artery disease is considered a coronary artery disease risk equivalent; therefore, statin therapy is indicated. Repeating ultrasonography annually to monitor for progression of the disease and to guide intervention is also considered reasonable. According to the 2014 guidelines for the primary prevention of stroke issued by the American Heart Association/American Stroke Association, prophylactic carotid artery stenting might be considered in highly selected asymptomatic patients with >70% carotid stenosis, but the effectiveness of this intervention compared with statin therapy alone is not well established. The guidelines also state that it is reasonable to consider carotid endarterectomy for asymptomatic patients with >70% stenosis if the risks of perioperative complications are low.

At a routine well child check, the mother of an 18-month-old female expresses concern about the child's development. Which one of the following should prompt consideration of a developmental delay? A) A vocabulary of less than six words B) Failure to point to pictures or body parts when named C) Inability to follow one-step directions D) Inability to run well E) Inability to copy a vertical line

ANSWER: C At 18 months of age a child should follow one-step directions. Approximately 90% of 18-month-olds say at least three words, and 50%-90% say six words. The ability to point to body parts or pictures after they are named is expected at 2 years of age. Not walking at 18 months would be a red flag for delay, but running well may not yet be accomplished. At 18 months a child would be expected to scribble spontaneously but not to copy a vertical line.

A 25-year-old male presents to your office with anxiety. During questioning he reveals that he struggles with opioid addiction since he underwent an appendectomy 2 years ago, and is anxious because of random drug screens in his workplace. He averages using about 30 mg of hydrocodone daily and goes through cycles of use and withdrawal. Screening is negative for alcohol and substance use disorders other than opioids, and for depression or other mental health disorders. You consider maintenance treatment for opioid use disorder with buprenorphine. For this patient, buprenorphine therapy A) is inferior to methadone maintenance therapy B) should not be combined with naloxone therapy because of potential side effects C) should be initiated when he is in mild to moderate withdrawal from opioids D) can be initiated only after inpatient detoxification E) should be continued for 1 year and then tapered

ANSWER: C Buprenorphine therapy is an important option for maintenance therapy for patients with opioid use disorder. It can be initiated in the outpatient setting but should be done when the patient is in mild to moderate withdrawal in order to avoid the risk of precipitated withdrawal (SOR C). Buprenorphine therapy is more convenient than methadone maintenance therapy and is equally as effective. Buprenorphine/naloxone combinations are preferred over buprenorphine monotherapy due to lower abuse potential, except when naloxone is contraindicated such as in patients who are pregnant or breastfeeding (SOR C). Because relapse rates are higher in patients who discontinue medication-assisted therapy for opioid use disorder, long-term use is recommended.

A 75-year-old male nursing home resident is brought to the emergency department with a cough and fever. His past medical history is significant for coronary artery disease, COPD, hypertension, and osteoarthritis. On examination he has a blood pressure of 145/90 mm Hg, a pulse rate of 84 beats/min, and an oxygen saturation of 89% on room air. A physical examination is remarkable for mildly labored breathing and crackles in his left lower lung field. A chest radiograph confirms left lower lobe pneumonia. He is admitted to the hospital for intravenous antibiotics. Which one of the following would be the most appropriate antibiotic treatment? A) Cefdinir only B) Piperacillin/tazobactam (Zosyn) only C) Ceftriaxone and azithromycin (Zithromax) D) Cefixime (Suprax) and vancomycin (Vancocin) E) Piperacillin/tazobactam, vancomycin, and ciprofloxacin (Cipro)

ANSWER: C Current recommendations state that nursing home-acquired pneumonia should be treated as community-acquired pneumonia unless patients have severe illness, chronic wounds, foreign bodies in the airway, a history of antibiotic use in the last 90 days or recent hospitalization, colonization with multidrug-resistant pathogens, or very low functional status, or reside in a facility with a high prevalence of multidrug-resistant pathogens. Community-acquired pneumonia should be treated with either a respiratory fluoroquinolone or an advanced macrolide plus a -lactam antibiotic. Doxycycline could also be used in place of the macrolide.

Which one of the following groups has the highest prevalence of syphilis? A) Baby boomers B) Incarcerated females C) Men who have sex with men D) Rural men 20-29 years of age E) Individuals with a history of illicit intravenous drug use

ANSWER: C Factors associated with increased prevalence rates for syphilis in the United States include a history of incarceration or commercial sex work, living in the southern or western United States, residing in a major metropolitan area, African-American ethnicity, and being a male younger than 29 years of age. The risk for syphilis infection is highest among men who have sex with men and among persons who are HIV-positive. The U.S. Preventive Services Task Force (USPSTF) recommends that asymptomatic, nonpregnant adults and adolescents who are at increased risk be screened for syphilis infection (A recommendation). The USPSTF also recommends that local community and socioeconomic factors be considered when identifying patients at increased risk for infection who should be screened.

To reduce overuse of antibiotics, the CDC promotes antibiotic stewardship. The recommended intervention is the implementation of an antibiotic time-out to improve outcomes when prescribing antibiotics in hospitals. When should an antibiotic time-out be scheduled when prescribing an antibiotic at the time a patient is admitted to the hospital? A) Before starting the initial antibiotic order B) 12-24 hours after the initial antibiotic order C) 48 hours after the initial antibiotic order D) 5-7 days after the initial antibiotic order E) Prior to an antibiotic order at discharge

ANSWER: C For patients started on empiric antibiotic therapy at hospital admission, the CDC recommends an antibiotic time-out 48 hours after the initial order to determine if it can be stopped or needs to be changed. The dose, route, and duration should also be reviewed. The rationale is that antibiotics are often ordered empirically at the time of admission, while cultures and other studies are also being ordered. The original empiric order should be reassessed, incorporating the results of these studies while considering the evolving clinical status of the patient. Studies show this reassessment with antibiotic modification does not reliably occur.

A 30-year-old female sees you for a routine health maintenance visit. She has myopia and says she is considering LASIK eye surgery and wants your advice. You tell her that LASIK is associated with A) a moderate reduction in problems with glare B) a moderate reduction in problems with dry eyes C) satisfactory improvement of vision in almost all patients D) prevention of presbyopia

ANSWER: C LASIK corrective vision surgery has become increasingly common over the last 20 years. A laser is used to cut a flap the size of a contact lens consisting of corneal epithelium and stroma. This flap is repositioned and heals without sutures. It is important to counsel patients on realistic expectations. Vision following the procedure may not be as clear as with glasses or contact lenses and some individuals still require external correction. Up to 40% of patients experience dry eyes following the surgery (SOR B). These symptoms may be worse in patients with chronic pain syndromes such as fibromyalgia, migraine, and irritable bowel syndrome (SOR C). Glares, halos, and starbursts may affect up to 20% of patients following LASIK. This may be especially bothersome at night (SOR B). LASIK does not correct age-related presbyopia (SOR C). Reading glasses may be necessary if this develops in certain patients. Overall, however, most patients are satisfied with their results and only 3% are unhappy with their vision following surgery (SOR C).

An 85-year-old male with mild dementia, stage 4 chronic kidney disease, and insulin-dependent diabetes mellitus has received a letter from his gastroenterologist stating that he is due for his next colonoscopy. He had a normal colonoscopy 10 years ago and has no known high-risk indications for surveillance beyond those recommended for the general public. Proceeding with a screening colonoscopy at this time is an example of A) compliance with recommended guidelines B) appropriate modification of recommended guidelines C) overscreening D) overdiagnosis E) overtreatment

ANSWER: C Overscreening is defined as the use of screening tests at ages younger or older than the range recommended by national guidelines or at shorter intervals than recommended. Current guidelines from the U.S. Preventive Services Task Force recommend screening for colon cancer starting at age 50 and continuing until age 75 (A recommendation). The decision to screen adults 76-85 years of age should be an individual one, taking into account the patient's overall health and prior screening history (C recommendation). In this case ordering a screening colonoscopy would be an example of overscreening because the patient is above the recommended upper age of 75 and he has multiple comorbid conditions that limit his life expectancy. Overscreening can result in increased cost, morbidity, and/or mortality, especially in older individuals. Overdiagnosis is when a diagnosis is made correctly, but the diagnosis is irrelevant because it will never cause symptoms or death during a patient's ordinarily expected lifetime. Overtreatment refers to unnecessary medical interventions.

A 43-year-old female comes to your office to establish care. She reports a long history of uncontrolled hypertension. She is taking several medications that have been prescribed over the years. Which one of the following medications should be discontinued 4-6 weeks prior to obtaining a plasma aldosterone/renin activity ratio? A) Hydralazine B) Potassium chloride C) Spironolactone (Aldactone) D) Terazosin (Hytrin) E) Verapamil (Calan)

ANSWER: C Patients with secondary hypertension frequently take several medications and are classified as having "resistant" hypertension. It is important to recognize that many antihypertensive medications can affect the results of the aldosterone/renin ratio. An aldosterone antagonist, such as spironolactone or eplerenone, can increase renin and aldosterone levels and should be discontinued 4-6 weeks before obtaining a ratio. Angiotensin receptor blockers and ACE inhibitors can increase renin levels. Oral potassium supplements should be continued or started in patients with hypokalemia, since aldosterone biosynthesis is dependent on potassium. Medications such as hydralazine, terazosin, and verapamil may be continued or started for blood pressure control because they do not affect aldosterone or renin levels.

The father of a healthy 14-year-old male calls you about a recent mumps outbreak in your community. The child never received the MMR vaccine because the parents declined the immunization despite extensive counseling about the topic. You advise the father that A) mumps typically starts with a cough, coryza, and conjunctivitis B) mumps causes a pruritic rash with fluid-filled blisters C) mumps can cause orchitis, possibly resulting in decreased fertility D) Koplik spots or whitish papules in the mouth are pathognomonic for mumps E) the MMR vaccine is not recommended for patients in this age range

ANSWER: C Prodromal symptoms of mumps include myalgia, fatigue, loss of appetite, fever, and headache. Parotitis is the most common manifestation. Infertility, meningitis, and encephalitis are serious complications of orchitis. Measles is characterized by cough, coryza, conjunctivitis, and Koplik spots. Varicella is characterized by a pruritic rash with fluid-filled blisters. MMR vaccine is indicated for this child.

An otherwise asymptomatic 7-year-old male has a blood pressure above the 95th percentile for gender, age, and height on serial measurements. Which one of the following studies would be most appropriate at this time? A) Renin and aldosterone levels B) 24-hour urinary fractionated metanephrines and normetanephrines C) Renal ultrasonography D) Doppler ultrasonography of the renal arteries E) A sleep study

ANSWER: C Renal parenchymal diseases such as glomerulonephritis, congenital abnormalities, and reflux nephropathy are the most common cause of hypertension in preadolescent children. Preadolescent children with hypertension should be evaluated for possible secondary causes and renal ultrasonography should be the first choice of imaging in this age group. Renin and aldosterone levels are indicated if there is a reason to suspect primary hyperaldosteronism, such as unexplained hypokalemia. Measurement of 24-hour urinary fractionated metanephrines and normetanephrines is used to diagnose pheochromocytomas, which are rare and usually present with a triad of symptoms including headache, palpitations, and sweating. Doppler ultrasonography of the renal arteries is useful for diagnosing renal artery stenosis, which should be suspected in patients with coronary or peripheral atherosclerosis or young adults, especially women 19-39 years of age, who are more at risk for renal artery stenosis due to fibromuscular dysplasia. Sleep studies are indicated in patients who are obese or have signs or symptoms of obstructive sleep apnea.

A 42-year-old female with diabetes mellitus comes to your office because of recurrent yeast infections. She is taking numerous agents in an attempt to lower her glucose level. Which one of the following classes of antidiabetic agents is associated with an increased risk for candidiasis? A) Biguanides such as metformin (Glucophage) B) DPP-4 inhibitors such as sitagliptin (Januvia) C) SGLT2 inhibitors such as empagliflozin (Jardiance) D) GLP-1 receptor agonists such as liraglutide (Victoza) E) Sulfonylureas such as glipizide (Glucotrol)

ANSWER: C SGLT2 inhibitors are known to cause an increased risk of yeast vaginitis because their mechanism of action involves blocking renal uptake of glucose, which results in an increase in glucosuria (SOR A). Common side effects of metformin include gastrointestinal upset. DPP-4 inhibitors have very few side effects. GLP-1 receptor agonists typically cause nausea and early satiety and weight loss. Sulfonylureas are associated with weight gain and hypoglycemia.

In asymptomatic patients with sarcoidosis, which one of the following organ systems should be examined yearly to detect extrapulmonary manifestations of the disease? A) Cardiac B) Neurologic C) Ocular D) Integumentary

ANSWER: C Sarcoidosis has numerous extrapulmonary manifestations. Because inflammation of the eye can result in permanent impairment and is often asymptomatic, patients require yearly eye examinations as well as additional monitoring with disease flares. Although skin involvement is common it is usually readily apparent and rarely has serious sequelae. Cardiac sarcoidosis can potentially lead to progressive heart failure and sudden death, but evaluation is needed only in patients who are symptomatic. Similarly, evaluation for neurologic involvement is needed only in patients who are symptomatic.

A 38-year-old female with a 6-month history of mild shortness of breath associated with some intermittent wheezing during upper respiratory infections presents for follow-up. You previously prescribed albuterol (Proventil, Ventolin) via metered-dose inhaler, which she says helps her symptoms. You suspect asthma. Pulmonary function testing reveals a normal FEV1/FVC ratio for her age. Which one of the following would be the most appropriate next step? A) Consider an alternative diagnosis B) Assess her bronchodilator response C) Perform a methacholine challenge D) Prescribe an inhaled corticosteroid E) Proceed with treatment for COPD

ANSWER: C Spirometry is central to confirming the diagnosis of asthma, which is characterized by a reversible obstructive pattern of pulmonary function. In this case the patient's FEV1/FVC ratio is normal, which neither confirms nor rules out asthma. A methacholine challenge is recommended in this scenario to assess for the airway hyperresponsiveness that is the hallmark of asthma. Methacholine is a cholinergic agonist. Bronchoconstriction (defined as a reduction in FEV1 20%) observed at low levels of methacholine administration (<4 mg/mL) is consistent with asthma. If the FEV1/FVC ratio is reduced on initial spirometry, a bronchodilator response should be tested. A fixed or partially reversible obstructive pattern suggests an alternative diagnosis such as COPD, and full reversal after bronchodilator use is consistent with asthma. Inhaled corticosteroids are not appropriate for intermittent asthma.

A 90-year-old male presents to the emergency department with chest pain, dyspnea, and diaphoresis. He has experienced these symptoms intermittently since his wife died last week. An EKG shows ST elevation in the anterior leads, and cardiac enzymes are elevated. An echocardiogram shows apical ballooning of the left ventricle. Cardiac catheterization does not reveal coronary vascular disease. You plan to discharge the patient after observation overnight. Which one of the following would be the most appropriate management of this patient's stress-induced (Takotsubo) cardiomyopathy after discharge? A) Home medications only B) A cardiac event monitor to detect any rhythm abnormalities C) A diuretic, ACE inhibitor, and -blocker until his symptoms and the abnormalities seen on the echocardiogram resolve D) A statin, diuretic, ACE inhibitor, and -blocker to be continued indefinitely E) Pacemaker placement

ANSWER: C Takotsubo cardiomyopathy, also known as stress-induced cardiomyopathy, can develop following emotional distress and is characterized by the abrupt onset of dysfunction of the left ventricle. The clinical presentation and laboratory studies can mirror acute coronary syndrome and should be treated similarly. Once symptoms and cardiac abnormalities resolve, treatment is no longer indicated and may be withdrawn if there are no signs of coronary disease. Because this patient currently has cardiomyopathic abnormalities, a diuretic, ACE inhibitor, and -blocker are indicated. Ambulatory cardiac monitors are not indicated for this patient with a known diagnosis of Takotsubo cardiomyopathy. A pacemaker is not indicated in the absence of arrhythmias caused by conduction abnormalities.

A 52-year-old female sees you because of concerns about developing lung cancer. She reports that she quit smoking last month after learning that her father has stage IV lung cancer. She had smoked a pack of cigarettes per day since she was 18 years old. She has no history of cough, shortness of breath, or weight loss. She is worried about developing lung cancer and wants to know how to "catch it early." Based on the recommendations of the U.S. Preventive Services Task Force, in addition to providing ongoing smoking cessation support, which one of the following should you recommend? A) A chest radiograph today B) Low-dose chest CT today C) Low-dose chest CT at age 55 D) No imaging, since she has already quit smoking E) No imaging, since she is female

ANSWER: C The 2013 U.S. Preventive Services Task Force lung cancer screening guidelines recommend annual low-dose CT screening for all adults between the ages of 55 and 80 who have a 30-pack-year smoking history and either currently smoke or have smoked within the past 15 years (B recommendation).

A 4-year-old male is brought to your office by his adoptive parents who are concerned about his poor behavior and intellectual development. The patient is in the 7th percentile for height and weight. There have been some indications of alcohol abuse by his biological mother. Which one of the following facial dysmorphologies would be most consistent with a diagnosis of fetal alcohol syndrome? A) A central chin "dimple" B) Low-set ears C) A smooth philtrum D) A widened vermilion border of the lower lip E) Elongated palpebral fissures

ANSWER: C The classic facial dysmorphologies associated with fetal alcohol syndrome are a smooth philtrum, shortened palpebral fissures, and a thin vermilion border of the upper lip. Two out of these three characteristics are required for the diagnosis of fetal alcohol syndrome. Low-set ears and a central chin dimple are not associated findings.

A 22-year-old male presents to your office the morning after falling onto his outstretched right hand as he tripped while leaving a bar. He has a deep, dull ache in the right wrist on the radial side. The pain is worsened by gripping and squeezing. On examination there is some wrist fullness and the wrist is tender to palpation over the anatomic snuffbox. Radiographs of the wrist are negative. Which one of the following would be most appropriate at this time? A) Rest, ice, compression, elevation, and NSAIDs with no specific follow-up B) Rest, ice, compression, elevation, and NSAIDs with a follow-up examination in 2 weeks C) Placement of a thumb spica splint, with a follow-up examination in 2 weeks D) CT of the wrist to detect an occult fracture E) Ultrasonography of the wrist to detect a ligament injury

ANSWER: C The history, symptoms, and physical examination findings in this case suggest a scaphoid fracture. The scaphoid bone is the most commonly fractured carpal bone and a fall on an outstretched hand can produce enough force to cause this fracture. This fracture is most common in males 15-30 years of age. The finding of anatomic snuffbox tenderness is highly sensitive but not specific for a scaphoid fracture. Initial radiographs often do not demonstrate a fracture. When there is a high clinical suspicion for a scaphoid fracture but radiographs are negative, it is reasonable to immobilize in a thumb spica splint and reevaluate in 2 weeks.

A 69-year-old female presents to your office with a 5-day history of cough and low-grade fever. She has a past history of hypertension and obstructive sleep apnea. Her daughter brought her in this morning because of worsening symptoms. The patient's temperature is 37.4°C (99.3°F), her blood pressure is 110/74 mm Hg, her pulse rate is 88 beats/min, her respiratory rate is 36/min, and her oxygen saturation is 95% on room air. She is alert and oriented to person, place, and time. A CBC and basic metabolic panel are normal except for an elevated WBC count of 12,500/mm3 (N 4300-10,800). A chest radiograph shows a right lower lobe infiltrate. This patient has a higher risk of mortality and should be considered for inpatient treatment due to her A) female sex B) underlying hypertension C) respiratory rate D) elevated WBC count E) abnormal chest radiograph

ANSWER: C There are several decision support tools to assist in predicting 30-day mortality for patients with community-acquired pneumonia. Calculating the number of high-risk markers can aid in deciding whether to admit the patient to the hospital. The risk of mortality increases with a respiratory rate 30/min, hypotension, confusion or disorientation, a BUN level 20 mg/dL, age >65 years, male sex, or the presence of heart failure or COPD.

A 69-year-old female with hypertension, hyperlipidemia, and coronary artery disease had a myocardial infarction 1 year ago that was treated with percutaneous stenting. She was recently diagnosed with atrial fibrillation and takes diltiazem (Cardizem) for rate control. She is also taking lisinopril (Prinivil, Zestril), atorvastatin (Lipitor), atenolol (Tenormin), and aspirin, 81 mg. Of the following, which option would be best for thromboembolism prevention in this patient? A) Continue the current medication regimen B) Increase the aspirin dosage to 325 mg daily C) Discontinue aspirin and begin apixaban (Eliquis) D) Add clopidogrel (Plavix)

ANSWER: C This patient has a CHA2DS2-VASc score of 3 (hypertension, age 65-74, female), which classifies her as high risk for thromboembolism. Oral anticoagulation is indicated for patients with a score of 2 or more, who are at high risk for pulmonary embolism/deep vein thrombosis (PE/DVT) (SOR C). Patients with a score of 0-1 have a low to medium risk and may use aspirin with or without clopidogrel. In patients with atrial fibrillation and stable coronary artery disease, novel oral anticoagulants are preferred (SOR A). They reduce the risk of reinfarction, stroke, and overall mortality in patients with a past history of myocardial infarction, and also help prevent PE/DVT.

A 75-year-old male presents to your office with the growth shown below that has developed under his eye over the past several months. He says the growth is painless but appears to be slowly enlarging. Which one of the following is the most likely diagnosis? A) Actinic keratosis B) Fibrous papule of the face C) Nodular basal cell carcinoma D) Sebaceous hyperplasia E) Seborrheic keratosis

ANSWER: C This patient has a nodular basal cell carcinoma, which has a pearly papular appearance with telangiectasia. Basal cell carcinoma is the most common cutaneous malignancy. The incidence increases with age and occurs most commonly in skin types 1 and 2. The tumors occur most frequently on the face, scalp, ears, and neck, and less frequently on the torso and extremities. While locally destructive, basal cell carcinoma rarely metastasizes. The skin becomes slightly rough with actinic keratosis, and a slight scale forms gradually. Sebaceous hyperplasia begins as elevated papules that eventually become dome-shaped and umbilicated. It consists of small tumors made up of sebaceous glands. Fibrous papule of the face is a variant of angiofibroma. It is usually 1-5 mm in diameter and most often appears on the nose. Seborrheic keratoses are benign skin neoplasms that are tan or black, well circumscribed, and have a stuck-on appearance.

A 67-year-old male is admitted to the hospital for community-acquired pneumonia. An examination reveals a temperature of 40.0°C (104.0°F), a respiratory rate of 50/min, a pulse rate of 110 beats/min, a blood pressure of 90/50 mm Hg, and an oxygen saturation of 88% on room air. The patient is confused and requires aggressive fluid resuscitation for hypotension and he is transferred to the intensive-care unit. He has no known additional risk factors or exposures. In addition to treatment with ceftriaxone and azithromycin (Zithromax), which one of the following medications is most likely to result in improved outcomes? A) Clindamycin (Cleocin) B) Levofloxacin (Levaquin) C) Methylprednisolone (Medrol) D) Oseltamivir (Tamiflu)

ANSWER: C This patient has severe community-acquired pneumonia based on clinical criteria, including an elevated respiratory rate, confusion, and hypotension requiring aggressive fluid resuscitation. Corticosteroids such as methylprednisolone have been shown to improve clinical outcomes such as length of stay, duration of antibiotic treatment, and the risk of developing adult respiratory distress syndrome. The preferred choice of antibiotic treatment for patients in the intensive-care unit is a -lactam antibiotic (ceftriaxone, cefotaxime) or ampicillin/sulbactam, plus a macrolide alone or a macrolide and a respiratory fluoroquinolone. The addition of levofloxacin is not necessarily preferred over just ceftriaxone and azithromycin. Clindamycin is not indicated in the absence of risk factors for anaerobic infection such as aspiration or alcoholism. Oseltamivir is not indicated in the absence of known or suspected influenza infection.

A 67-year-old male presents to your office because of fatigue and a syncopal episode. His vital signs in the office are normal. An examination reveals a harsh systolic murmur best heard over the second right intercostal space radiating to the neck. Echocardiography confirms your suspected diagnosis. Which one of the following is the only treatment that improves mortality with this condition? A) -Blockers B) Antimicrobial prophylaxis for bacterial endocarditis C) Aortic valve replacement D) Mitral valve repair E) Ventricular septal defect closure

ANSWER: C This patient has symptomatic severe aortic stenosis. The only treatment that improves this condition is aortic valve replacement (SOR B). Transcutaneous aortic valve replacement may be an alternative for patients who are not candidates for surgery. -Blockers must be used with caution due to the risk of depressing left ventricular systolic function. They have not been shown to improve mortality. Antimicrobial prophylaxis is not indicated unless a patient has undergone valve replacement or has a history of endocarditis (SOR C). Atrial fibrillation is common in patients with aortic stenosis and rate control is important. Symptomatic mitral valve regurgitation may require mitral valve intervention. However, these murmurs are holosystolic, high pitched, and best heard at the cardiac apex. A ventricular septal defect can cause a loud holosystolic murmur with an associated thrill heard best at the third/fourth interspace along the sternal border.

A 57-year-old male presents with left posterior heel pain that started several weeks ago. An examination reveals a nodular appearance at the site of insertion of the Achilles tendon to the calcaneus, and local tenderness of the distal tendon. Which one of the following would be the safest and most appropriate initial management? A) Local injection with a corticosteroid B) Local injection with platelet-rich plasma C) Physical therapy with eccentric calf-strengthening exercises D) Immobilization of the ankle in a cast or boot for 4-6 weeks E) Surgical debridement of the calcification about the distal tendon

ANSWER: C This patient has typical symptoms and findings of Achilles tendinopathy. The best management involves eccentric calf-strengthening exercises. A local injection with corticosteroids or with platelet-rich plasma is ineffective and may increase the risk of a tendon rupture. Immobilization and surgical debridement may be considered if more conservative therapies have failed.

A 69-year-old male presents for an annual health maintenance examination. His medical history is significant for hypertension and worsening back pain over the last 6 months. Laboratory studies reveal a hemoglobin level of 8.6 g/dL (N 14.0-18.0) and a mean corpuscular volume of 88 m 3 (N 80-94). The remainder of the CBC is normal. A peripheral smear and a ferritin level are both normal. A comprehensive metabolic panel is normal except for a serum creatinine level of 1.6 mg/dL (N 0.7-1.3). Which one of the following would be the most appropriate next step in the evaluation of this patient? A) A vitamin B12 level B) A haptoglobin level C) Serum protein electrophoresis D) Flow cytometry E) A bone marrow biopsy

ANSWER: C This patient's laboratory results and back pain suggest multiple myeloma (MM). He has a normocytic anemia and evidence of renal insufficiency, which can indicate MM. The laboratory findings along with worsening back pain indicate a need to order serum protein electrophoresis to look for MM. Flow cytometry is generally used in patients with an elevated WBC count and suspected lymphoma. The remainder of this patient's CBC is normal, which makes a bone marrow issue less likely. His mean corpuscular volume is also normal, making vitamin B12 deficiency less likely. A haptoglobin level could be ordered, but protein electrophoresis is a better choice because the peripheral smear demonstrated no evidence of a hemolytic problem.

A 37-year-old female presents with a 3-day history of left thumb pain. She first noted diffuse pain in her left thumb after a fall while skiing. She does not recall the mechanism of injury. The pain is greatest at the medial metacarpophalangeal joint, but there is no mass or instability. A radiograph does not show any fractures. An examination reveals mild laxity in her ulnar collateral ligament with 30° of joint opening when abduction stress is applied to the distal thumb while stabilizing the metacarpal. The right side has only 15° of joint opening. Which one of the following would be most appropriate at this point? A) Reassurance and follow-up if symptoms do not improve B) Anti-inflammatory medication for 7-10 days C) Immobilization of the left thumb in a thumb spica cast or brace for 6 weeks D) Immediate referral to an orthopedic surgeon for surgical repair

ANSWER: C Ulnar collateral ligament (UCL) disruption, or "skier's thumb," should be suspected in traumatic thumb injuries. It is important to recognize and treat this injury because it can lead to joint instability and a weak pincer grip if untreated. Initial treatment of UCL disruption involves immobilization of the affected thumb in a thumb spica cast or brace for 6 weeks. In the absence of an avulsion fracture, indications for referral to an orthopedic surgeon would include 35°-40° of joint opening or no end point on stress abduction testing. A Stener lesion (entrapment of the UCL outside of the adductor aponeurosis) would usually present with joint instability and a tender mass and would necessitate an orthopedic referral.

An 8-year-old male was brought to your office 7 months ago because of frequent diarrhea and abdominal discomfort that had been present for 1½-2 years. An examination revealed no abnormalities. A CBC, a serum iron level, and a metabolic panel were normal. Serology was consistent with celiac disease and a duodenal biopsy confirmed the diagnosis. The patient's symptoms resolved after his parents took him to a nutritionist who recommended a gluten-free diet. Today you see the child for preventive care and he remains asymptomatic. In addition to normal well child care, which one of the following would you recommend? A) No further testing B) Gradual reintroduction of gluten into the diet C) IgG antigliadin antibody D) IgA tissue transglutaminase antibody E) A duodenal mucosal biopsy to ensure healing

ANSWER: D Celiac disease affects approximately 1% of the U.S. population and can affect all ages. Individuals with northern European ancestry are most commonly affected. The condition is caused by autoimmunity induced by gluten-containing foods in susceptible individuals. Untreated celiac disease is associated with anemia, malabsorption, osteoporosis, weight loss, and gastrointestinal lymphomas. In children, growth stunting and delayed puberty are also common. With strict adherence to a gluten-free diet most complications from celiac disease are preventable and, in children, growth and development return to normal. The World Gastroenterology Organisation recommends annual monitoring of children and adolescents with celiac disease by anthropometry, pubertal development, and celiac serology. The preferred serology is IgA antibody to tissue transglutaminase (IgA anti-tTG) due to its balance of good performance and low cost. Failure of IgA anti-tTG titers to decrease in 6 months suggests continued ingestion of gluten. Repeat

A 72-year-old female presents for a routine health maintenance visit. Which one of the following medications in her current regimen places her at risk for osteoporosis? A) Atorvastatin (Lipitor) B) Hydrochlorothiazide C) Metformin (Glucophage) D) Phenytoin (Dilantin) E) Ranitidine (Zantac)

ANSWER: D Medications reported to be associated with osteoporosis and increased fracture risk include antiepileptic drugs, long-term heparin, cyclosporine, tacrolimus, aromatase inhibitors, glucocorticoids, gonadotropin-releasing hormone agonists, thiazolidinediones, excessive doses of levothyroxine, proton pump inhibitors, SSRIs, parenteral nutrients, medroxyprogesterone contraceptives, methotrexate, and aluminum antacids. Atorvastatin, hydrochlorothiazide, metformin, and ranitidine are not associated with osteoporosis.

A 10-year-old male is brought to the emergency department with a history of group A -hemolytic Streptococcus confirmed with a throat culture 2 weeks ago. His parents gave him antibiotics for 3 days then stopped them because his symptoms were gone. He now has a temperature of 38.9°C (102.0°F), a heart rate of 122 beats/min, and right hip and left knee pain with swelling. Which one of the following would be the most appropriate initial pharmacologic therapy for acute rheumatic fever in this patient? A) Acetaminophen B) Gabapentin (Neurontin) C) Hydrocodone D) Naproxen

ANSWER: D Once the diagnosis of acute rheumatic fever is made, NSAIDs such as aspirin or naproxen should be administered (SOR B). The therapeutic response to NSAIDs is often remarkable. Acetaminophen has not been shown to be a superior analgesic for acute rheumatic fever. Gabapentin is not indicated, especially considering that the pain does not have a neuropathic etiology. Opioids would not be considered first-line treatment because of their adverse effects and the dramatic response of NSAIDs alone.

A 46-year-old male who uses injectable heroin daily presents to establish primary care. He reports no symptoms and feels well overall. He does not smoke cigarettes, use alcohol, or use any drugs other than heroin. He works as an accountant for the federal government and has not had any arrests for illegal activity. In addition to assessing his readiness for treatment of his addiction, you should order testing for A) HIV only B) HIV, hepatitis B, and hepatitis C C) HIV, hepatitis B, hepatitis C, and TB D) HIV, hepatitis A, hepatitis B, hepatitis C, and TB E) HIV, hepatitis A, hepatitis B, hepatitis C, TB, gonorrhea, and Chlamydia

ANSWER: D Persons who inject drugs are at increased risk for HIV, hepatitis A, hepatitis B, hepatitis C, and latent tuberculosis. These patients should be screened at the initial visit and treated for any conditions found, according to routine guidelines. If titers are low or absent for hepatitis A or B, the patient should be vaccinated. Although gonorrhea and Chlamydia screening is recommended for females under 25 years of age who use injectable drugs, it is not appropriate in this asymptomatic patient.

Which one of the following is the most common cause of iron deficiency anemia in premenopausal women? A) Blood donation B) Peptic ulcer disease C) Hematuria D) Abnormal uterine bleeding E) Colon cancer

ANSWER: D Abnormal uterine bleeding is the most common cause of iron deficiency anemia in premenopausal women, accounting for 20%-30% of cases. Gastrointestinal causes are less common but should be considered if the gynecologic evaluation is normal or the anemia fails to resolve with iron supplementation. Blood donation and hematuria are less common causes.

An 88-year-old female hospice patient is experiencing significant agitation and delirium. After attempting conservative treatments and ruling out reversible causes of her symptoms, which one of the following pharmacologic agents would be most appropriate? A) Alprazolam (Xanax) B) Amitriptyline C) Diphenhydramine (Benadryl) D) Risperidone (Risperdal)

ANSWER: D Agitation and delirium are common end-of-life symptoms. It is important to assess for treatable causes, including constipation, urinary retention, uncontrolled pain, and adverse medication effects. The antipsychotic medication risperidone is effective for treating agitation and nausea at this stage, but dosing is much lower than when this medication is used for psychiatric disorders. Benzodiazepines can provoke increased agitation and should be used with caution; however, they can be useful for treating significant end-of-life anxiety. Generally, a longer-acting form such as lorazepam would be a better choice than short-acting alprazolam. Amitriptyline and diphenhydramine can both cause urinary retention, potentially leading to delirium and agitation.

According to state child abuse mandatory reporter laws, which one of the following is the threshold that must be met for physicians to make a report to local child protective services or law enforcement? A) If they cannot prove that abuse or neglect did not occur B) If they consider the possibility of abuse or neglect, even if further evaluation indicates that it is unlikely C) If they observe signs or symptoms that may occur in cases of abuse or neglect, but that are nonspecific for abuse or neglect D) If they suspect that a child has experienced abuse or neglect E) Only if they have proof that abuse or neglect has occurred

ANSWER: D Although each state has its own laws regarding obligations to report child abuse, all 50 U.S. states require physicians, whether as a specified professional group or as a part of universal mandated reporting, to report a suspicion of child abuse. The standard is generally suspicion or cause to believe that abuse has occurred. If the possibility of abuse is briefly considered but rejected, or if nonspecific signs are present that do not create a significant suspicion of abuse, this standard is not met. There is no burden of proof placed on the physician to make such a report.

A 52-year-old male business executive sees you for a routine health maintenance examination. He reports good health and does not have any chest pain, shortness of breath, or fatigue. He has no family history of premature vascular disease. A screening test for diabetes mellitus was normal 2 years ago. His lifestyle is sedentary, and you note that his weight is up 4 kg from last year with a current BMI of 33 kg/m2 . He smokes an occasional cigar. The patient tells you he wants his heart checked out thoroughly because a friend suffered a major heart attack this year. In addition to counseling on diet, fitness, and healthy weight, you would recommend A) no EKG B) a stress EKG only C) a resting EKG and, if the findings are abnormal, follow-up with a stress EKG D) a resting EKG followed by a stress EKG

ANSWER: D Americans who live in rural areas face many health challenges compared to their counterparts who live in urban areas. This includes health care workforce shortage problems. The number of primary care and specialty care physicians, physician assistants, advanced practice nurses, registered nurses, dentists, and mental health professionals per 10,000 residents is lower. People who live in rural areas are also affected by socioeconomic factors. They tend to be older and poorer on average, with a higher percentage living below the poverty level. Rural areas also have higher unemployment rates. In 2017, 18.1% of rural populations were over the age of 64 compared to 14.3% of urban populations. Residents of rural areas are more likely to be uninsured and rely more heavily on the Supplemental Nutrition Assistance Program (SNAP) and Medicaid. Fewer 18- to 24-year-olds in rural areas have high school diplomas. Tobacco use is higher among rural youth, including cigarette smoking and smokeless tobacco use. Transportation difficulties are also more common. Health inequity issues are also challenges in rural areas, including higher rates of diabetes mellitus and coronary heart disease. Deaths related to drug overdoses have been rising and in 2017 overtook the death rate for people who live in urban areas. Family physicians are only 15% of the U.S. outpatient physician workforce nationwide, but they provide 42% of the care in rural areas.

Which one of the following is higher on average in rural areas compared to urban areas? A) The number of advanced practice nurses per 10,000 people B) The number of primary care physicians per 10,000 people C) The percentage of the population above the poverty line D) The percentage of the population age 65 and older E) The percentage of 18- to 24-year-olds with a high school diploma

ANSWER: D Americans who live in rural areas face many health challenges compared to their counterparts who live in urban areas. This includes health care workforce shortage problems. The number of primary care and specialty care physicians, physician assistants, advanced practice nurses, registered nurses, dentists, and mental health professionals per 10,000 residents is lower. People who live in rural areas are also affected by socioeconomic factors. They tend to be older and poorer on average, with a higher percentage living below the poverty level. Rural areas also have higher unemployment rates. In 2017, 18.1% of rural populations were over the age of 64 compared to 14.3% of urban populations. Residents of rural areas are more likely to be uninsured and rely more heavily on the Supplemental Nutrition Assistance Program (SNAP) and Medicaid. Fewer 18- to 24-year-olds in rural areas have high school diplomas. Tobacco use is higher among rural youth, including cigarette smoking and smokeless tobacco use. Transportation difficulties are also more common. Health inequity issues are also challenges in rural areas, including higher rates of diabetes mellitus and coronary heart disease. Deaths related to drug overdoses have been rising and in 2017 overtook the death rate for people who live in urban areas. Family physicians are only 15% of the U.S. outpatient physician workforce nationwide, but they provide 42% of the care in rural areas.

Which one of the following is an indication for considering atrial ventricular nodal ablation in a patient with atrial fibrillation? A) Hemodynamic instability B) Successful cardioversion to sinus rhythm C) Inability of the patient to be anticoagulated D) Atrial fibrillation refractory to medical therapy E) Persistent atrial fibrillation in a patient who has been successfully rate controlled and anticoagulated for several years

ANSWER: D Atrial ventricular nodal ablation is recommended for patients whose atrial fibrillation is refractory to medical therapy, and requires that patients be anticoagulated for at least 1 month prior to the procedure and for several months afterward (SOR C). Patients with atrial fibrillation who are hemodynamically unstable should be considered for emergent cardioversion (SOR C). Atrial ventricular nodal ablation is not necessary for patients successfully converted to sinus rhythm or for those who are successfully treated with medical interventions for rate control and anticoagulation.

A 3-month-old white male is brought to your office for evaluation of three lesions on his buttocks. The lesions began as vesicles that quickly progressed to flaccid bullae with sharp margins and no surrounding erythema. The lesions ruptured and formed yellow crusts, which oozed yellowish liquid. Which one of the following would be the most appropriate treatment? A) Oral azithromycin (Zithromax) B) Oral penicillin V potassium C) Topical bacitracin/neomycin/polymyxin B (Neosporin) D) Topical mupirocin (Bactroban) E) Topical nystatin

ANSWER: D Bullous impetigo is caused by Staphylococcus aureus, which produces a toxin responsible for flaccid bullae and is more likely to affect the intertriginous areas. This usually resolves within 3 weeks without scarring. Impetigo, either bullous or nonbullous, may be treated with topical antibiotics such as mupirocin. Because of emerging drug resistance, oral azithromycin and other macrolides should not be used. Oral penicillin is no longer recommended because it is less effective than other antibiotics such as amoxicillin/clavulanate, cephalexin, clindamycin, or dicloxacillin. Bacitracin/neomycin/polymyxin B is not indicated for either form of impetigo. Nystatin is recommended for Candida infections.

A 42-year-old bricklayer was diagnosed with acute bronchitis at an urgent care center 6-7 weeks ago. A chest radiograph was negative for pneumonia but revealed a solitary pulmonary nodule. No previous chest radiograph was available, so a follow-up chest radiograph was ordered for 4-6 weeks after the initial one. He is following up with you today to review those results. The patient's bronchitis has since resolved, and he feels well. He has no significant past medical history and does not take any medications. He has no constitutional symptoms and a physical examination today is within normal limits. He has a 10-pack-year cigarette smoking history and quit 15 years ago. If a lesion is noted on the chest radiograph, which one of the following characteristics would be most suspicious for malignancy? A) A diameter of 5 mm B) Concentric calcifications C) Doubling in size in less than 1 month D) A nonsolid "ground glass" appearance E) Smooth borders

ANSWER: D Characteristics that are more commonly associated with malignant lesions include a nonsolid "ground glass" appearance, a size >6 mm, noncalcified lesions, a lesion size or volume doubling time between 1 month and 1 year, and irregular or spiculated borders. Findings on a chest radiograph that are more commonly associated with benign lesions include a lesion size <6 mm, concentric or "popcorn-like" calcifications, doubling times of <1 month or >2-4 years, dense solid-appearing lesions, and lesions with smooth regular borders. Other diagnostic imaging modalities are also utilized, including CT and PET, and a biopsy is sometimes necessary to establish the diagnosis. Chest radiographs are still useful for monitoring patients with multiple findings that correlate most often with benign lesions. Informed decision making by the patient and family physician can sensibly guide the follow-up of patients with solitary pulmonary nodules without automatically referring them to specialists or ordering the most sophisticated imaging.

Which one of the following is the strongest indication for formal allergy testing? A) Erythema and tenderness surrounding an insect sting for 24 hours B) A fever for 3 days followed by a diffuse urticarial rash in a child C) A diffuse whole-body rash following ingestion of trimethoprim/sulfamethoxazole (Bactrim) in a patient with no documented drug allergies D) Recurrent or persistent upper respiratory symptoms E) Persistent epigastric pain following ingestion of tomato products

ANSWER: D Despite 10%-30% of the population being affected by allergic disease, allergy testing does have limitations and is most useful in certain clinical situations. Allergy testing can be helpful in patients with persistent sinus infections, allergic rhinitis, and poorly controlled asthma. Allergy testing for insect stings is indicated only following systemic/anaphylactic or large local reactions, not with limited localized reactions. Three days of fever followed by a diffuse urticarial rash likely represents a rash associated with a limited viral illness. Allergy testing for penicillin has a negative predictive value of 95%-98%. Testing for allergy to other antibiotics has a much lower sensitivity and specificity but does have limited use to help guide medication choices in patients with multiple allergies and when limited antibiotic options are available. Persistent epigastric pain following the ingestion of tomato products is more indicative of acid reflux symptoms rather than a tomato allergy.

A 35-year-old male presents with depression that started when his wife asked him for a divorce last month. A depression screen is positive and he has some passive suicidal ideation. He does not have any prior history of suicide attempts or a specific plan. He does not have any health issues, a family history of mental health issues, or a history of adverse childhood events. You would be most concerned that the patient will die from suicide if he A) has limited support from his family B) has no religious affiliation C) has a history of "cutting" as an adolescent D) has easy access to firearms E) was hospitalized for an appendectomy 2 months ago

ANSWER: D Easy access to a lethal means of suicide is a major risk factor for a successful suicide attempt. It is important to eliminate access to firearms, drugs, or toxins for a patient with any suicidal ideation. Other risk factors include, but are not limited to, a family history of suicide, previous suicide attempts, a history of mental disorders, a history of alcohol or substance abuse, and physical illness. Another risk factor in this patient is loss of a personal relationship. A history of borderline personality disorder (associated with cutting) is not a risk for successful suicide. Any support from family or friends is helpful, even if it is limited.

You have diagnosed a Chlamydia infection in a 24-year-old male. He reports having three sexual partners in the last month. He does not feel comfortable revealing their names or genders at this time. In almost all U.S. states, which one of the following would be the most appropriate plan for timely treatment of the involved parties with azithromycin (Zithromax) as a single dose? A) One prescription in the patient's name B) One prescription in the patient's name and instructions to the patient to inform his partners C) One prescription in the patient's name with three refills D) One prescription in the patient's name and three prescriptions for expedited partner therapy E) One prescription in the patient's name and three blank prescriptions

ANSWER: D Expedited partner therapy (EPT) is the clinical practice of treating the sex partners of patients diagnosed with Chlamydia or gonorrhea by providing prescriptions or medications to the patient to take to his/her partner without the health care provider first examining the partner. When patients have been diagnosed with gonorrhea or Chlamydia, EPT has been shown to reduce the overall burden of disease in a given population when the partners cannot be linked to care. In this case, as the partners' identities are unknown, it is impossible for the clinician to examine them or even contact them, so four prescriptions should be written, one with the patient's name and the other three for EPT. The CDC states that EPT is particularly effective in treating the female partners of infected males. The CDC considers having the partners visit a health care provider to be the optimal course of treatment but this is not often practically feasible due to a lack of resources or social factors. Although conventional practice is to treat only the patient, this does not provide timely treatment for the patient's partners. A prescription written to "EPT" can be filled at the pharmacy without the individual's name or date of birth. Kentucky and South Carolina are the only states that do not allow this practice. A prescription with three refills would be unethical, as presumably the physician would be advising the patient to distribute medications that had been prescribed to only the patient. Blank prescriptions would require the partners to reveal their identities, which may lead to a reluctance to fill the prescriptions.

A 48-year-old male is brought to the medical tent of a marathon after he became confused and dizzy in the middle of the race. On examination his rectal temperature is 41.1°C (106.0°F). Which one of the following should be initiated immediately? A) Passive cooling in an air-conditioned space B) Gradual cooling with a cold water spray and a fan C) Ice packs applied to the groin, neck, and axilla D) Full-body immersion in an ice bath E) Cold intravenous fluids

ANSWER: D Heatstroke can be nonexertional from prolonged exposure to a high heat index, or it can be exertional, as in this case. A core temperature >40°C (104°F) is consistent with heatstroke. In treating patients with either clinical variant of heatstroke, cold or ice-water immersion is the most effective treatment and should be initiated as soon as possible, without delaying for transfer to the hospital setting (SOR A). Treatment should continue until the core body temperature is <39°C (102°F). If cold water immersion is not possible other forms of cooling such as cold intravenous fluids, ice packs, cold water immersion of the extremities, and evaporative cooling have been shown to have some benefit. Once the body temperature is decreased patients should be transferred to a hospital for evaluation for known complications of heatstroke, including coagulopathy, renal and hepatic dysfunction, hypoglycemia, electrolyte disturbance, and rhabdomyolysi

A 69-year-old female presents to your office to discuss the results of a recent lung biopsy. While informing the patient she has lung cancer, appropriate communication strategies include A) limiting the number of family members that are present while delivering the bad news B) making your best educated guess for how much time the patient has left to live C) using layman's terms to describe detailed treatment options and the prognosis D) allowing adequate time to deliver the diagnosis in a private setting with limited interruptions

ANSWER: D Physicians should respect the patient's individual preferences for receiving bad news and allow adequate time to deliver the diagnosis in a private setting with limited interruptions. After delivering the news it is best to avoid extensive treatment details and making estimates of the patient's survival, and to focus instead on patient-directed questions and providing empathy. Patients should be allowed to have as few or as many family members and friends present as they desire at the time of communication, and this often varies depending on the patient's cultural background.

A 5-year-old female is brought to your office with a progressive rash on her legs (shown below) and buttocks. No rash is noted above the level of the mid-torso. Her mother also reports that the child had two episodes of bloody diarrhea 3 days ago. She also has abdominal pain and on examination she has abdominal tenderness with no rigidity but some voluntary guarding. You also note swelling and tenderness in her left wrist and right knee. A CBC, platelets, prothrombin time, and partial thromboplastin time are normal. A urinalysis reveals mild proteinuria and 5-10 RBCs/hpf. Which one of the following is the most likely diagnosis? A) Erythema infectiosum (fifth disease) B) Gianotti-Crosti syndrome C) Hemolytic uremic syndrome D) Henoch-Schönlein purpura E) Thrombotic thrombocytopenic purpura

ANSWER: D Henoch-Schönlein purpura is an IgA vasculitis that is usually diagnosed clinically. It presents as palpable purpura of the lower extremities without thrombocytopenia or coagulopathy. It is often associated with arthralgias and arthritis, abdominal pain, and renal dysfunction. It is self-limited and treatment is supportive only. Erythema infectiosum (fifth disease) can be identified by an erythematous rash on the cheeks and a lacy reticular rash on the extremities. Gianotti-Crosti syndrome is a sudden papular or papulovesicular eruption on the extensor surfaces of the arms, legs, buttocks, and face, and it is not purpuric. Hemolytic uremic syndrome presents with the classic triad of hemolytic anemia, thrombocytopenia, and kidney injury. Thrombotic thrombocytopenic purpura is rare in the pediatric age group.

A 13-year-old who was assigned at birth as a female has been diagnosed with gender dysphoria. He presents to your office after relocating because of bullying at school. The patient's parents are present, with past medical records, and fully support the desire of their child to affirm his gender as a male. On examination the patient has a sexual maturity rating of stage 3. Which one of the following steps would be appropriate for optimal support and therapy for this patient? A) Encourage conversion of the patient's gender identity to be congruent with the gender assigned at birth B) Recommend delaying gender-affirming treatment until he is at least 18 years old to prevent adverse psychosocial outcomes of puberty suppression C) Order genetic testing and ultrasonography to confirm the gender assigned at birth D) Offer gonadotropin-releasing hormone analogue treatment

ANSWER: D Hormonal intervention therapy should not be offered prior to puberty. However, stage 2 or 3 of sexual maturity is an appropriate time to consider gonadotropin-releasing hormone analogue therapy (SOR B). Individuals who receive hormone therapy often report less anxiety, increased self-esteem, and better quality of life (SOR A). Delaying gender-affirming therapy may actually increase emotional distress and gender-associated abuse. It is unethical and against recommended guidelines to convert a person's gender identity to the sex assigned at birth (SOR C). This patient has brought medical records to the appointment, making it unnecessary to order confirmatory testing, which may also induce emotional harm and additional medical costs.

Pregnancy increases the risk for all of the following dental disorders EXCEPT A) dental caries B) loose teeth C) gingivitis D) impacted molars E) periodontitis

ANSWER: D Impacted molars are a mechanical problem where the tooth does not emerge properly because of abnormal angulation or not enough room in the mouth. Pregnant women may experience impacted third molars (wisdom teeth) because they typically emerge in the late teens or early twenties, but pregnancy itself does not predispose to this condition. However, the risk of dental caries, loose teeth, gingivitis, and periodontitis all increase during pregnancy. Pregnant women are at higher risk for dental caries because the oral cavity is exposed to more gastric acidity due to vomiting with morning sickness, and acid reflux due to a lax esophageal sphincter and upward pressure from a gravid uterus. Increased levels of estrogen and progesterone during pregnancy can relax the ligaments and bones that support the teeth, leading to a temporary loosening of teeth. Gingivitis and periodontitis increase during pregnancy because of a fluctuation in estrogen and progesterone levels in combination with changes in oral flora and a decreased immune response. Periodontitis is of particular concern during pregnancy and should be treated because of its association with preterm birth and low birth weights.

An otherwise healthy 57-year-old male presents with mild fatigue, decreased libido, and erectile dysfunction. A subsequent evaluation of serum testosterone reveals hypogonadism. Which one of the following would you recommend at this time? A) No further diagnostic testing B) A prolactin level C) A serum iron level and total iron binding capacity D) FSH and LH levels E) Karyotyping

ANSWER: D In men who are diagnosed with hypogonadism with symptoms of testosterone deficiency and unequivocally and consistently low serum testosterone concentrations, further evaluation with FSH and LH levels is advised as the initial workup to distinguish between primary and secondary hypogonadism. If secondary hypogonadism is indicated by low or inappropriately normal FSH and LH levels, prolactin and serum iron levels and measurement of total iron binding capacity are recommended to determine secondary causes of hypogonadism, with possible further evaluation to include other pituitary hormone levels and MRI of the pituitary. If primary hypogonadism is found, karyotyping may be indicated for Klinefelter's syndrome.

A 38-year-old female presents for a health maintenance examination. Her laboratory results are unremarkable and she has no acute symptoms. She has no family history of cancer. According to the U.S. Preventive Services Task Force, this patient should be screened for which one of the following? A) Breast cancer B) Colon cancer C) Ovarian cancer D) Intimate partner violence E) Hepatitis C

ANSWER: D Intimate partner violence and the abuse of older or vulnerable adults are common in the United States. Immediate effects such as injury and death and other health consequences, including mental health conditions, substance abuse, sexually transmitted infections, unintended pregnancies, and chronic pain, often affect the traumatized individual. The U.S. Preventive Services Task Force recommends screening for intimate partner violence in all women of reproductive age (B recommendation). Screening for breast cancer, colon cancer, ovarian cancer, or hepatitis C is not appropriate for this patient at this time.

A 27-year-old gravida 3 para 2 presents in her first trimester for obstetric care. Her previous two pregnancies did not have any complications. She asks which immunizations she needs during this pregnancy. Which one of the following vaccines should you recommend? A) Hepatitis B B) MMR C) PCV13 D) Tdap E) Varicella

ANSWER: D Maternal vaccination helps prevent disease in newborn infants. Currently, two vaccines are recommended for women during each pregnancy: Tdap and inactivated influenza vaccine. Live vaccines, such as MMR and varicella, are contraindicated. Hepatitis B vaccine is recommended for high-risk women but not routinely. Pneumococcal vaccine is currently being studied for this use but is not recommended.

A 42-year-old female presents with a 2-week history of throbbing medial heel pain that is most painful when she first steps out of bed in the morning. The pain improves after she walks around for several minutes. Which one of the following is the most likely diagnosis? A) Achilles tendinopathy B) Calcaneal stress fracture C) Neuroma D) Plantar fasciitis

ANSWER: D Plantar fasciitis is the most common cause of heel pain, affecting more than 2 million people each year. The pain is typically worst when the patient first gets out of bed and improves with activity. Calcaneal stress fractures follow an increase in activity, and the pain tends to worsen with activity and is eventually present all of the time. Achilles tendinopathy is an aching pain that also worsens with increased activity, and there is often tenderness along the tendon. Neuromas present with a burning, tingling, or numb sensation and a painful lump.

You are administering a mental status examination to a 92-year-old male with suspected dementia. You give the patient a pencil and ask him to show how it is used. He gives you a bewildered look and eventually puts the pencil in his mouth and demonstrates using it as a toothbrush. This task assesses A) executive functioning B) gnosia C) orientation D) praxis E) visuospatial proficiency

ANSWER: D Praxis is the ability to carry out intentional motor acts and is commonly assessed by giving the patient a common object such as a hairbrush or pencil and asking the patient to show how it is used. A patient unable to carry out such motor acts is referred to as having apraxia (SOR C). Several other common components of the cognitive assessment will be impaired in persons with dementia. Executive functioning is the ordering and implementation of cognitive functions necessary to engage in appropriate behavior and is often assessed by asking a patient to draw a clock with the hands set at a certain time. Gnosia is the ability to name objects and their function and is often assessed by showing a patient a common object such as a pen, watch, or stethoscope and asking whether he or she can identify it and describe how it is used. Orientation is the ability of the patient to recognize his or her place in time and space. Orientation is commonly assessed by asking a patient the date, the current location, his or her name, and his or her place of birth. Visuospatial proficiency is the ability to perceive and manipulate objects and shapes in space. It is often assessed by asking the patient to copy intersecting pentagons or a three-dimensional cube on paper.

You suspect that a 28-year-old female patient may have a somatic symptom disorder, specifically a conversion disorder. Which one of the following is the most appropriate pharmacologic treatment of this disorder? A) Bupropion (Wellbutrin) B) Clozapine C) Selegiline (Eldepryl) D) Sertraline (Zoloft) E) Topiramate (Topamax)

ANSWER: D Somatic symptom disorders account for approximately 5% of primary care visits. Effective pharmacologic treatment includes sertraline and other SSRI-based therapy in addition to cognitive-behavioral therapy (SOR B). Bupropion, monoamine oxidase inhibitors such as selegiline, anticonvulsants such as topiramate, and antipsychotics such as clozapine are ineffective.

Among patients prescribed metered-dose inhaler treatments, technical errors in using the device are most likely in patients A) 15-30 years of age B) 30-60 years of age C) 60-75 years of age D) 75 years of age

ANSWER: D Studies have documented an increased rate of metered-dose inhaler (MDI) use errors in certain patient subgroups that could adversely affect the efficacy of treatment. For patients over the age of 15, the ability to demonstrate correct MDI use drops significantly with increasing age. One large study of MDI technique, after a minimum of 3 months of prescribed use, confirmed error frequencies of 61% in patients 15-30 years of age, 70% in patients 30-60 years of age, 77% in patients 60-75 years of age, and 86% in patients 75 years of age. Other patient characteristics also affect MDI error rates. A higher level of education and a diagnosis of asthma rather than COPD are both associated with fewer errors, whereas the frequency of errors is higher for females and lower-income patients. Disease severity and the presence of comorbidities has also been found to affect MDI use. The error rate is lower for patients who have had prior training, and the rate of proper usage relates directly to the duration of the training session.

A 71-year-old male is hospitalized for community-acquired pneumonia. He has a past medical history of hypertension and a small, stable abdominal aortic aneurysm. Which one of the following antibiotics is likely to increase the risk of rupture of this patient's aneurysm? A) Aztreonam (Azactam) B) Ceftriaxone C) Doxycycline D) Levofloxacin

ANSWER: D The FDA issued a warning that systemic fluoroquinolones can increase the occurrence of aortic dissections or ruptures. Drugs in this group should be avoided in patients with an existing aortic aneurysm or in patients at increased risk for developing an aortic aneurysm unless there are no other treatment options available. Patients at increased risk include those with peripheral vascular disease, hypertension, Marfan syndrome, or Ehlers-Danlos syndrome. Similarly, the use of systemic fluoroquinolones should be avoided in the elderly. Aztreonam, ceftriaxone, and doxycycline are not associated with this side effect (SOR A).

The U.S. Preventive Services Task Force recommends which one of the following for prevention of falls in community-dwelling adults 65 years of age who are at increased risk for falls? A) Empirical vitamin D supplementation B) Psychological evaluation and treatment programs C) In-home environmental evaluation and modification D) Regular participation in an exercise program

ANSWER: D The U.S. Preventive Services Task Force (USPSTF) recommends exercise interventions to prevent falls in community-dwelling adults 65 years of age who are at increased risk for falls (B recommendation). This recommendation is based on several studies that demonstrated improved fall-related outcomes for individuals from this population who participated in exercise programs. Strength and resistance exercises were specifically identified as beneficial. The evidence exists to support group-based exercises is less convincing. It is also recommended that clinicians selectively offer multifactorial interventions to prevent falls in this population, based on the possible small benefit and minimal risk (C recommendation). The USPSTF recommends against vitamin D supplementation to prevent falls in community-dwelling adults 65 years of age with the caveat that this applies only to those who are not known to have osteoporosis or vitamin D deficiency (D recommendation).

During an outbreak of head lice at a local school the principal asks you for advice to send home to the parents. In addition to treatment with topical permethrin (Nix), which one of the following would you recommend? A) Using a hairbrush to remove any lice or eggs B) Applying petroleum jelly to the hair and scalp C) Using a conditioner or a combined shampoo and conditioner when applying treatment D) Washing clothing and bedding in hot water and drying with hot air E) Treating household pets such as cats and dogs

ANSWER: D The appropriate recommendation for head lice is to wash any recently used bedding and clothing with hot water or expose them for 5 minutes to a temperature >130°F to kill lice and eggs. Items that cannot be washed or dried in this manner or dry-cleaned should be sealed in a plastic bag for 2 weeks. Additional or alternative treatments include topical ivermectin, benzoyl alcohol, malathion, and spinosad. Other recommended measures include removal of any visible nits (eggs) with a nit comb, not a brush. Topical petroleum jelly is not an effective treatment. Conditioners can interfere with the action of permethrin, decreasing its effectiveness. Human head lice are specific to humans, so pets are not affected.

Which one of the following laboratory tests is best for assessing thyroid function? A) Free T3 B) Reverse T3 C) Free T4 D) TSH E) Thyroid peroxidase antibody

ANSWER: D The best laboratory assessment of thyroid function is a serum TSH test, which is the preferred test for diagnosing primary hypothyroidism. If an elevated serum TSH level is detected and hypothyroidism is suspected, then a free T4 measurement would be indicated.

Which one of the following is the greatest risk factor for abdominal aortic aneurysm (AAA)? A) Male sex B) Female sex C) White race D) A long duration of smoking E) Having a first degree relative with an AAA

ANSWER: D The following factors have been found to increase the risk for developing an abdominal aortic aneurysm (AAA): a history of smoking, advanced age, above-average height, having a first degree relative with an AAA, a personal history of atherosclerosis, high cholesterol levels, and hypertension. Smokers have a seven times greater risk of developing an AAA compared with nonsmokers. This single factor outweighs all of the other risk factors except age. Although women are less likely to develop an AAA, they have a 2-3 times greater chance of an AAA rupturing if it is present. In addition to significantly increasing the risk for AAA development, current smoking increases the risk for further AAA expansion and rupture. Epidemiologic studies suggest that the duration of smoking influences the risk for AAA significantly more than the total number of cigarettes smoked. The U.S. Preventive Services Task Force currently recommends one-time screening for AAA in males between the ages of 65 and 75 who have ever smoked (B recommendation). There was not enough evidence to determine the risk and benefits of screening females with the same risk factors (I recommendation).

A 70-year-old female comes to your office with a 10-day history of a subjective fever at home, facial and tooth pain, sinus pressure, and a green nasal discharge. There has been no change in her symptoms. The patient has a history of allergic rhinitis treated with immunotherapy, and a history of developing hives while taking penicillin. On examination her temperature is 38.1°C (100.6°F). In addition to nasal saline and analgesics, which one of the following would be the most appropriate management? A) Standard-dose amoxicillin/clavulanate (Augmentin) B) High-dose amoxicillin/clavulanate C) Azithromycin (Zithromax) D) Doxycycline

ANSWER: D This patient has a history of symptoms consistent with acute bacterial rhinosinusitis that have persisted for 10 days, warranting empiric antibiotic therapy (SOR B). Doxycycline is an appropriate alternative to amoxicillin/clavulanate for a patient with a history of a reaction to penicillin. Macrolides and trimethoprim/sulfamethoxazole are not recommended as empiric therapy because of high rates of resistance.

A 47-year-old male presents to your office in a small town in Massachusetts. He has fevers, chills, and muscle and joint aches. He does not have any joint swelling, numbness, tingling, headache, or chest pain. He says he was working outside last Saturday and his wife removed a tick from his back that evening. On examination he is febrile and appears moderately ill. The examination is notable for an erythematous patch on his back. The remainder of the examination is unremarkable. Which one of the following would be the most appropriate next step? A) An enzyme immunoassay test B) A Western blot C) Doxycycline, 200 mg once D) Doxycycline, 100 mg twice daily for 10 days E) Doxycycline, 100 mg twice daily for 28 days

ANSWER: D This patient has early Lyme disease as evidenced by erythema migrans, fever, myalgias, and arthralgias. The treatment of early Lyme disease is doxycycline for 7-21 days. Single-dose doxycycline is used for prophylaxis to prevent Lyme disease after a tick bite in an area where Lyme disease is endemic. Longer treatment periods are required for Lyme arthritis and patients may require intravenous antibiotics. With this patient's clinical presentation, antibiotic treatment would be recommended and serologic testing is not needed. In clinical scenarios where diagnostic testing is needed, testing for early Lyme disease should be performed with a two-tier test starting with an enzyme immunoassay and, if positive, confirmed with a Western blot.

A 19-year-old female member of a college cross-country team presents with a 1-week history of right knee pain. She does not have any acute injury to the knee. An examination reveals no deformity and she has a normal gait. She has tenderness and subtle swelling localized 1 cm distal to the right medial joint line, and examinations of the knee and hip are otherwise normal. Which one of the following is the most likely diagnosis? A) Fibular head stress fracture B) Iliotibial band syndrome C) Medial meniscal tear D) Pes anserine bursitis E) Tibial apophysitis (Osgood-Schlatter disease)

ANSWER: D This patient has medial knee pain related to repetitive use, most likely caused by pes anserine bursitis. Iliotibial band syndrome is often related to overuse but causes pain in the lateral knee. The fibular head is also lateral to the knee joint. Osgood-Schlatter disease is also often related to overuse but causes pain at the insertion of the patellar ligament on the midline proximal tibia. A medial meniscal tear would localize to the medial joint line rather than distal to the joint line and would more likely be associated with positive findings from other examinations, such as a McMurray test.

A 38-year-old female presents for follow-up of a second hospitalization in the past 3 months for acute hepatitis, thrombocytopenia, and alcohol withdrawal symptoms treated with benzodiazepines. She says that prior to her hospitalization a week ago she had been drinking a half pint of vodka daily. She reports that her drinking has gradually increased over the past 10 years but increased significantly 6 months ago after she lost her job at a bar and grill. She knows her alcohol consumption is causing damage to her liver and tells you that her aunt died of alcoholic cirrhosis this year. Despite this knowledge she does not want to stop drinking at this time. She has looked into several alcohol cessation programs in the area but does not think that they are a good fit for her. She currently lives with her boyfriend who also uses alcohol and cocaine regularly. She is not currently speaking to her mother because they "don't see eye to eye." She tells you that she has not consumed alcohol since her discharge from the hospital 2 days ago. She reports that her abdominal pain, nausea, and vomiting have resolved and she is feeling well. The most likely diagnosis is A) alcohol intoxication B) alcohol withdrawal C) alcohol use disorder in early remission D) severe alcohol use disorder

ANSWER: D This patient presents with 6 out of 11 symptoms of alcohol use disorder within a 12-month period, including a strong desire or urge to use alcohol, recurrent alcohol use that has contributed to the inability to fulfill work obligations, continued alcohol use despite interpersonal problems with her family, continued alcohol use despite knowledge that it is causing physical damage to her liver, development of a tolerance to the effects of alcohol over time, and withdrawal symptoms that require treatment with benzodiazepines. Mild alcohol use disorder is defined by the presence of 2-3 of the 11 symptoms documented in the DSM-5, whereas 3-5 symptoms indicate moderate alcohol use disorder and 6 or more symptoms indicate severe alcohol use disorder. This patient has severe alcohol use disorder that is currently active. Early remission is defined as the absence of symptoms for at least 3 months but less than 12 months. She is not currently intoxicated, and she does not currently have withdrawal symptoms related to her alcohol use over a week ago.

A 24-year-old male presents with swelling of the right testicle. The pain started yesterday and has persistently worsened. A physical examination reveals swelling on the right side of his scrotum. His entire testicle is painful, and elevation of the scrotum improves his pain. Ultrasonography reveals hyperemia, swelling, and increased blood flow to the testis and epididymis. Which one of the following would be the most appropriate management? A) Doxycycline B) Levofloxacin (Levaquin) C) Trimethoprim/sulfamethoxazole (Bactrim) D) Doxycycline plus ceftriaxone E) Urgent surgical evaluation 6

ANSWER: D This patient presents with acute epididymitis. Typical symptoms develop gradually over 1-2 days with posterior scrotal pain and swelling. Additional symptoms may include fever, hematuria, dysuria, and urinary frequency. The pain may radiate to the lower abdomen. Physical examination findings may include tenderness of the epididymis and testis along with swelling of the scrotum. Elevation of the scrotum may decrease the pain (Prehn sign). Typical ultrasound findings include hyperemia, swelling, and increased blood flow to the epididymis. With testicular torsion the pain is often sudden in onset and severe, with associated nausea and vomiting and no other urologic symptoms. A physical examination often demonstrates a high-riding testis that may lie transversely in the scrotum. The cremasteric reflex may be absent. Ultrasound findings would demonstrate decreased or absent blood flow with testicular torsion. In sexually active adults <35 years of age, gonorrhea and Chlamydia are the most common causative organisms of acute epididymitis. Ceftriaxone, 250 mg intramuscularly or intravenously once, with oral doxycycline, 100 mg twice daily for 10 days, would be the appropriate treatment for acute epididymitis (SOR C). In men over the age of 35 or those with a history of recent urinary tract surgery or instrumentation, enteric organisms are the most likely cause and monotherapy with oral levofloxacin or ofloxacin for 10 days would be the recommended treatment.

At a routine health maintenance visit a 36-year-old female reports that she had pharyngitis while she was in high school and within 1-2 days of starting a course of penicillin she developed a nonpruritic rash. The penicillin was stopped and she was given an alternative antibiotic and told she had an allergy to penicillin. Six weeks ago she was inadvertently treated with amoxicillin in an evening clinic and had no adverse reaction. Which one of the following would you advise? A) She should still permanently avoid penicillin and its derivatives B) She should have skin testing to determine her penicillin allergy status C) She should have RAST testing to determine her penicillin allergy status D) She does not have a penicillin allergy and can use penicillin and its derivatives in the future

ANSWER: D This patient was inadvertently challenged with amoxicillin and did not develop an allergic reaction. Her demonstrated tolerance allows the future use of all penicillin antibiotics. Although a low-risk history allows for an amoxicillin challenge in the clinic under medical observation, patients with moderate-risk histories of an urticarial or other pruritic rash should undergo skin testing. If that testing is negative, it should be followed with an amoxicillin challenge under observation. RAST testing is not indicated for penicillin allergy testing.

A 57-year-old male with diabetes mellitus and hypertension presents with a 1-month history of pain in his hands and elbows. His hands are shown below. On examination they are tender and he has soft swelling of the wrists, metacarpophalangeal (MCP) joints, and proximal interphalangeal (PIP) joints. Plain films show mild, diffuse bony erosions in the MCP and PIP joints. Which one of the following is the most likely diagnosis? A) Dermatomyositis B) Osteoarthritis C) Psoriatic arthritis D) Rheumatoid arthritis E) Systemic lupus erythematosus

ANSWER: D This patient's clinical findings and radiographs indicate a diagnosis of inflammatory arthritis, most likely rheumatoid arthritis. Symmetric small-joint inflammatory arthritis is typical of rheumatoid arthritis and systemic lupus erythematosus (SLE), but bony erosions are not seen in SLE. Psoriatic arthritis can also affect small joints but is typically not symmetric. Dermatomyositis can present with a thick, bright red rash over the metacarpophalangeal (MCP) and interphalangeal joints (Gottron's sign) but is typically associated with proximal muscle weakness rather than joint pain or erosions that can be seen on radiographs. Osteoarthritis does not typically cause the soft-tissue swelling seen in the image. It usually affects the distal and proximal interphalangeal joints while sparing the MCP joints, and it results in osteophytes and joint space narrowing that can be seen on radiographs.

A 52-year-old female sees you because of a vaginal discharge. An examination reveals a malodorous, greenish-yellow, frothy discharge, and inflammation of the cervix and vagina. Which one of the following is the most likely diagnosis? A) Atrophic vaginitis B) Irritant/allergic vaginitis C) Bacterial vaginosis D) Trichomoniasis E) Vulvovaginal candidiasis

ANSWER: D Trichomoniasis classically presents as a greenish-yellow, frothy discharge with a foul odor. Erythema and inflammation of the vagina and cervix are often present and can include punctate hemorrhages (strawberry cervix). Atrophic vaginitis may cause a thin, clear discharge and is usually associated with a thin, friable vaginal mucosa. Irritant/allergic vaginitis causes burning and soreness with vulvar erythema but usually does not cause any significant discharge. Bacterial vaginosis more commonly presents as a thin, homogenous discharge with a fishy odor and no cervical or vaginal inflammation. Vulvovaginal candidiasis presents with white, thick, cheesy, or curdy discharge. Ref: Paladine HL, Desai UA: Vaginitis: Diagnosis and treatment. Am Fam Physi

A 45-year-old African-American male with a several-month history of fatigue and anorexia with weight loss is found to be anemic, with a hemoglobin level of 9.2 g/dL (N 14.0-18.0). Which one of the following microscopic findings on a peripheral blood smear would be most characteristic of vitamin B12 deficiency anemia in this patient? A) Microcytic, hypochromic RBCs B) Crescent- or sickle-shaped RBCs C) Basophilic stippling of RBCs D) Hypersegmented polymorphonuclear WBCs E) Schistocytes

ANSWER: D Vitamin B12 deficiency, which is most commonly related to a deficiency of intrinsic factor that is produced by the gastric mucosa and is necessary for absorption of vitamin B12 in the terminal ileum, is manifested as a megaloblastic macrocytic anemia. In addition to an elevated mean corpuscular volume, the classic finding on microscopy of the peripheral smear is the presence of multiple (usually five or more) segments in the WBC nuclei. Vitamin B12 levels are low and a high methylmalonic acid level helps to confirm the diagnosis. A methylmalonic acid level is recommended in patients who have borderline low vitamin B12 levels but are at risk for deficiency. Risk factors include chronic proton pump inhibitor use; chronic metformin therapy; chronic malnutrition due to alcoholism, chronic gastritis, or peptic ulcer disease; and diseases of the terminal ileum, such as Crohn's disease. People who have had gastric bypass surgery are also at risk for vitamin B12 deficiency, which is why supplementation should be provided in these patients. Microcytic, hypochromic RBCs are typical of iron deficiency anemia but may also be seen in anemia of chronic disease. Sickled or crescent-shaped cells are seen with sickle cell anemia, and basophilic stippling may be seen in anemia associated with lead toxicity. Schistocytes are seen in hemolytic anemia.

Which one of the following is an indication to consider ultrasonography to screen for developmental dysplasia of the hip in a newborn? A) A firstborn female infant with a vertex presentation at delivery B) A firstborn male infant with a vertex presentation at delivery C) A large-for-gestational-age male infant with a vertex presentation at delivery D) A male infant who was in a breech position until 28 weeks gestation E) A female infant with a first degree relative with developmental dysplasia of the hip

ANSWER: E Screening for developmental dysplasia of the hip (DDH) is somewhat controversial because the benefit of treatment remains somewhat unclear. Despite the widespread practice of screening for DDH, ethical newborn practices are difficult to determine. The American Academy of Family Physicians and the U.S. Preventive Services Task Force have found insufficient evidence to recommend routine screening for DDH. The American Academy of Pediatrics, however, recommends routine screening of all newborns with physical examination maneuvers, and targeted screening ultrasonography for infants who were breech in the third trimester, have a family history of DDH, or have a personal history of instability. Given this, decisions should be individualized. Additional risk factors include female sex, firstborn status, oligohydramnios, and a large-for-gestational-age infant.

Lymphadenopathy is associated with the highest risk of malignancy in which one of the following locations? A) Axillary B) Inguinal C) Posterior cervical D) Preauricular E) Supraclavicular

ANSWER: E In primary care practice the annual incidence of unexplained lymphadenopathy is only 0.6%. Only 1.1% of these cases are related to malignancy. This percentage increases with age. Supraclavicular adenopathy is associated with a high risk of intra-abdominal malignancy in both adults and children, with studies finding 34%-50% of these patients having a malignancy. Lymphadenopathy in the other locations listed is associated with a lower risk of malignancy.

A 93-year-old female presents to your office after a choking episode. She also says she has had gradually worsening difficulty with swallowing over the past several months. A modified barium swallow and esophagogram are shown below. Which one of the following is the most likely diagnosis? A) Cervical web B) Pharyngeal neoplasm C) Schatzki ring D) Tracheoesophageal fistula E) Zenker's diverticulum

ANSWER: E A Zenker's diverticulum is an oropharyngeal structural lesion and is most common in the elderly population, with an estimated prevalence of 1:1000 to 1:10,000 in this age group. The pathogenesis involves stenosis of the cricopharyngeus and increased hypopharyngeal pressure during swallowing that causes a pulsion diverticulum above the cricopharyngeus. The radiographic findings in this case are consistent with a Zenker's diverticulum. A pharyngeal neoplasm would not necessarily retain barium in this fashion. A cervical web or Schatzki ring would be identified by a short segment of narrowing without diverticula. Barium would be seen in the trachea with a tracheoesophageal fistula.

A 45-year-old female presents with pain in her right hand and wrist. She is right hand-dominant and most symptoms are in the right hand and wrist. She has been awakening at night with numbness of the hand that improves after she shakes her wrist. Her work as a software engineer involves lots of typing. On examination you note a positive Tinel's sign at the right wrist consistent with carpal tunnel syndrome. She has an important project at work over the next several months and will not be able to take any time off. Which one of the following treatments has the best evidence for delaying the need for definitive surgical therapy? A) Night splints B) Physical therapy C) Therapeutic ultrasound D) Oral corticosteroids E) Corticosteroid injection

ANSWER: E Carpal tunnel syndrome of mild to moderate severity can be treated nonsurgically. Patients with severe symptoms or nerve damage seen on electromyography should be referred for surgical therapy. Nonsurgical management options include splinting, physical therapy, therapeutic ultrasound, and corticosteroids (oral or injection). Oral prednisone, 20 mg daily, for 10-14 days improves symptoms and function compared with placebo, for up to 8 weeks, but oral corticosteroids are less effective than corticosteroid injections. In a 2013 double-blind, placebo-controlled, randomized clinical trial comparing methylprednisolone injection (40 mg and 80 mg) to saline injection, patients in the 80-mg injection group were less likely to have surgery at 12 months. While corticosteroid injections have the best evidence for delaying the need for surgery, night splints, physical therapy, and therapeutic ultrasound have some evidence of benefit. Splinting was found to be effective in a Cochrane review. It is low cost and safe and especially recommended in pregnancy-related disease. Limited evidence supports the use of physical therapy to treat carpal tunnel syndrome. Nerve glide exercises are simple hand and finger movements that are easy to learn, can be performed at home, and can be combined with other treatments such as splinting. Therapeutic ultrasound also has limited evidence of benefit. It requires an experienced therapist and requires multiple sessions, typically 5 days/week for 2-4 weeks.

A 30-year-old female presents for follow-up after an emergency department visit for an episode of symptomatic supraventricular tachycardia that was diagnosed as Wolff-Parkinson-White syndrome. Which one of the following would be most appropriate for the initial long-term management of this patient? A) Adenosine (Adenocard) B) Amiodarone (Cordarone) C) Diltiazem (Cardizem) D) Metoprolol E) Catheter ablation

ANSWER: E Catheter ablation is the most appropriate treatment for a patient with symptomatic Wolff-Parkinson-White syndrome (WPW). Catheter ablation has a very high immediate success rate (96%-98%). The most significant risk associated with the procedure is permanent atrioventricular block, which occurs in approximately 0.4% of procedures. Adenosine and amiodarone are used for the acute management of supraventricular tachycardia, but not for long-term management. Node-blocking medications such as diltiazem and metoprolol should not be used for the long-term treatment of WPW, due to the increased risk of ventricular fibrillation.

A 34-year-old female has posttraumatic stress disorder that started 6 months ago after she was a victim of an armed robbery. She also has nightmares, anxiety, hypervigilance, and some paranoia and has been reexperiencing the robbery. She has been receiving psychotherapy but would now like to start a medication. Which one of the following would be the most appropriate medication to start? A) Bupropion (Wellbutrin) B) Buspirone C) Lorazepam (Ativan) D) Risperidone (Risperdal) E) Venlafaxine (Effexor XR)

ANSWER: E It is important for patients with posttraumatic stress disorder (PTSD) to receive psychotherapy. For those who continue to have symptoms it is appropriate to initiate medications. SSRIs and venlafaxine are considered first-line medications for the treatment of PTSD. Bupropion and buspirone have not been found to be effective treatments. The use of benzodiazepine medications is not recommended because of the high risk of misuse. Antipsychotic medications would be appropriate in patients with disabling symptoms and behaviors that do not respond to psychological or drug treatment.

A patient comes to your outpatient clinic with a persistent migraine that she has been unable to treat effectively at home. The symptoms began several hours ago and are typical for her. She has already tried her usual treatments of ibuprofen, 800 mg, and rizatriptan (Maxalt), 10 mg, but they have not provided any relief. She took a second dose of rizatriptan 2 hours later without benefit. She is in significant pain, which is causing mild nausea, and she has photophobia and phonophobia. Which one of the following would be most appropriate at this point? A) Oral butalbital/acetaminophen/caffeine (Fioricet) B) Oral ergotamine/caffeine (Cafergot) C) Subcutaneous sumatriptan (Imitrex) D) Intramuscular morphine E) Intramuscular prochlorperazine

ANSWER: E Multiple studies have determined that parenteral antiemetics have benefits for the treatment of acute migraine beyond their effect on nausea. Most outpatient clinics do not have the ability to administer intravenous metoclopramide, which is the preferred treatment. However, most clinics do have the ability to administer intramuscular prochlorperazine or promethazine. Due to concerns about oversedation, misuse, and rebound, treatment with parenteral opiates is discouraged but may be an option if other treatments fail. Oral butalbital/acetaminophen/caffeine and oral ergotamine/caffeine have less evidence of success in the treatment of acute migraine. Sumatriptan is contraindicated within 24 hours of the use of rizatriptan.

A 28-year-old female who was recently diagnosed with polycystic ovary syndrome presents to discuss treatment of irregular menses. She has 2-3 menstrual periods every 6 months that happen at irregular times and can often produce heavy bleeding. She is not obese and has no significant acne or hirsutism. She does not desire pregnancy and her primary goal is to decrease the heavy menstrual bleeding. Which one of the following would be the most effective initial recommendation? A) Dietary modifications aimed at weight loss B) Clomiphene C) Metformin (Glucophage) D) Spironolactone (Aldactone) E) Placement of a levonorgestrel IUD (Mirena)

ANSWER: E Polycystic ovary syndrome can significantly affect multiple organ systems, and menstrual irregularities from anovulatory cycles are very common. Treatment should be based on the patient's goals and modified based on her desire for fertility. In a patient who is not interested in near-term fertility and whose goal is to control menstrual irregularities, a levonorgestrel IUD is most likely to reduce the frequency, duration, and volume of bleeding. Metformin is used to treat insulin resistance, dietary modifications are used to treat obesity, spironolactone can be used to treat hirsutism or acne, and clomiphene is used to induce ovulation and fertility.

A 35-year-old female comes to your office for evaluation of a tremor. During the interview you note jerking movements first in one hand and then the other, but when the patient is distracted the symptom resolves. Aside from the intermittent tremor the neurologic examination is unremarkable. She does not drink caffeinated beverages and takes no medications. Which one of the following is the most likely diagnosis? A) Parkinson's disease B) Cerebellar tremor C) Essential tremor D) Physiologic tremor E) Psychogenic tremor

ANSWER: E Psychogenic tremor is characterized by an abrupt onset, spontaneous remission, changing characteristics, and extinction with distraction. Cerebellar tremor is an intention tremor with ipsilateral involvement on the side of the lesion. Neurologic testing will reveal past-pointing on finger-to-nose testing. CT or MRI of the head is the diagnostic test of choice. Parkinsonian tremor is noted at rest, is asymmetric, and decreases with voluntary movement. Bradykinesia, rigidity, and postural instability are generally noted. For atypical presentations a single-photon emission CT or positron emission tomography may help with the diagnosis. One of the treatment options is carbidopa/levodopa. Patients who have essential tremor have symmetric, fine tremors that may involve the hands, wrists, head, voice, or lower extremities. This may improve with ingestion of small amounts of alcohol. There is no specific diagnostic test but the tremor is treated with propranolol or primidone. Enhanced physiologic tremor is a postural tremor of low amplitude exacerbated by medication. There is usually a history of caffeine use or anxiety.

A 17-year-old female presents for acute care after tripping and falling on her right knee when stepping off her electric scooter. An examination does not reveal gross deformity or notable effusion. Range of motion in the right knee is limited to 100° of flexion. There is tenderness over the proximal tibia. She can only take two steps before being unable to bear weight on the knee due to pain. Which one of the following findings on the patient's history and examination should prompt you to order immediate knee radiographs? A) Her age B) Her sex C) Bony tenderness over the proximal tibia D) Limited range of motion E) The inability to take more than two steps

ANSWER: E Several decision support tools can help guide the decision to order imaging of an injured knee, such as the Ottawa Knee Rule, the Pittsburgh Knee Rule, and American College of Radiology (ACR) criteria. The inability to take four or more steps immediately after an injury or in the emergency setting is an indication for radiography in all three rules. Age is an indication for radiography in acute knee pain in patients over 55 years of age according to the Ottawa rule, or under 12 or over 50 years of age according to the Pittsburgh rule. The patient's sex does not factor into the criteria for imaging. Bony tenderness is an indication for imaging according to the ACR and Ottawa rules, but only if isolated over the proximal fibula or over the patella without other bony tenderness. The inability to flex the knee to 90° is also an indication for imaging according to the ACR and Ottawa rules.

A 57-year-old female who spends a lot of time working in her yard has pigmented areas on sun-exposed skin. She has several of these and would like to address them for cosmetic reasons. An examination is consistent with solar lentigines. One of these on her posterior leg has increased in size more rapidly. Which one of the following would be the most appropriate next step for the lesion on the posterior leg? A) Topical hydroquinone (Lustra) B) A topical retinoid C) Cryotherapy D) Laser therapy E) A biopsy

ANSWER: E Solar lentigines occur on sun-exposed skin and are known commonly as liver spots. A biopsy should be performed if they grow rapidly, change rapidly, are painful, itch, bleed easily, heal poorly, or have an atypical or suspicious appearance. If no suspicious changes or symptoms are present there are various options for treatment, including topical therapy with hydroquinone or retinoids, or ablative therapy with chemical peels, cryotherapy, intense pulsed light, or laser therapies.

For urinary stones 5-10 mm in diameter, which one of the following has been proven effective in facilitating expulsion from the distal ureter? A) Ciprofloxacin (Cipro) B) Naproxen C) Nitrofurantoin (Macrodantin) D) Promethazine E) Tamsulosin (Flomax)

ANSWER: E Tamsulosin promotes passage of ureter stones that are 5-10 mm in diameter. The number needed to treat is five patients to cause the expulsion of one stone (SOR B). There was no difference in the percentage of patients passing stones smaller than 5 mm when comparing tamsulosin to placebo, as these stones have a high rate of spontaneous passage without any intervention. Naproxen and promethazine are sometimes used for the management of pain and nausea associated with stones, but they have not been shown to facilitate stone expulsion. Ciprofloxacin and nitrofurantoin are used to treat urinary tract infections but have not been shown to facilitate stone expulsion.

A 38-year-old female has a 4-year history of intermittent abdominal pain occurring several days per week. She went to an urgent care clinic over the weekend and has brought her laboratory results for you to review. A comprehensive metabolic panel, CBC, and TSH level are all normal. She states that her stools have become harder and difficult to pass, with frequent straining and pain. She reports now having a bowel movement only 2-3 times a week. She tells you that she tried psyllium but it resulted in abdominal bloating. She is concerned because she has been missing work because of her symptoms. Her past and family medical histories are otherwise negative. Her vital signs are normal and her weight is unchanged from a previous wellness visit 6 months ago. An abdominal examination is notable for generalized pain with deep palpation. She does not have any masses, hepatosplenomegaly, or rebound tenderness. A rectal examination is also normal. Which one of the following is the most likely diagnosis? A) Celiac disease B) Colon cancer C) Hyperparathyroidism D) Inflammatory bowel disease E) Irritable bowel syndrome

ANSWER: E The Rome IV criteria for irritable bowel syndrome are symptom-based diagnostic criteria used to identify patients with irritable bowel syndrome (IBS). According to these criteria, IBS is defined as recurrent abdominal pain at least 1 day per week for at least 3 months. At least two of the following must be present: abdominal pain related to defecation, a change in stool form, and/or a change in stool frequency. The diagnosis can be made from the patient history, a physical examination, and a minimal laboratory evaluation. This patient meets the criteria. This patient does not have any "alarm" symptoms (a positive family history of colorectal cancer, rectal bleeding in the absence of documented bleeding hemorrhoids or anal fissures, unintentional weight loss, or anemia) so colon cancer would be unlikely. Celiac disease and inflammatory bowel disease are unlikely in a patient with constipation-predominant IBS. Serologic and inflammatory marker testing would be warranted in patients with predominant diarrhea or mixed types of IBS. Hyperparathyroidism does not usually present with this clinical picture, and calcium levels would likely be elevated.

A 43-year-old female presents to your office with a 3-month history of left low back and posterior hip pain. She does not recall an injury but says she was very active during a move to a new home prior to the onset of the pain. An examination reveals that her gait, lower extremity strength, straight leg resistance, and hip and knee range of motion are normal. A log roll test is also normal. A flexion, abduction, external rotation (FABER) test produces posterior pain. Which one of the following is the most likely diagnosis? A) Femoroacetabular impingement B) Greater trochanteric pain syndrome C) Osteoarthritis D) Piriformis syndrome E) Sacroiliac joint dysfunction

ANSWER: E The cause of hip pain is generally determined from the patient's history and physical examination. A positive flexion, abduction, external rotation (FABER) test that produces pain at the sacroiliac joint, lumbar spine, and posterior hip is associated with sacroiliac joint dysfunction. The log roll test involves passive supine internal and external rotation of the hip. When this test is positive for pain it is associated with piriformis syndrome. While femoroacetabular impingement may be associated with a positive FABER test, it would produce pain in the groin. Greater trochanteric pain syndrome results in lateral hip pain rather than posterior pain. Osteoarthritis is usually associated with a limited range of motion and groin pain.

An 83-year-old female tells you she has had pain in her knees for the past 15 years, although she has never discussed this problem with you before. After a thorough history and physical examination you diagnose osteoarthritis. Which one of the following would you recommend as the most appropriate first-line treatment to reduce this patient's pain? A) Vitamin D B) Glucosamine and chondroitin C) Low-dose tramadol (Ultram) D) Corticosteroid injection of the knees E) Supervised exercise

ANSWER: E The goals of osteoarthritis therapy are to minimize pain and improve function. The American Academy of Orthopedic Surgery and the American College of Rheumatology have agreed that first-line treatment includes aerobic exercise, resistance training, and weight loss. For patients with osteoarthritis of the knee, supervised exercise was found to reduce pain and improve physical function and quality of life (SOR A). Vitamin D is not currently recommended, and glucosamine and chondroitin are less effective than placebo. Initial pharmacotherapy includes full-strength acetaminophen, or oral or topical NSAIDs. If patients have an inadequate response to these agents other treatments to consider include tramadol, other opioids, duloxetine, or intra-articular injections with corticosteroids or hyaluronate.

A 25-year-old male presents to your office with a nearly 3-month history of a persistent, mildly productive cough. He does not have any fevers, myalgias, or night sweats but is often congested. He has never smoked and is normotensive. An examination is notable for a normal weight and vital signs, clear lungs, a mildly erythematous oropharynx, and pale, edematous nasal mucosa. There is no lymphadenopathy. Which one of the following is the most likely cause of this patient's chronic cough? A) Bronchogenic carcinoma B) Chronic aspiration C) Obstructive sleep apnea D) Tuberculosis E) Upper airway cough syndrome

ANSWER: E The most common causes of chronic cough in adults include upper airway cough syndrome, tobacco use, GERD, asthma, and ACE inhibitor use. The physical examination of this patient is most consistent with upper airway cough syndrome, previously referred to as postnasal drip syndrome. Given the patient's lack of tobacco use and normal blood pressure, bronchogenic carcinoma and obstructive sleep apnea are less likely. There are no risk factors in this patient's history to suggest chronic aspiration or tuberculosis.

A 50-year-old male presents with difficulty straightening his left ring finger. Examination of the affected hand reveals a nodule of the palmar aponeurosis and associated fibrous band that limits full extension of the fourth finger. He is unable to fully extend both the metacarpophalangeal (MCP) joint and the proximal interphalangeal (PIP) joint, with MCP and PIP contractures estimated at 40° and 20°, respectively. Which one of the following would be the most appropriate management strategy? A) Observation until the PIP contracture is >90° B) Serial intralesional injection with a corticosteroid C) Cryosurgery of the fibrous nodule D) Referral for physical therapy E) Referral for surgical release of the contracture

ANSWER: E This patient has Dupuytren's disease with a contracture of the affected finger. Surgical release is indicated when the metacarpophalangeal joint contracture reaches 30° or with any degree of contracture of the proximal interphalangeal joint. Intralesional injection may reduce the need for later surgery in a patient with grade 1 disease, but not if there is a contracture. There is no evidence to support the use of physical therapy or cryosurgery.

A 48-year-old male with an 8-week history of the gradual onset of nonradiating, worsening left heel pain sees you for follow-up. He started running on his neighborhood streets 3 months ago to train for a 10K race. The pain limits his training significantly. His symptoms improved when he began taking ibuprofen and took 2 weeks off from running a month ago. A radiograph of the left foot 4 weeks ago was normal. There is no other pain and he feels well otherwise. On examination you note that his left heel is slightly swollen compared to the right and very tender when squeezed on the sides. The anterior aspect and Achilles tendon insertion of the heel are nontender. There is no erythema or warmth and the remainder of the left lower extremity examination is normal. His vital signs are normal. Which one of the following would you recommend at this point? A) A C-reactive protein level B) Nerve conduction velocity testing C) A repeat radiograph D) Ultrasonography E) MRI

ANSWER: E This patient has a calcaneal stress fracture as suggested by the history of increased running on a hard surface, improvement with rest, and a positive calcaneal squeeze on examination. A delay in diagnosis increases the risk of delayed union. MRI is the preferred imaging modality because radiographs often do not detect a calcaneal stress fracture. A C-reactive protein level could be indicated if there were symptoms or signs of infection or autoimmune illness. The clinical picture does not suggest a neurologic condition, so nerve conduction velocity testing is not appropriate. While there are some case reports of the diagnosis of stress fractures using ultrasonography, this is not the preferred imaging method.

A 46-year-old female with a history of hyperthyroidism controlled with methimazole (Tapazole), 10 mg daily, returns to your office after an absence of several years. She has new symptoms of palpitations, heat intolerance, and hoarseness. A physical examination reveals an enlarged thyroid and a radioactive iodine uptake scan shows accumulation of tracer in multiple areas. Which one of the following is the appropriate definitive treatment for this patient? A) Methimazole alone, 20 mg daily B) Methimazole, 20 mg daily, plus propranolol, 80 mg twice daily C) Propylthiouracil alone, 50 mg 3 times daily D) Radioactive iodine E) Thyroidectomy

ANSWER: E This patient has a medical history, physical examination, and radioactive iodine uptake scan consistent with toxic multinodular goiter, which is the second most common cause of hyperthyroidism in the United States. Although the addition of propranolol and an increase in methimazole may control her palpitations and other symptoms of hyperthyroidism, these measures will not permanently eliminate the problem. Radioactive iodine ablation and thyroidectomy with subsequent thyroid hormone replacement are both appropriate treatments for toxic multinodular goiter, but thyroidectomy is indicated for this patient because she has compressive symptoms from the goiter itself.

A 58-year-old female presents to your office to discuss a new diagnosis of COPD. She has a 40-pack-year smoking history, and she quit using tobacco 18 months ago. Spirometry performed last week showed an FEV1/FVC ratio of 0.62 with an FEV1 that is 75% of predicted. She was first treated for an exacerbation last month and has never required hospitalization for any respiratory illness. You administer a COPD assessment test and she rates herself at 16/40, which is a moderately high score. Which one of the following inhaled medications would be appropriate to initiate today? A) Albuterol (Proventil, Ventolin) B) Fluticasone (Flovent) C) Fluticasone/salmeterol (Advair) D) Ipratropium (Atrovent) E) Tiotropium (Spiriva)

ANSWER: E This patient has airflow obstruction consistent with a diagnosis of COPD, evidenced by an FEV1/FVC ratio <0.7. An evidence-based standard approach to COPD is found in the annual guidelines published by the Global Initiative for Chronic Obstructive Lung Disease (GOLD). According to the 2019 report, this patient's FEV1 of 75% of predicted puts her in the GOLD grade 2 (moderate) category of airflow limitation. Her symptoms and risk of exacerbations places her in GOLD group B. Patients in this category have symptoms that bother them regularly without having frequent COPD exacerbations. Patients in this category benefit from daily use of long-acting bronchodilators, either long-acting -agonists (LABAs) or long-acting muscarinic agents (LAMAs). Long-acting agents such as tiotropium (a LAMA) or salmeterol (a LABA) are preferred over the short-acting agents ipratropium and albuterol for patients in this category of disease severity. Patients with persistent symptoms while using one of these agents may benefit from a combination of a LABA and a LAMA. Monotherapy with inhaled corticosteroids has not been shown to improve mortality or prevent a long-term decline in FEV1. The combination of an inhaled corticosteroid plus a LABA has evidence of superiority over either agent alone for improving lung function and health status, and for reducing exacerbations in patients who have more severe disease. No improvement in all-cause mortality has been noted.

A 30-year-old gravida 1 para 0 develops erythematous patches with slightly elevated scaly borders during her first trimester. There was a 2-cm herald patch 2 weeks before multiple smaller patches appeared. The rash on the back has a "Christmas tree" pattern. She has not had any prenatal laboratory work. This condition is associated with A) no additional pregnancy risk B) a small-for-gestational-age newborn C) congenital cataracts D) multiple birth defects E) spontaneous abortion

ANSWER: E This patient has classic pityriasis rosea. This is generally a benign disease except in pregnancy. The epidemiology and clinical course suggest an infectious etiology. Pregnant women are more susceptible to pityriasis rosea because of decreased immunity. Pityriasis rosea is associated with an increased rate of spontaneous abortion in the first 15 weeks of gestation. It is not associated with an increased risk for a small-for-gestational-age newborn, congenital cataracts, or multiple birth defects.

A 48-year-old female presents to your office for follow-up of painful rectal bleeding with bowel movements. She has increased her fiber consumption and is using a stool softener as you recommended at her last visit 2 weeks ago. She reports that her pain has worsened since yesterday and she is very uncomfortable. An examination reveals a firm and tender right posterior hemorrhoid below the dentate line. Which one of the following would be the most appropriate next step in providing relief for this thrombosed hemorrhoid? A) Topical corticosteroids B) Bioflavonoid supplements C) Oral antibiotics D) Rubber band ligation E) Excision

ANSWER: E This patient has tried first-line treatment for hemorrhoids with increased fiber intake but has returned with symptoms of a thrombosed external hemorrhoid. Office-based surgical excision of the thrombosed external hemorrhoid within 2-3 days of symptom onset may provide significant symptomatic relief (SOR B) and result in a lower risk of recurrence. While conservative treatment with topical therapies such as corticosteroids may be helpful, symptomatic relief is prolonged with excision of the thrombosed hemorrhoid. Bioflavonoids are used outside the United States for symptomatic treatment of hemorrhoids but evidence is lacking and they are not approved by the FDA for this use. Oral antibiotic therapy has no role in the treatment of thrombosed external hemorrhoids but may be beneficial in treating an abscess, which would present with a gradual onset of pain and a fluctuant rectal mass. Rubber band ligation is an appropriate treatment for grades I-III internal hemorrhoids (SOR A).

A 57-year-old long-time patient presents for an annual wellness visit. He has a BMI of 46 kg/m2 , type 2 diabetes, obstructive sleep apnea, hypertension, and cardiomyopathy. He is adherent to his medication regimen and CPAP therapy, and these problems have been controlled. As you discuss lifestyle modification recommendations, he tells you that he wants to address his obesity this year. You have counseled him on low-carbohydrate and low-calorie diets over the years. He has had difficulty following these recommendations due to his work obligations. He states that he has tried multiple fad diets, none of which have been helpful. Which one of the following would be the most effective management? A) Worksite intervention B) Exercise therapy C) Behavioral therapy D) Pharmacotherapy E) Bariatric surgery

ANSWER: E This patient presents with morbid obesity complicated by several obesity-related conditions. Bariatric surgery has been shown to result in greater weight loss compared to nonsurgical interventions (SOR A). It has also been shown to be highly effective in treating obesity-related comorbid conditions such as diabetes mellitus (SOR A). Patients with a BMI 40 kg/m2 should be referred for consideration of bariatric surgery (SOR B). While worksite intervention, exercise therapy, behavioral therapy, and pharmacotherapy are appropriate treatments for obesity, these interventions are all less effective than bariatric surgery.

A 58-year-old female presents to your office after being seen in the emergency department last weekend for her first episode of renal colic. After undergoing CT urography she passed a calcium phosphate kidney stone. Which one of the following medications in her current regimen places her at higher risk for kidney stone formation? A) Escitalopram (Lexapro) B) Levothyroxine (Synthroid) C) Lisinopril (Prinivil, Zestril) D) Metformin (Glucophage) E) Topiramate (Topamax)

ANSWER: E Topiramate increases the risk of kidney stones. It is a carbonic anhydrase inhibitor, which induces a metabolic acidosis that leads to hypercalciuria and the formation of calcium phosphate stones. The risk of kidney stones is not increased by escitalopram, levothyroxine, lisinopril, or metformin.

Which one of the following treatments has been shown to improve the quality of life for a patient with tinnitus? A) Antidepressant therapy B) Ginkgo biloba C) Niacin D) Vitamin B12 E) Cognitive-behavioral therapy

ANSWER: E Treatments to reduce awareness of tinnitus and tinnitus-related distress include cognitive-behavioral therapy, acoustic stimulation, and educational counseling. No medications, supplements, or herbal remedies have been shown to substantially reduce the severity of tinnitus.

Which disorder of sexual development is associated with an increased risk of endometrial cancer, celiac disease, and structural heart defects? A) Complete androgen insensitivity syndrome B) Complete gonadal dysgenesis C) Congenital adrenal hyperplasia D) Klinefelter syndrome E) Turner syndrome

ANSWER: E Turner syndrome is associated with higher risks of endometrial cancer, celiac disease, and structural heart defects. Physicians providing primary care to patients with Turner syndrome should recognize these increased risks and provide testing for celiac disease and serial screening for cardiac and/or aortic defects, and should also have a low threshold for evaluating any abnormal uterine bleeding. Complete androgen insensitivity syndrome, gonadal dysgenesis, congenital adrenal hyperplasia, and Klinefelter syndrome carry increased risks of other conditions, but not endometrial cancer, celiac disease, or structural heart defects.

Immunotherapy can prevent recurrence of severe allergic reactions triggered by which one of the following common allergens? A) Penicillin B) Poison ivy C) Shellfish D) Tree nuts E) Wasp stings

ANSWER: E Wasp stings, penicillin, shellfish, and tree nuts are all relatively common causes of severe allergic reactions, including anaphylaxis. Immunotherapy is available and recommended for the stings of insects, including wasps and bees, to prevent recurrent anaphylaxis (SOR B). It is not available for penicillin, poison ivy, shellfish, or tree nut allergens. Poison ivy typically causes a type 4 hypersensitivity contact dermatitis rather than anaphylaxis.

A 67-year-old male who recently had an ST-elevation myocardial infarction (STEMI) and underwent placement of a circumflex artery stent presents for hospital follow-up. His past medical history also includes hypertension, obesity, stage 3a chronic kidney disease, and coronary artery disease. He is feeling well today but his cardiologist told him he probably has diabetes mellitus and should see you to discuss treatment options. The cardiologist told the patient that there are new medications for diabetes that will help lower his risk for future myocardial infarctions and he is interested in starting one of these. A physical examination is unremarkable. The patient weighs 145 kg (320 lb) and his BMI is 44 kg/m2 . His blood pressure is 132/78 mm Hg and his heart rate is 65 beats/min. Laboratory studies from the hospital include the following: Hemoglobin A1c 8.2% TSH normal Sodium 141 mEq/L (N 136-142) Potassium 4.5 mEq/L (N 3.5-5.0) Chloride 100 mEq/L (N 98-107) Bicarbonate 30 mEq/L (N 22-29) BUN 26 mg/dL (N 8-23) Creatinine 1.34 mg/dL (N 0.6-1.2) Glomerular filtration rate 51 mL/min/1.73 m2 Based on American Diabetes Association guidelines, in addition to lifestyle modifications, which one of the following would be most appropriate at this time? A) No pharmacologic therapy B) Empagliflozin (Jardiance) C) Insulin glargine (Lantus) D) Liraglutide (Victoza) E) Metformin (Glucophage)

ANSWER: E With the advent of new medications to treat diabetes mellitus, including many medications that help lower cardiovascular risk, it is tempting to begin treatment with these medications. Unless there are contraindications, however, metformin is still the initial medication of choice for patients with newly diagnosed diabetes. If this patient's hemoglobin A1c were 10%, it would be reasonable to start insulin therapy. If it becomes necessary to add an additional antidiabetic agent it would be reasonable to consider a GLP-1 receptor agonist or a DPP-4 inhibitor in addition to metformin at that time, given the patient's history of coronary artery disease.

A 36-year-old male went skiing last year for the first time and when he made it to the top of the mountain he developed a headache, nausea, and dizziness, but no respiratory difficulty. That night he had difficulty sleeping. He asks for your recommendation on preventing a recurrence of the problem when he goes skiing again this year. Which one of the following medications would you recommend he start the day before his ascent and continue until his descent is complete? A) Acetazolamide (Diamox Sequels) B) Aspirin C) Dexamethasone (Decadron) D) Tadalafil (Adcirca) E) Zolpidem (Ambien

Acetazolamide is the preferred agent for preventing acute mountain sickness (AMS). Multiple trials have demonstrated its efficacy in preventing AMS. Dexamethasone is a first-line treatment for acute mountain sickness of any severity but is a second-line drug for prevention because of its side-effect profile. Tadalafil is advised as a second-line treatment after nifedipine for the prevention and treatment of high-altitude pulmonary edema. Zolpidem may help with sleep but not AMS, and aspirin is not recommended for prevention of AMS.

A 67-year-old male presents with a 12-hour history of the inability to urinate. He also reports mild symptoms of benign prostatic hyperplasia that he has tolerated for the last couple of years. Today he has significant discomfort in his suprapubic area, and his bladder is palpable. He has not had any fever or recent painful urination. Ultrasonography shows 300 cc of retained urine in the bladder. Which one of the following would be the initial treatment for this patient's acute urinary retention? A) An oral -blocker followed by urethral catheter placement if he is unable to void within 24 hours B) Urethral catheter placement with immediate removal after draining the bladder C) Urethral catheter placement followed by continuous drainage for 3 days D) Suprapubic catheter placement E) Transurethral resection of the prostate

Acute symptomatic urinary retention should be treated with immediate urethral catheterization. The catheter should be left in place for 3 days, followed by a voiding trial. During this time the patient should be evaluated for underlying causes of the retention, such as infection, and prescribed or over-the-counter medications. Starting an -blocker while the catheter is in place should be considered, but initial treatment with an -blocker without catheterization is not indicated. Suprapubic catheter placement should only be used when urethral catheterization is unsuccessful. Transurethral resection of the prostate may be necessary later but is not the initial treatment.

A patient with moderately severe Alzheimer's disease has been taking quetiapine (Seroquel), 50 mg daily at bedtime, to manage behavioral symptoms related to the dementia. The patient's symptoms have been stable on the quetiapine for 6 months. The patient's spouse is the primary caregiver and is not aware of any adverse effects. The patient does not have a history of other psychiatric diagnoses such as schizophrenia or bipolar disorder. Which one of the following would be the most appropriate intervention at this time? A) Continue quetiapine at the current dosage B) Reduce quetiapine to a lower maintenance dosage C) Taper the quetiapine dosage with the goal of stopping it D) Start diphenhydramine (Benadryl) while tapering quetiapine with the goal of stopping it E) Start lorazepam (Ativan) while tapering quetiapine with the goal of stopping it

Behavioral and psychological symptoms of dementia include delusions, hallucinations, aggression, and agitation. Antipsychotics are frequently used for treatment of these symptoms and are continued indefinitely. For patients who have been taking antipsychotics for 3 months and whose symptoms have stabilized, or for patients who have not responded to an adequate trial of an antipsychotic, it is recommended that the drug be tapered slowly (SOR B). Physicians should collaborate with the patient and caregivers when deciding whether to use an antipsychotic. This is recommended because antipsychotic medications have adverse effects, including an increased overall risk of death, cerebrovascular events, extrapyramidal symptoms, gait disturbances, falls, somnolence, edema, urinary tract infections, weight gain, and diabetes mellitus. The risk of these harms increases with prolonged use in the elderly. One tapering method to consider is to reduce the daily dose to 75%, 50%, and 25% of the original dose every 2 weeks until stopping the medication. This reduction pace can be slowed for some patients. Diphenhydramine and lorazepam are on the Beers list of potentially inappropriate medications to use in older patients and would not be recommended.

A 30-year-old female presents with a 5-day history of subjective fever and malaise. She does not have a thermometer at home but has felt alternately warm and chilled. She has felt generally unwell and is sleeping more than usual. She has had a decreased appetite but has been drinking fluids without difficulty. She does not have a runny nose, cough, headache, abdominal pain, vomiting, diarrhea, joint pain, rash, or pain with urination. Her medical history includes substance use disorder and she takes buprenorphine/naloxone (Suboxone). She smokes one pack of cigarettes daily, has 0-2 alcoholic drinks daily, and began using intravenous heroin again 1 week ago. An examination reveals a blood pressure of 112/68 mm Hg, a pulse rate of 88 beats/min, a respiratory rate of 16/min, a temperature of 38.9°C (102.0°F), and an oxygen saturation of 95% on room air. The patient appears fatigued and uncomfortable but nontoxic. Her heart has a regular rate and rhythm with no murmur. Her lungs are clear to auscultation bilaterally and her abdomen is soft and nontender. There is no swelling or redness in the extremities and a skin examination reveals no rashes or lesions. Which one of the following would be most important at this point? A) A viral swab B) An antinuclear antibody level C) Blood cultures D) An erythrocyte sedimentation rate E) A chest radiograph

C. A patient who uses intravenous drugs and has a fever without a clear source must be evaluated for infectious endocarditis (IE). The first step in this evaluation is to obtain blood cultures. Although this patient might have a less serious condition, it is critical to evaluate for bacteremia in this situation. If the concern for IE is high, blood cultures should be obtained and antibiotics may be started while waiting for results and arranging for urgent echocardiography. IE in people who inject drugs is more likely to be right-sided, specifically involving the tricuspid valve. Right-sided IE is less frequently associated with systemic findings of endocarditis such as Janeway lesions or Roth spots. Patients often do not have a heart murmur.

A 4-year-old female is brought to your office because of a history of constipation over the past several months. Her mother reports that the child has 1-2 bowel movements per week composed of small lumps of hard stool. She strains to have the bowel movements, and they are painful. The child eats normally like her two siblings. Which one of the following would be most effective at this time? A) Daily fiber supplements B) Lactulose C) Magnesium hydroxide (Milk of Magnesia) D) Polyethylene glycol (MiraLAX) E) Senna

D. This patient presents with symptoms compatible with functional constipation. Daily use of polyethylene glycol (PEG) solution has been found to be more effective than lactulose, senna, or magnesium hydroxide in head-to-head studies. Evidence does not support the use of fiber supplements in the treatment of functional constipation. No adverse effects were reported with PEG therapy at any dosing regimen. Low-dose regimens of PEG are 0.3 g/kg/day and high-dose regimens are up to 1.0-1.5 g/kg/day.

A healthy 35-year-old female presents to your office to discuss an upcoming trip to Bangladesh. She currently feels well and has no health problems. She is a nurse and will be traveling with a church group to work in a clinic for 1 month. This area is known to have a high prevalence of tuberculosis (TB). She is worried about contracting TB while she is there and asks for recommendations regarding TB screening. She had a negative TB skin test about 1 year ago at work. A TB skin test today is negative. Assuming she remains asymptomatic, which one of the following would you recommend? A) Prophylactic treatment with isoniazid starting 1 month prior to departure and continuing throughout her trip B) Prophylactic treatment with rifampin (Rifadin) starting 1 month prior to departure and continuing throughout her trip C) A repeat TB skin test 2 months after she returns D) A chest radiograph 2 months after she returns E) An interferon-gamma release assay (IGRA) 6 months after she returns

Individuals who travel internationally to areas with a high prevalence of tuberculosis (TB) are at risk for contracting the disease if they have prolonged exposure to individuals with TB, such as working in a health care setting. The CDC recommends either a TB skin test or an interferon-gamma release assay prior to leaving the United States. If the test is negative, the individual should repeat the testing 8-10 weeks after returning. A chest radiograph in asymptomatic individuals or prophylactic treatment at any point is not recommended. Isoniazid and rifampin are options for treatment of latent TB.

A 25-year-old landscaper presents with a 1-day history of pain and swelling in his left hand. Three days ago he sustained a puncture wound in the palm of the hand when he was stuck by a large thorn. He has generalized achiness and chills in addition to the hand pain. He has no significant past medical history. On examination the patient's temperature is 37.9°C (100.2°F). His left third finger is diffusely swollen, erythematous, and held in flexion. There is tenderness along the third tendon in the palm. A radiograph of the hand is negative for a fracture or foreign body. Which one of the following would be the most appropriate next step? A) Amoxicillin/clavulanate (Augmentin), 875/125 mg twice daily for 10 days B) Incision and drainage in the office with a culture of the wound C) A hand splint and nonurgent referral to an orthopedic surgeon D) Urgent MRI of the hand E) Urgent surgical consultation

Item 145 ANSWER: E Pyogenic flexor tenosynovitis usually develops 2-5 days after a penetrating hand injury. The flexor tendon sheath has a poor vascular supply and the synovial fluid is a prime growth medium for bacteria. Flexor tenosynovitis is a clinical diagnosis characterized by the four "Kanavel" signs: pain with passive extension, tenderness with palpation of the tendon sheath, flexed position of the involved finger, and fusiform swelling of the finger. Treatment includes prompt intravenous antibiotics and surgical debridement and irrigation. Flexor tenosynovitis requires urgent surgical consultation and treatment. Patients with suspected flexor tenosynovitis should be seen by a surgeon within 72 hours of symptom onset (SOR C). Oral antibiotics and splinting of the hand alone are insufficient treatments for the condition. Incision and drainage would also not be sufficient to clear the infection. Ordering MRI can unnecessarily delay surgical consultation, although the surgeon may obtain one to guide treatment.

A 24-year-old female sees you for a routine wellness examination. She tells you that she and her husband would like to have a baby in the near future. Which one of the following supplements should you recommend? A) Folic acid B) Vitamin B6 C) Vitamin B12 D) Vitamin C E) Vitamin D

Item 153 ANSWER: A Women of reproductive age should take a daily supplement of 0.4-0.8 mg of folic acid starting before conception in order to reduce the risk of neural tube defects. A higher folic acid dosage of 4 mg daily starting 1 month prior to conception is recommended for women who have a high risk of pregnancy complicated by a neural tube defect. This includes women with a previous pregnancy with a neural tube defect, a personal or family history of neural tube defects, type 1 diabetes, or a seizure disorder. Vitamin B6, vitamin B12, vitamin C, and vitamin D do not require supplementation prior to conception.

According to the most recent American College of Cardiology/American Heart Association guidelines, hypertension is defined as a blood pressure reading greater than A) 120/80 mm Hg B) 130/80 mm Hg C) 135/85 mm Hg D) 140/90 mm Hg E) 150/90 mm Hg

The latest American College of Cardiology/American Heart Association guidelines promote a radical change in the management of hypertension, which they now define as a blood pressure 130/80 mm Hg. Elevated blood pressure is defined as a systolic pressure of 120-129 mm Hg and a diastolic pressure <80 mm Hg. A blood pressure of 130-139/80-89 mm Hg is classified as stage 1 hypertension and a systolic pressure 140 mm Hg or a diastolic pressure 90 mm Hg is classified as stage 2 hypertension.

A 78-year-old male is brought to your office by his daughter. She is concerned that her father is no longer attending his weekly cribbage and bingo games, has stopped bathing regularly, and is eating much less. Which one of the following would be most appropriate at this time? A) Administering the CAGE screening questionnaire B) Administering the PHQ-9 screening questionnaire C) A trial of megestrol D) A trial of nortriptyline (Pamelor) E) MRI of the brain

This elderly patient is exhibiting classic signs of depression. The PHQ-2 has a similar sensitivity to the PHQ-9, but the PHQ-9 has a higher specificity in diagnosing depression (91%-94% compared to 78%-92%) and can assist in diagnosing depression. In addition to the PHQ-2 and PHQ-9 there are specific screening tools for use in the elderly population, including the Geriatric Depression Scale and the Cornell Scale for Depression in Dementia. Somatic issues and dementia can make it more difficult to screen for and diagnose depression in this population. The CAGE questionnaire screens for substance abuse. Megestrol is used to stimulate the appetite, but in this patient the appetite symptoms are likely secondary to depression so treating the depression would be a more appropriate starting point. The tricyclic nortriptyline is used to treat depression but is not first-line therapy, especially in the elderly. In general, a more extensive medical history and a physical examination are indicated before ordering MRI of the brain. Ref: Maur

A 13-year-old female is brought to your office by her father for evaluation of left knee pain. It has been slowly worsening over the past several weeks and she does not recall any inciting injury or event. She plays soccer and participates in practice 3 days a week with games on the weekends. She is finding it increasingly difficult to participate fully due to pain. She notes some swelling and tenderness just below her kneecap. She has tried icing the area after activity and has recently started taking ibuprofen with moderate relief of the pain. On examination you note prominence of the tibial tubercle with tenderness to palpation. The remainder of the knee examination is unremarkable. Which one of the following is indicated at this time to further assess this condition? A) No imaging B) Radiography C) Ultrasonography D) CT E) MRI

This patient has Osgood-Schlatter disease, a common cause of knee pain in active children with immature skeletons. It occurs as a result of abnormal development, injury, or overuse of the growth plate and the surrounding ossification centers. Osteochondrosis is a more general term for this condition, which can occur at growth plates around other joints, including the hip, foot, elbow, and back. In Osgood-Schlatter disease repetitive traction of the patellar tendon on the tibial tubercle ossification center leads to inflammation and pain. Imaging is not required to make the diagnosis when patients present with typical symptoms and physical examination findings. Radiographs may be obtained if there is uncertainty about the diagnosis. Radiographic findings in Osgood-Schlatter disease include soft-tissue swelling and fragmentation of the tibial tubercle. This condition is self-limited and treatment consists of activity modification and the use of acetaminophen or NSAIDs. An orthopedic referral is indicated if symptoms persist in a patient with a mature skeleton.

A 49-year-old African-American male sees you for a routine health maintenance examination. His past medical history is significant for sarcoidosis. He has noticed some fatigue and shortness of breath over the last several months, but he is asymptomatic today. His vital signs are normal except for an irregular pulse. An EKG performed in the office is shown below. Which one of the following would be most appropriate at this point? A) Observation only B) Amiodarone (Cordarone) C) Apixaban (Eliquis) D) Metoprolol succinate (Toprol-XL) E) A cardiology assessment for placement of a pacemaker

This patient's EKG shows type II second degree (Mobitz type II) atrioventricular (AV) block. Conduction disturbances are one of the most common manifestations of cardiac sarcoidosis. In addition to AV block, supraventricular and ventricular arrhythmias can be seen. Mobitz type II AV block is treated with pacemaker placement. Metoprolol could be used for treatment of nonsustained ventricular tachycardia, apixaban for anticoagulation in patients with atrial fibrillation or atrial flutter, and amiodarone for either supraventricular or ventricular tachycardias


Related study sets

Ch. 16 nursing mgmt during PP period

View Set

Chapter 6 Organizational Behavior

View Set

MIS 309 Posey Ole Miss Test 1 Excel Terms + CH 1-6, MIS 309 posey test 2, MIS 309 Posey Chapters 10-12

View Set

Chapter 3: Theoretical Models of Personality Development

View Set

Management of Patients with Oral and Esophageal Disorders

View Set

Business Law Test 2 Cases and Business Questions

View Set

08-02-02 Sets - set addition/removal

View Set

Intro to Info Systems (Homework #4)

View Set